Eureka 2002

185
CONTEÚDO XXIII OLIMPÍADA BRASILEIRA DE MATEMÁTICA 2 Problemas e Soluções da Primeira Fase XXIII OLIMPÍADA BRASILEIRA DE MATEMÁTICA 11 Problemas e Soluções da Segunda Fase XXIII OLIMPÍADA BRASILEIRA DE MATEMÁTICA 21 Problemas e Soluções da Terceira Fase XXIII OLIMPÍADA BRASILEIRA DE MATEMÁTICA 41 Problemas e Soluções da Primeira Fase - Nível Universitário XXIII OLIMPÍADA BRASILEIRA DE MATEMÁTICA 46 Problemas e Soluções da Segunda Fase - Nível Universitário XXIII OLIMPÍADA BRASILEIRA DE MATEMÁTICA 57 Premiados AGENDA OLÍMPICA 61 COORDENADORES REGIONAIS 62

description

XXIII OLIMPÍADA BRASILEIRA DE MATEMÁTICA 2 Problemas e Soluções da Primeira Fase XXIII OLIMPÍADA BRASILEIRA DE MATEMÁTICA 11 Problemas e Soluções da Segunda Fase XXIII OLIMPÍADA BRASILEIRA DE MATEMÁTICA 21 Problemas e Soluções da Terceira Fase CONTEÚDO XXIII OLIMPÍADA BRASILEIRA DE MATEMÁTICA 46 Problemas e Soluções da Segunda Fase - Nível Universitário XXIII OLIMPÍADA BRASILEIRA DE MATEMÁTICA 41 Problemas e Soluções da Primeira Fase - Nível Universitário

Transcript of Eureka 2002

Page 1: Eureka 2002

CONTEÚDO

XXIII OLIMPÍADA BRASILEIRA DE MATEMÁTICA 2Problemas e Soluções da Primeira Fase

XXIII OLIMPÍADA BRASILEIRA DE MATEMÁTICA 11Problemas e Soluções da Segunda Fase

XXIII OLIMPÍADA BRASILEIRA DE MATEMÁTICA 21Problemas e Soluções da Terceira Fase

XXIII OLIMPÍADA BRASILEIRA DE MATEMÁTICA 41Problemas e Soluções da Primeira Fase - Nível Universitário

XXIII OLIMPÍADA BRASILEIRA DE MATEMÁTICA 46Problemas e Soluções da Segunda Fase - Nível Universitário

XXIII OLIMPÍADA BRASILEIRA DE MATEMÁTICA 57Premiados

AGENDA OLÍMPICA 61

COORDENADORES REGIONAIS 62

Page 2: Eureka 2002

Sociedade Brasileira de Matemática

EUREKA! N°13, 2002

2

XXIII OLIMPÍADA BRASILEIRA DE MATEMÁTICAProblemas e Soluções da Primeira Fase

PROBLEMAS – NÍVEL 1

1. Considere dois números naturais, cada um deles com três algarismos diferentes.O maior deles só tem algarismos pares e o menor só tem algarismos ímpares. Omenor valor possível para a diferença entre eles é:A) 111 B) 49 C) 29 D) 69 E) 5

2. Na figura abaixo, temos 4 circunferências e alguns pontos destacados no interiordessas circunferências. Escolhendo exatamente um desses pontos dentro de cadauma das circunferências, e unindo-os por segmentos de reta que não se cruzam,formamos um quadrilátero. Quantos quadriláteros diferentes seremos capazes dedesenhar nessas condições?

A) 4 B) 14 C) 60 D) 120 E) 24

3. Joana escreve a seqüência de números naturais 1, 6, 11,..., onde cada número,com exceção do primeiro, é igual ao anterior mais cinco. Joana pára quandoencontra o primeiro número de três algarismos. Esse número é:A) 100 B) 104 C) 101 D) 103 E) 102

4. Quantos números de dois algarismos não são primos nem múltiplos de 2, 3 ou 5?A) 1 B) 3 C) 2 D) 4 E) mais de 4

5. No conjunto 101, 1 001, 10 001,..., 1 000 000 000 001 cada elemento é umnúmero formado pelo algarismo 1 nas extremidades e por algarismos 0 entreeles. Alguns desses elementos são números primos e outros são compostos.Sobre a quantidade de números compostos podemos afirmar que:A) é igual 11B) é igual a 4C) é menor do que 3 D) é maior do que 4 e menor do que 11 E) é 3

Page 3: Eureka 2002

Sociedade Brasileira de Matemática

EUREKA! N°13, 2002

3

6. Uma pêra tem cerca de 90% de água e 10% de matéria sólida. Um produtorcoloca 100 quilogramas de pêra para desidratar até o ponto em que a águarepresente 60% da massa total. Quantos litros de água serão evaporados?(lembre-se: 1 litro de água tem massa de 1 quilograma).A) 15 litros B) 45 litros C) 75 litros D) 80 litrosE) 30 litros

7. O triângulo equilátero T à direita tem lado 1. Juntandotriângulos congruentes a esse, podemos formar outrostriângulos equiláteros maiores, conforme indicado no desenhoabaixo.

Qual é o lado do triângulo equilátero formado por 49 dos triângulos T?A) 7 B) 49 C) 13 D) 21E) é impossível formar um triângulo equilátero com esse número de triângulos T

8. Os números inteiros positivos de 1 a 1000 são escritos lado a lado, em ordemcrescente, formando a seqüência 123456789101112131415... 9991000. Nestaseqüência, quantas vezes aparece o grupo “89” ?A) 98 B) 32 C) 22 D) 89 E) 21

9. Um serralheiro tem 10 pedaços de 3 elos de ferro cada um, mostrados abaixo.

Ele quer fazer uma única corrente de 30 elos. Para abrir e depois soldar um elo oserralheiro leva 5 minutos. Quantos minutos no mínimo ele levará para fazer acorrente?

A) 30 B) 35 C) 40 D) 45 E) 50

10. Escrevem-se os números naturais numa faixa decorativa, da seguinte maneira:1

2 4

3 5

6 8

7

Assinale a figura correta:

Page 4: Eureka 2002

Sociedade Brasileira de Matemática

EUREKA! N°13, 2002

4

a) b )

d )

c)

e)

2 00 0 2 00 0

2 00 0

2 00 0

2 00 1 2 00 1

2 00 1

2 00 1

2 00 0

2 00 1

A) B) C)

D) E)

11. 2 melancias custam o mesmo que 9 laranjas mais 6 bananas; além disso, meiadúzia de bananas custa a metade de uma melancia. Portanto, o preço pago poruma dúzia de laranjas e uma dúzia de bananas é igual ao preço de:A) 3 melancias B) 4 melancias C) 6 melancias D) 5 melanciasE) 2 melancias

12. Qual é o último algarismo da soma de 70 números inteiros positivosconsecutivos?A) 4 B) 0 C) 7 D) 5 E) Faltam dados

13. Em Tumbólia, um quilograma de moedas de 50 centavos equivale em dinheiro adois quilogramas de moedas de 20 centavos. Sendo 8 gramas o peso de umamoeda de 20 centavos, uma moeda de 50 centavos pesará:A) 15 gramas B) 10 gramas C) 12 gramas D) 20 gramasE) 22 gramas

14. As medidas dos lados de um retângulo são números inteiros distintos. Operímetro e a área do retângulo se exprimem pelo mesmo número. Determineesse número.A) 18 B) 12 C) 24 D) 9 E) 36

15. O número N de três algarismos multiplicado por 7 deu como resultado umnúmero que termina em 171.A soma dos algarismos de N é:A) 10 B) 11 C) 12 D) 13 E) 14

16. Em um tabuleiro retangular com 6 linhas e 9 colunas, 32 casas estão ocupadas.Podemos afirmar que:

A) Todas as colunas têm pelo menos 3 casas ocupadas.B) Nenhuma coluna tem mais de 3 casas ocupadas.C) Alguma coluna não tem casas ocupadas.

Page 5: Eureka 2002

Sociedade Brasileira de Matemática

EUREKA! N°13, 2002

5

D) Alguma linha tem pelo menos 6 casas ocupadas.E) Todas as linhas têm pelo menos 4 casas ocupadas.

17. Contando-se os alunos de uma classe de 4 em 4 sobram 2, e contando-se de 5 em5 sobra 1. Sabendo-se que 15 alunos são meninas e que nesta classe o número demeninas é maior que o número de meninos, o número de meninos nesta classe é:A) 7 B) 8 C) 9 D) 10 E) 11

18. São escritos todos os números de 1 a 999 nos quais o algarismo 1 apareceexatamente 2 vezes (tais como, 11, 121, 411, etc). A soma de todos estesnúmeros é:A) 6882 B) 5994 C) 4668 D) 7224 E) 3448

19. Cinco animais A, B, C, D, e E, são cães ou são lobos. Cães sempre contam averdade e lobos sempre mentem. A diz que B é um cão. B diz que C é um lobo. Cdiz que D é um lobo. D diz que B e E são animais de espécies diferentes. E dizque A é um cão. Quantos lobos há entre os cinco animais?A) 1 B) 2 C) 3 D) 4 E) 5

20. Com azulejos quadrados brancos e pretos todos do mesmo tamanho, construímosos seguintes mosaicos.

A regra para se construir estes mosaicos é a seguinte: inicialmente formamos umquadrado com 1 azulejo branco cercado por azulejos pretos; e em seguida, outroquadrado, este com 4 azulejos brancos, também cercado por azulejos pretos; e assimsucessivamente.Com 80 azulejos pretos, quantos azulejos brancos serão necessários para se fazeruma seqüência de mosaicos como esta? A) 55 B) 65 C) 75 D) 85 E) 100

PROBLEMAS – NÍVEL 2

1. Veja o problema 4 do Nível 1.

Page 6: Eureka 2002

Sociedade Brasileira de Matemática

EUREKA! N°13, 2002

6

2. O triângulo CDE pode ser obtido pela rotação do triângulo ABC de 90o nosentido anti-horário ao redor de C, conforme mostrado no desenho abaixo.Podemos afirmar que α é igual a:

A

6 0 O

4 0 O

α

B

C

D

E A) 75o B) 65o C) 70o D) 45o E) 55o

3. Veja o problema 5 do Nível 1. 4. Veja o problema 6 do Nível 1.5. Veja o problema 8 do Nível 1. 6. Veja o problema 9 do Nível 1.7. Veja o problema 11 do Nível 1. 8. Veja o problema 12 do Nível 1.9. Veja o problema 14 do Nível 1. 10. Veja o problema 15 do Nível 1.

11. Os pontos P1, P2, P3, … estão nesta ordem sobre uma circunferência e são taisque o arco que une cada ponto ao seguinte mede 35°. O menor valor de n > 1 talque Pn coincide com P1 é:

A) 37 B) 73 C) 109 D) 141 E) 361

12. Veja o problema 16 do Nível 1.

13. ABCDE é um pentágono regular e ABF é um triângulo equilátero interior. Oângulo FCD mede:A) 38° B) 40° C) 42° D) 44° E) 46°

14. Veja o problema 19 do Nível 1.

15. Um círculo é dividido, por 2n + 1 raios, em 2n + 1 setores congruentes. Qual é onúmero máximo de regiões do círculo determinadas por estes raios e por umareta?A) 3n B) 3n + 1 C) 3n + 2 D) 3n + 3 E) 4n

16. Paulo e Cezar têm algum dinheiro. Paulo dá a Cezar R$5,00 e, em seguida,

Cezar dá a Paulo 3

1 do que possui. Assim, ambos ficam com R$18,00. A

diferença entre as quantias que cada um tinha inicialmente é:A) R$7,00 B) R$8,00 C) R$9,00 D) R$10,00 E) R$11,00

Page 7: Eureka 2002

Sociedade Brasileira de Matemática

EUREKA! N°13, 2002

7

17. Um fazendeiro tinha 24 vacas e ração para alimentá-las por 60 dias. Entretanto,10 dias depois, ele comprou mais 6 vacas e 10 dias depois dessa compra elevendeu 20 vacas. Por mais quantos dias após esta última compra ele podealimentar o gado com a ração restante?A) 50 B) 60 C) 70 D) 80 E) 90

18. Veja o problema 18 do Nível 1.

19. Uma mesa retangular, cujos pés têm rodas, deve ser empurrada por um corredorde largura constante, que forma um ângulo reto.

a

b

Se as dimensões da mesa são a e b (com 2a < b), qual deve ser a largura mínimado corredor para que a mesa possa ser empurrada através dele?

A) a + b B)2

2)( ba + C)

4

2)( ba + D)

4

2)(2 ba + E)

4

2)2( ba +

20. Somente uma das figuras a seguir representa a planificação de um cubo na qualestá destacada a sua interseção com um plano. Qual?A) B) C) D) E)

21. Quantos dígitos tem o menor quadrado perfeito cujos quatro últimos dígitos são

2001?A) 9 B) 5 C) 6 D) 7 E) 8

22. Papa-Léguas participou de uma corrida (junto com o Ligeirinho e o Flash), queconsistia em dar 100 voltas em um circuito. Como sempre, o Coiote queria pegaro Papa-Léguas e colocou um monte de alpiste no meio da pista. É claro que oCoiote não conseguiu pegar o Papa-Léguas, mas ele fez com que a velocidademédia dele na primeira volta fosse de apenas 200 km/h. Sabendo disso, avelocidade média do Papa-Léguas na corrida:A) Não ultrapassa 200 km/h. B) Não ultrapassa 250 km/h, mas pode ultrapassar 200km/h.

Page 8: Eureka 2002

Sociedade Brasileira de Matemática

EUREKA! N°13, 2002

8

C) Não ultrapassa 2000 km/h, mas pode ultrapassar 250km/h.D) Não ultrapassa 20000 km/h, mas pode ultrapassar os 2000km/h.E) Pode ultrapassar 20000 km/h.

23. Veja o problema 20 do Nível 1.24. Veja o problema 19 do Nível 1.

25. O hexágono ABCDEF é circunscritível. Se AB = 1, BC = 2, CD = 3, DE = 4 e EF= 5, quanto mede FA?

12

3

4 5

?

A

B

C

D

E

F

A) 1 B) 3 C) 15/8 D) 6 E) 9

PROBLEMAS – NÍVEL 3

1. Veja o problema 4 do Nível 1. 2. Veja o problema 2 do Nível 2.3. Veja o problema 5 do Nível 1. 4. Veja o problema 6 do Nível 1.5. Veja o problema 8 do Nível 1. 6. Veja o problema 9 do Nível 1.7. Veja o problema 15 do Nível 1. 8. Veja o problema 11 do Nível 2.9. Veja o problema 13 do Nível 2. 10. Veja o problema 15 do Nível 2.11. Veja o problema 22 do Nível 2.

12. O número de soluções inteiras distintas da equação 4)2126( 222 2

=−+− +− xxxx é:

A) 0 B) 1 C) 2 D) 3 E) 4

13. Uma rifa foi organizada entre os 30 alunos da turma do Pedro. Para tal, 30bolinhas numeradas de 1 a 30 foram colocadas em uma urna. Uma delas foi,então, retirada da urna. No entanto, a bola caiu no chão e se perdeu e umasegunda bola teve que ser sorteada entre as 29 restantes. Qual a probabilidade deque o número de Pedro tenha sido o sorteado desta segunda vez?A) 1/29 B) 1/30 C) 1/31 D) 1/60 E) 2/31

14. Cinco animais A, B, C, D, e E, são cães ou são lobos. Cães sempre contam averdade e lobos sempre mentem. A diz que B é um cão. B diz que C é um lobo. C

Page 9: Eureka 2002

Sociedade Brasileira de Matemática

EUREKA! N°13, 2002

9

diz que D é um lobo. D diz que B e E são animais de espécies diferentes. E dizque A é um cão. Quantos lobos há entre os cinco animais?A) 1 B) 2 C) 3 D) 4 E) 5

15. São escritos todos os números de 1 a 999 nos quais o algarismo 1 apareceexatamente 2 vezes (tais como, 11, 121, 411, etc). A soma de todos estesnúmeros é:A) 6882 B) 5994 C) 4668 D) 7224 E) 3448

16. Veja o problema 19 do Nível 2.17. Veja o problema 20 do Nível 2.

18. Seja f(x) = x2 – 3x + 4. Quantas soluções reais tem a equação f(f(f(...f(x)))) = 2(onde f é aplicada 2001 vezes)?A) 0 B) 1 C) 2 D) 2001 E) 22001

19. Veja o problema 21 do Nível 2.

20. Seja ABCD um trapézio retângulo cujos únicos ângulos retos são A e B . M e Nsão os pontos médios de AB e CD, respectivamente. A respeito dos

ângulos α = BNA ˆ e β = DMC ˆ , podemos dizer que:A) α < βB) α > βC) α = βD) pode ocorrer qualquer uma das situações das alternativas A), B) e C).E) o ângulo α é reto

21. A soma dos valores reais de x tais que x2 + x + 1 = 156/(x2 + x) é:A) 13 B) 6 C) –1 D) –2 E) –6

22. Para cada ponto pertencente ao interior e aos lados de um triângulo acutânguloABC, considere a soma de suas distâncias aos três lados do triângulo. O valormáximo desta soma é igualA) à média aritmética das 3 alturas do triângulo.B) ao maior lado do triângulo.C) à maior altura do triânguloD) ao triplo do raio do círculo inscrito no triângulo.E) ao diâmetro do círculo circunscrito ao triângulo.

Page 10: Eureka 2002

Sociedade Brasileira de Matemática

EUREKA! N°13, 2002

10

23. Seja f uma função de Z em Z definida como f(x) = x/10 se x é divisívelpor 10 ef(x) = x + 1 caso contrário. Se a0 = 2001 e an+1 = f(an), qual é o menor valor de npara o qual an = 1?A) 20B) 38C) 93D) 2000E) an nunca é igual a 1

24. Veja o problema 25 do Nível 2.

25. No triângulo ABC, AB = 5 e BC = 6. Qual é a área do triângulo ABC, sabendo

que o ângulo C tem a maior medida possível?

A) 15 B) 75 C) 2/77 D) 113

E) 2/115

GABARITO

NÍVEL 1 (5a. e 6a. séries)1) E 6) C 11) A 16) D2) D 7) A 12) D 17) E3) C 8) B 13) B 18) A4) B 9) B 14) A 19) D5) D 10) D 15) C 20) A

NÍVEL 2 (7a. e 8a. séries)1) B 6) B 11) B 16) B 21) B2) E 7) A 12) D 17) E 22) D3) D 8) D 13) C 18) A 23) A4) C 9) Anulada 14) E 19) D 24) D5) B 10) C 15) D 20) B 25) B

NÍVEL 3 (Ensino Médio)1) B 6) B 11) D 16) D 21) C2) E 7) C 12) D 17) B 22) C3) D 8) B 13) B 18) C 23) B4) C 9) C 14) D 19) B 24) B5) B 10) D 15) A 20) A 25) E

Page 11: Eureka 2002

Sociedade Brasileira de Matemática

EUREKA! N°13, 2002

11

XXIII OLIMPÍADA BRASILEIRA DE MATEMÁTICAProblemas e Soluções da Segunda Fase

PROBLEMAS – NÍVEL 1

PROBLEMA 1O jogo de dominó é formado por 28 peças retangulares distintas, cada uma com duaspartes, com cada parte contendo de 0 a 6 pontinhos. Por exemplo, veja três dessaspeças:

Qual é o número total de pontinhos de todas as peças?

PROBLEMA 2As peças de um jogo chamado Tangram são construídas cortando-se um quadradoem sete partes, como mostra o desenho: dois triângulos retângulos grandes, umtriângulo retângulo médio, dois triângulos retângulos pequenos, um quadrado e umparalelogramo. Se a área do quadrado grande é 1, qual é a área do paralelogramo?

PROBLEMA 3Carlinhos faz um furo numa folha de papel retangular. Dobra a folha ao meio e furao papel dobrado; em seguida, dobra e fura novamente o papel dobrado. Ele poderepetir esse procedimento quantas vezes quiser, evitando furar onde já havia furos.Ao desdobrar a folha, ele conta o número total de furos feitos. No mínimo, quantasdobras deverá fazer para obter mais de 100 furos na folha?

PROBLEMA 4Os pontos da rede quadriculada abaixo são numerados a partir do vértice inferioresquerdo seguindo o caminho poligonal sugerido no desenho. Considere o pontocorrespondente ao número 2001. Quais são os números dos pontos situadosimediatamente abaixo e imediatamente à esquerda dele?

Page 12: Eureka 2002

Sociedade Brasileira de Matemática

EUREKA! N°13, 2002

12

1 2

34

5 6 7

8

9 10

11

12

13

PROBLEMA 5Apresente todos os números inteiros positivos menores do que 1000 que têmexatamente três divisores positivos. Por exemplo: o número 4 tem exatamente trêsdivisores positivos: 1, 2 e 4.

PROBLEMA 6Seja N o número inteiro positivo dado por N = 12 + 22 + 32 + 42 +…+ (196883)2 .Qual é o algarismo das unidades de N ?

PROBLEMAS – NÍVEL 2

PROBLEMA 1: Veja o problema 2 do Nível 1.PROBLEMA 2: Veja o problema 4 do Nível 1.

PROBLEMA 3Se a n-ésima OBM é realizada em um ano que é divisível por n, dizemos que esseano é super-olímpico. Por exemplo, o ano 2001, em que está sendo realizada a 23a

OBM, é super-olímpico pois 2001 = 87 ⋅ 23 é divisível por 23. Determine todos osanos super-olímpicos, sabendo que a OBM nunca deixou de ser realizada desde suaprimeira edição, em 1979, e supondo que continuará sendo realizada todo ano.

PROBLEMA 4

As medidas dos ângulos do triângulo ABC são tais que CBA ˆ90ˆˆ <<< . As

bissetrizes externas dos ângulos A e C cortam os prolongamentos dos ladosopostos BC e AB nos pontos P e Q, respectivamente. Sabendo que

ACCQAP == , determine os ângulos de ABC.

PROBLEMA 5Dizemos que um conjunto A formado por 4 algarismos distintos e não nulos éintercambiável se podemos formar dois pares de números, cada um com 2

Page 13: Eureka 2002

Sociedade Brasileira de Matemática

EUREKA! N°13, 2002

13

algarismos de A, de modo que o produto dos números de cada par seja o mesmo eque, em cada par, todos os dígitos de A sejam utilizados.Por exemplo, o conjunto 1;2;3;6 é intercambiável pois 21 ⋅ 36 = 12 ⋅ 63.Determine todos os conjuntos intercambiáveis.

PROBLEMA 6O matemático excêntrico Jones, especialista em Teoria dos Nós, tem uma bota com 5pares de furos pelos quais o cadarço deve passar. Para não se aborrecer, ele gosta dediversificar as maneiras de passar o cadarço pelos furos, obedecendo sempre àsseguintes regras:• o cadarço deve formar um padrão simétrico em relação ao eixo vertical;• o cadarço deve passar exatamente uma vez por cada furo, sendo indiferente se

ele o faz por cima ou por baixo;• o cadarço deve começar e terminar nos dois furos superiores e deve ligar

diretamente (isto é, sem passar por outros furos) os dois furos inferiores.Representamos a seguir algumas possibilidades.

Qual é o número total de possibilidades que o matemático tem para amarrar seucadarço, obedecendo às regras acima?Observação: Maneiras como as exibidas a seguir devem ser consideradas iguais (istoé, deve ser levada em conta apenas a ordem na qual o cadarço passa pelos furos).

PROBLEMAS – NÍVEL 3

PROBLEMA 1: Veja o problema 3 do Nível 2.

PROBLEMA 2No triângulo ABC, a mediana e a altura relativas ao vértice A dividem o ângulo BÂCem três ângulos de mesma medida. Determine as medidas dos ângulos do triânguloABC.

Page 14: Eureka 2002

Sociedade Brasileira de Matemática

EUREKA! N°13, 2002

14

PROBLEMA 3Determine todas as funções f: → tais que f(x) = f(–x) e f(x + y) = f(x) + f(y) + 8xy+ 115 para todos os reais x e y.

PROBLEMA 4: Veja o problema 5 do Nível 2.

PROBLEMA 5O matemático excêntrico Jones, especialista em Teoria dos Nós, tem uma bota com npares de furos pelos quais o cadarço deve passar. Para não se aborrecer, ele gosta dediversificar as maneiras de passar o cadarço pelos furos, obedecendo sempre àsseguintes regras:• o cadarço deve formar um padrão simétrico em relação ao eixo vertical;• o cadarço deve passar exatamente uma vez por cada furo, sendo indiferente se

ele o faz por cima ou por baixo;• o cadarço deve começar e terminar nos dois furos superiores e deve ligar

diretamente (isto é, sem passar por outros furos) os dois furos inferiores.Por exemplo, para n = 4, representamos a seguir algumas possibilidades.

Determine, em função de n ≥ 2, o número total de maneiras de passar o cadarçopelos furos obedecendo às regras acima.Observação: Maneiras como as exibidas a seguir devem ser consideradas iguais.

PROBLEMA 6

Seja f(x) = 2

2

1 x

x

+. Calcule

f

1

1 + f

1

2 + f

1

3 + ... + f

1

n

+ f

2

1 + f

2

2 + f

2

3 + ... + f

2

n

Page 15: Eureka 2002

Sociedade Brasileira de Matemática

EUREKA! N°13, 2002

15

+ f

3

1 + f

3

2 + f

3

3 + ... + f

3

n

+ ...

+ f

n

1 + f

n

2 + f

n

3 + ... + f

n

n,

sendo n inteiro positivo.

SOLUÇÕES – NÍVEL 1

SOLUÇÃO DO PROBLEMA 1:Cada tipo de pontuação aparece 8 vezes dentre as 28 peças do dominó. Portanto onúmero total de pontos é: 8.(0 + 1 + 2 + 3 + 4 + 5 + 6) = 168.

SOLUÇÃO DO PROBLEMA 2:Traçando a menor diagonal do paralelogramo, observamos que metade do mesmoequivale a um triângulo retângulo pequeno, cuja área é ¼ da área do triânguloretângulo grande, que, por sua vez, é ¼ da área do quadrado. Logo a área doparalelogramo é igual a 2 × 1/16 = 1/8.

SOLUÇÃO DO PROBLEMA 3:Ao furar após a primeira dobra, Carlinhos faz 2 furos; após a segunda dobra, faz 4furos, após a terceira dobra, faz 8 furos, etc. Assim, ao desdobrar a folha, ele irácontar ++++ 8421 furos. Notando que:1 + 2 = 22 − 1 (após a primeira dobra)1 + 2 + 4 = 23 − 1 (após a segunda dobra)1 + 2 + 4 + 8 = 24 − 1 (após a terceira dobra), etc

Basta encontrar o menor k tal que 2k – 1 é maior ou igual a 100

710012 ≥⇔≥− kk

Assim, o menor k vale 7. Isso corresponde a 6 dobras.

SOLUÇÃO DO PROBLEMA 4:Os pontos correspondentes aos quadrados perfeitos pares e ímpares estão sobre oslados vertical e horizontal do quadriculado, respectivamente. Os quadrados perfeitosmais próximos de 2001 são 1936 = 442 e 2025 = 452. Como 2001 está maispróximo de 2025, o ponto correspondente está no segmento vertical descendente quetermina em 2025. Logo o ponto imediatamente abaixo dele corresponde ao número2002. Para achar o número do ponto imediatamente à esquerda, consideramos o

Page 16: Eureka 2002

Sociedade Brasileira de Matemática

EUREKA! N°13, 2002

16

quadrado perfeito ímpar anterior, que é 432 = 1849. O ponto desejado está nosegmento ascendente que começa em 1850 e situado à mesma distância que o ponto2001 está de 2025. Logo o número correspondente é: 1850 + (2025–2001) = 1850 +24 = 1874.

20251849 1850

20011874

2424

SOLUÇÃO DO PROBLEMA 5:Sabemos que todos os números inteiros maiores do que 1 admitem pelo menos umdivisor (ou fator) primo. Dessa forma,• se n tem dois divisores primos p e q então 1, p, q e pq são divisores de n; logo n

tem mais que três divisores;• se n é primo, então tem somente dois divisores: 1 e n;• se n é uma potência de um primo p, ou seja, é da forma ps, então 1, p, p2, ..., ps

são os divisores positivos de n. Para que n tenha três divisores s deverá ser iguala 2, isto é, n = p2. Assim, os inteiros menores que 1000 com três divisores são:4, 9, 25, 49, 121, 169, 289, 361, 529, 841, 961.

SOLUÇÃO DO PROBLEMA 6:Os algarismos das unidades dos quadrados dos números de 1 a 10 são,respectivamente, 1, 4, 9, 6, 5, 6, 9, 4, 1 e 0. Ora, a soma dos números formados poresses algarismos é 45. Portanto, a soma 12 + 22 + 32 + 42 + …+ 102 tem comoalgarismo das unidades o número 5. De 11 a 20, os algarismos das unidades dosnúmeros se repetem na mesma ordem; portanto, o algarismo das unidades da somade seus quadrados também é 5. Conseqüentemente, a soma dos quadrados dosnúmeros de 1 a 20 tem 0 como algarismo das unidades. Logo a soma 12 + 22 + 32 +42 + ... + n2 tem zero como algarismo das unidades se N é múltiplo de 20. Como N =12 + 22 + 32 + 42 + ... + 1968832 = 12 + 22 + 32 + 42 + … + 1968802 + 1968812 +1968822 + 1968832, concluímos que o algarismo das unidades de N é o mesmo donúmero 0 + 1 + 4 + 9 = 14, ou seja, 4.

Page 17: Eureka 2002

Sociedade Brasileira de Matemática

EUREKA! N°13, 2002

17

SOLUÇÕES – NÍVEL 2

SOLUÇÃO DO PROBLEMA 1: Veja a solução do problema 2 do Nível 1.SOLUÇÃO DO PROBLEMA 2: Veja a solução do problema 4 do Nível 1.

SOLUÇÃO DO PROBLEMA 3:Observando que no ano n é realizada a (n – 1978)-ésima OBM, temos que o ano n ésuper-olímpico se, e somente se, n – 1978 divide n. Assim, n – 1978 divide n – (n –1978) = 1978. Como os divisores positivos de 1978 são 1, 2, 23, 43, 46, 86, 989 e1978, os anos super-olímpicos são 1979, 1980, 2001, 2021, 2024, 2064, 2967 e3956.

SOLUÇÃO DO PROBLEMA 4:

Os triângulos ACQ e PAC são isósceles. No triângulo ACQ, temos: QAC ˆ ACQA ˆˆ ==A C Q = C + (180° – C )/2 = 90° + C /2

Logo A2 ˆ + (90° + C /2) = 180° (1)No triângulo PAC, temos:

C A P = (180° – A )/2

A C P = A P C = 180° – C

Logo (180° – A )/2 + 2(180° – C ) = 180° (2)

Resolvendo o sistema formado pelas equações (1) e (2), obtemos A = 12° e C = 132°;

daí, B = 180° – 12° – 132° = 36°.

2

ˆ90

C+

2

ˆ180 A−A

B

C

P

Q

C180 −

Â

Page 18: Eureka 2002

Sociedade Brasileira de Matemática

EUREKA! N°13, 2002

18

SOLUÇÃO DO PROBLEMA 5:Seja A = x ;y ;t ;z um conjunto intercambiável. Então podemos supor, sem perdade generalidade, que(10x + y)(10t + z) = (10y + x)(10z + t) ⇔ xt = yz (1) Por (1), temos que 5 e 7 não podem aparecer em A. Se o maior dos elementos de Afosse menor ou igual a 4, teríamos A = 1;2;3;4, que não é intercambiável. Logo Apossui pelo menos um dos dígitos 6, 8 ou 9.Se o maior elemento de A é 9, temos por (1) que 3 e 6 também pertencem a A. Nestecaso temos o conjunto intercambiável A = 2;3;6;9.Se o maior elemento de A é 8, temos que 4 e outro algarismo par estão em A. Assim,temos A = 1;2;4;8 ou A = 3;4;6;8.Se o maior elemento de A é 6, temos que 3 e outro algarismo par estão em A. Destaforma, A = 1;2;3;6 ou A = 2;3;4;6.Assim, temos no total 5 conjuntos intercambiáveis: 2;3;6;9, 1;2;4;8, 3;4;6;8,1;2;3;6 e 2;3;4;6.Obs. O enunciado não deixaria claro que as outras possibilidades, por exemplo:

)10()10()10()10( tyzxztyx +⋅+=+⋅+ , não deveriam ser consideradas. A análisedessas possibilidades torna o problema bem mais complicado, porém não acrescentanovos conjuntos intercambiáveis aos listados acima.

SOLUÇÃO DO PROBLEMA 6:Como o padrão deve ser simétrico, basta decidir os primeiros 5 furos pelos quais ocadarço deve passar. A partir daí, os furos ficam determinados pela simetria. Porexemplo, o 7° furo deve ser o outro furo da mesma linha visitada no 4° furo. Note,ainda, que a simetria implica em que as linhas visitadas nos 5 primeiros furos sãotodas distintas. Além disso, a primeira destas linhas é obrigatoriamente a de cima e a5ª é obrigatoriamente a de baixo, já que os furos da linha de baixo são visitadosconsecutivamente.Assim, para obter um padrão para o cadarço, podemos iniciar pelo furo da esquerdada linha superior e devemos decidir:• em que ordem as 3 linhas intermediárias são visitadas• de que lado queremos passar nestas 3 linhas e na linha de baixo.Para escolher a ordem das 3 linhas, observamos que a primeira pode ser escolhida de3 modos; a seguir, a segunda pode ser escolhida de 2 modos, ficando a terceiradeterminada. Logo há 6 possibilidades de escolha para a ordem das linhas.Para escolher o lado por onde passar nas 4 linhas, temos duas opções para cada umadelas, para um total de2 × 2 × 2 × 2 = 16 possibilidades. Logo o número total de modos de amarrar ocadarço é 6 × 16 = 96.

Page 19: Eureka 2002

Sociedade Brasileira de Matemática

EUREKA! N°13, 2002

19

Outra solução:Começando do lado esquerdo da linha superior, o segundo furo pode ser escolhidode 6 modos (qualquer um das linhas intermediárias); o terceiro de 4 modos (nas duasintermediárias restantes) e o quarto e quinto de 2 modos cada (suas linhas estãodeterminadas, bastando escolher o lado). Logo há um total de 6 × 4 × 2 × 2 = 96possibilidades.

SOLUÇÕES – NÍVEL 3

SOLUÇÃO DO PROBLEMA 1: Veja a solução do problema 3 do nível 2.SOLUÇÃO DO PROBLEMA 2:

α

αα

A

B CH M

Seja M o ponto médio de BC e H o pé da altura relativa a A. Temos que AH é comum

aos triângulos AHM e AHB, BHA ˆ ≅ MHA ˆ (retos) e HÂM ≅ HÂB, logo, pelo casoALA, os triângulos AHM e AHB são congruentes. Assim, BH = HM = MC/2, poisMC = MB. Como AM é bissetriz de HÂC, pelo teorema das bissetrizes AH/AC =HM/MC ⇔ AH/AC = 1/2 ⇔ cos2α = 1/2. Como 0 < 2α

o, 2α = 60o ⇔ α = 30o.

Portanto os ângulos do triângulo ABC são m(BÂC) = 3α = 90o, m( CBA ˆ ) = 90o – α

60o e m( BCA ˆ ) = 90o – 2α o.

SOLUÇÃO DO PROBLEMA 3:Fazendo y = –x, temos f(x + (–x)) = f(x) + f(–x) + 8x(–x) + 115 ⇔ f(0) = 2f(x) – 8x2 +115 ⇔ f(x) = 4x2 + (f(0) – 115)/2. Fazendo x = 0 nesta última igualdade, temos f(0) =4 ⋅ 02 + (f(0) – 115)/2 ⇔ f(0) = –115. Logo f(x) = 4x2 + (f(0) – 115)/2 ⇔ f(x) = 4x2 –115 e verificamos de fato que esta função satisfaz as condições do enunciado: f(–x) =4(–x)2 – 115 = 4x2 – 115 = f(x) e f(x) + f(y) + 8xy + 115 = 4x2 – 115 + 4y2 – 115 +8xy + 115 = 4(x + y)2 – 115 = f(x + y). Assim, f(x) = 4x2 – 115 é a única função quesatisfaz todas as condições do enunciado.

SOLUÇÃO DO PROBLEMA 4: Veja a solução do problema 5 do Nível 2.

SOLUÇÃO DO PROBLEMA 5:Numere os furos superiores com o número 1, os furos imediatamente abaixo com onúmero 2 e assim por diante, até os furos inferiores, que recebem o número n.

Page 20: Eureka 2002

Sociedade Brasileira de Matemática

EUREKA! N°13, 2002

20

Observe que basta estabelecermos os primeiros n furos onde o cadarço irá passar (opadrão é simétrico). Uma maneira pode ser definida por uma seqüência indicando osnúmeros dos primeiros n furos onde o laço passa (observe que tal seqüência temtodos os números de 1 a n, começa com 1 e termina com n) e por uma outraseqüência de comprimento n – 1 cujo k-ésimo termo indica se o cadarço muda delado ao passarmos do k-ésimo para o (k + 1)-ésimo termo da primeira seqüência. Porexemplo, (1, 3, 2, 4) e (muda, não muda, muda) representa

1 12 23 34 4

Assim, como há (n – 2)! seqüências com os números de 1 a n começando com 1 eterminando com n e 2n – 1 seqüências indicando se o cadarço muda de lado ou não, há(n – 2)! ⋅ 2n – 1 maneiras.

SOLUÇÃO DO PROBLEMA 6:

Seja S a soma pedida. Como f(x) + f(1/x) = 2

2

1 x

x

+ +

2

2

)/1(1

)/1(

x

x

+ = 1, podemos

escrever

2

2

parcelas) de pares (

3

1

1

3

1

2

2

1

1

1

1

12

321

22

3

2

2

2

1

11

3

1

2

1

1

321

22

3

2

2

2

1

11

3

1

2

1

12

2

2

2

nS

nS

nn

nf

n

nf

ffffffS

n

nf

nf

nf

nf

nffff

nffff

n

nf

nf

nf

nf

nffff

nffffS

=⇔

=⇔

+

++

+

+

+

+

+

=⇔

++

+

+

+

+

++

+

+

+

++

+

+

+

++

+

+

+

+

++

+

+

+

++

+

+

=

Page 21: Eureka 2002

Sociedade Brasileira de Matemática

EUREKA! N°13, 2002

21

XXIII OLIMPÍADA BRASILEIRA DE MATEMÁTICAProblemas e Soluções da Terceira Fase

PROBLEMAS – NÍVEL 1

PROBLEMA 1:Numa famosa joalheria estão armazenadas várias pedras preciosas dos seguintestipos: esmeraldas; safiras e rubis. Todas as pedras do mesmo tipo têm o mesmovalor. Além disso, 24 esmeraldas valem tanto quanto 12 rubis e também valem tantoquanto 8 safiras.Com R$350.000,00 um príncipe comprou um conjunto com 4 esmeraldas, 6 rubis e 4safiras. Quanto custa cada tipo de pedra?

PROBLEMA 2:Um cubinho foi colocado no canto de uma sala, conforme a Figura 1.Empilharam-se outros cubinhos iguais ao primeiro, de forma a cobrir as facesvisíveis do mesmo, usando-se o menor número possível de peças. Como se pode verna Figura 2, após a colocação dos novos cubinhos, restam 9 faces visíveis dessescubinhos.

Figura 1 Figura 2a) Quantos cubinhos iguais a esses, no mínimo, seria necessário empilhar, de forma

a cobrir aquelas 9 faces visíveis?b) Continua-se a fazer essa pilha, repetindo-se o procedimento descrito. Quando a

pilha tiver um total de 56 cubinhos, quantas faces poderão ser vistas?

PROBLEMA 3:No triângulo ABC tem-se que M é o ponto médio do lado AB (isto é, os segmentosAM e MB têm o mesmo comprimento). N é o ponto médio de MC e R é o pontomédio de NA.O triângulo ABC tem área 2000. Determine a área do triângulo AMR.

A M B

C

N

R

Page 22: Eureka 2002

Sociedade Brasileira de Matemática

EUREKA! N°13, 2002

22

PROBLEMA 4:Dizemos que um número natural é legal quando for soma de dois naturaisconsecutivos e também for soma de três naturais consecutivos.a) Mostre que 2001 é legal, mas 1999 e 2002 não são legais.

b) Mostre que 20012001 é legal.

PROBLEMA 5:As 42 crianças de uma escola infantil deram as mãos formando uma fila e cada umadelas recebeu um número da seguinte maneira: a primeira delas ficou com o número1, a segunda ficou com o número 2 e, assim sucessivamente, até a última, que ficoucom o número 42. Continuando de mãos dadas, foram para um pátio, onde cada umadelas ficou sobre uma lajota quadrada; duas crianças com números consecutivosficaram em lajotas vizinhas com um lado comum (ou seja, do lado esquerdo, do ladodireito, na frente ou atrás, mas nunca em diagonal).

Ao relatar esse fato para a diretora, a inspetora Maria fez o desenho à esquerda,mostrando a posição de três crianças sobre o retângulo formado pelas 42 lajotas,sobre as quais estavam as crianças. Num outro comunicado, a inspetora Célia fezoutro desenho, mostrado à direita, com a posição das mesmas crianças sobre omesmo retângulo. Ao receber os dois desenhos a diretora disse a uma das inspetoras:"O seu desenho está errado".

i) Com qual das duas inspetoras a diretora falou? Qual foi o raciocínio dadiretora?

ii) Complete o desenho correto satisfazendo as condições do enunciado.

(Desenho de Maria) (Desenho de Célia)

11 20

31

11 20

31

Page 23: Eureka 2002

Sociedade Brasileira de Matemática

EUREKA! N°13, 2002

23

PROBLEMAS – NÍVEL 2

PROBLEMA 1:Uma folha de papel retangular ABCD, de área 1, é dobrada em sua diagonal AC e,em seguida, desdobrada; depois é dobrada de forma que o vértice A coincida com ovértice C e, em seguida, desdobrada, deixando o vinco MN, conforme desenhoabaixo.

A BM

CND

a) Mostre que o quadrilátero AMCN é um losango.b) Se a diagonal AC é o dobro da largura AD, qual é a área do losango AMCN?

PROBLEMA 2: Veja o problema 5 do Nível 2.

PROBLEMA 3:Dado um inteiro positivo h demonstre que existe um número finito de triângulos delados inteiros a, b, c e altura relativa ao lado c igual a h .

h

c

a b

.

PROBLEMA 4:Mostre que não existem dois números inteiros a e b tais que (a + b) (a2 + b2) = 2001.

PROBLEMA 5:Sejam a, b e c números reais não nulos tais que a + b + c = 0.

Calcule os possíveis valores de .)(

)()(2555

4442333

cba

cbacba

++++++

PROBLEMA 6:Em um quadrilátero convexo, a altura em relação a um lado é definida como aperpendicular a esse lado passando pelo ponto médio do lado oposto. Prove que asquatro alturas têm um ponto comum se e somente se o quadrilátero é inscritível, istoé, se e somente se existe uma circunferência que contém seus quatro vértices.

Page 24: Eureka 2002

Sociedade Brasileira de Matemática

EUREKA! N°13, 2002

24

PROBLEMAS – NÍVEL 3

PROBLEMA 1:

Prove que )(2))(( cbaabccaba ++≥++ para quaisquer números reais positivos

a, b e c.

PROBLEMA 2:Dado um inteiro 10 >a definimos uma seqüência 0)( ≥nna da seguinte forma; para

cada 0≥k , 1+ka é o menor inteiro kk aa >+1 tal que mdc .1)...,( 101 =⋅⋅⋅+ kk aaaa

Diga para quais valores de 0a temos que todos os termos ak da seqüência sãoprimos ou potências de primos.

PROBLEMA 3:E e F são pontos do lado AB, do triângulo ABC, tais que AE = EF = FB. D é pontoda reta BC tal que BC é perpendicular a ED. AD é perpendicular a CF. Os ângulosBDF e CFA medem x e 3x, respectivamente. Calcule a razão (DB) / (DC).

PROBLEMA 4:Uma calculadora tem o número 1 na tela. Devemos efetuar 2001 operações, cadauma das quais consistindo em pressionar a tecla sen ou a tecla cos. Essas operaçõescalculam respectivamente o seno e o cosseno com argumentos em radianos. Qual é omaior resultado possível depois das 2001 operações?

PROBLEMA 5: Veja o problema 6 do Nível 2.

PROBLEMA 6:Temos uma fileira longa de copos e n pedras no copo central (copo 0). Os seguintesmovimentos são permitidos:

Movimento tipo A

i – 1 i i + 1 i + 2 i – 1 i i + 1 i + 2 ⇒

Se há pelo menos uma pedra no copo i e pelo menos uma no copo i + 1 podemosfazer uma pedra que está no copo i + 1 pular para o copo i – 1 eliminando uma pedrado copo i.

Movimento tipo B.

Page 25: Eureka 2002

Sociedade Brasileira de Matemática

EUREKA! N°13, 2002

25

i – 1 i i + 1 i + 2 i – 1 i i + 1 i + 2 ⇒

Se há pelo menos duas pedras no copo i podemos pular uma para o copo i + 2 e umaoutra para o copo i – 1.Demonstre o seguinte fato: fazendo os movimentos tipo A ou B durante um temposuficientemente longo sempre chegaremos a uma configuração a partir da qual não émais possível fazer nenhum desses dois tipos de movimento. Além disso essaconfiguração final não depende da escolha de movimentos durante o processo.

SOLUÇÕES – NÍVEL 1

PROBLEMA 1: SOLUÇÃO DE RAPHAEL RODRIGUES MATA (SALVADOR – BA)Se 24 esmeraldas equivalem a 12 rubis, significa que 1 rubi equivale a duasesmeraldas, e se 24 esmeraldas equivalem a 8 safiras, uma safira equivale a 3esmeraldas. Assim, se o príncipe comprar 6 rubis, é o mesmo que ele comprar 12esmeraldas, e se ele comprar 4 safiras, é o mesmo que ele comprar 12 esmeraldas.Assim, o conjunto comprado pelo príncipe tem o mesmo valor de 28 esmeraldas (4 +12 + 12 = 28).

Para se descobrir o valor de cada esmeralda, basta efetuar .1250028

350000 =

Sabemos que o rubi vale o dobro da esmeralda, assim, temos 12500 × 2 = 25000.Por fim, sendo a safira o triplo do valor da esmeralda, temos 12500 × 3 = 37500.Finalmente, descobrimos que a esmeralda custa R$12500,00; o rubi custaR$25000,00; e cada safira tem o valor de R$37500,00.

PROBLEMA 2: SOLUÇÃO DE EDUARDO FISCHER (ENCANTADO - RS)a) São 6 cubos; chego a esta solução apenas olhando. Na fileira de baixo se

acrescentam 3 cubos, na do meio 2 e na de cima 1 (a de cima antes estava vazia).Note que primeiro foi botado um cubo, depois 3, que é um mais dois, agora bota6, que é 1 + 2 + 3. Depois acrescentarei 10 (1 + 2 + 3 + 4) e depois 15 (1 + 2 + 3+ 4 + 5). Isso se deve ao fato que, ao quadricular o chão, na figura 1 seacrescenta um cubo ao nada. Depois se acrescenta 2 cubos (no chão) e 1 emcima. Após termos 4 cubos, se acrescenta 3 (no chão), 2 para cobrir os 2 queantes estão no chão (os mais distantes da parede) e 1 para cobrir lá em cima. Eassim segue.

b) Para chegar a 56, vou somando: 1, 3 = 1 + 2, 6 = 1 + 2 + 3, 10 = 1 + 2 + 3 + 4,15 = 1 + 2 + 3 + 4 + 5 e 21 = 1 + 2 + 3 + 4 + 5 + 6. A parede terá a altura de 6cubos, quando isso acontecer. Vamos listar as faces e cubos à mostra:

Page 26: Eureka 2002

Sociedade Brasileira de Matemática

EUREKA! N°13, 2002

26

No andar de cima há 1 cubo e 3 faces.No segundo andar há 6 faces e 2 cubos.No terceiro andar há 9 faces e 3 cubos.No quarto andar há 12 faces e 4 cubos.No quinto andar há 15 faces e 5 cubos.No andar de baixo há 18 faces e 6 cubos.Para cada cubo à mostra, há 3 faces vistas. São 21 cubos à mostra, 63 faces no total.

PROBLEMA 3: SOLUÇÃO DE FELIPE GONÇALVES ASSIS (CAMPINA GRANDE – PB)O triângulo ABC tem área 2000. Ao ser cortado pelo segmento de reta MC, divide-seem outros 2 triângulos menores, de mesma área, ACM e CMB. Pode-se perceber queeles têm áreas iguais pois:• A base de ambos tem o mesmo comprimento, pois AM = MB, já que M é o pontomédio de AB.• A altura dos dois também é a mesma.• Duas medidas que determinam a área de um triângulo são, justamente, base ealtura. Assim descobrimos uma propriedade dos triângulos:

Se um triângulo for cortado por um segmento de reta que parte do ponto médio deum dos segmentos que o compõem até o vértice formado pelas outras duas retas,obter-se-ão 2 novos triângulos, de mesma área, correspondente a metade da área doprimeiro triângulo.Isto é o que ocorre com o triângulo ABC que forma os triângulos ACM e CMB, cadaum com área 1000 (= 2000:2).Ocorre isto também com ACM, cortado pelo segmento AN, ele forma AMN e ACN,ambos com área 500 (= 1000:2).Acontece o mesmo com AMN que é cortado por RM, originando AMR e RMN, cadaqual com área 250(= 500:2).Resposta: A área de AMR é de 250.

PROBLEMA 4: SOLUÇÃO DE PAULO ANDRÉ CARVALHO DE MELO (RIO DE JANEIRO – RJ)Para um número ser a soma de 2 naturais consecutivos ele tem que ser:x + x + 1 = 2x + 1.Ou seja, ímpar.Para um número ser a soma de 3 naturais consecutivos ele tem que ser:x + x + 1 + x + 2 = 3x + 3.Ou seja, múltiplo de 3.Portanto um número legal é aquele que é múltiplo de 3 e ímpara) O 2001 é múltiplo de 3 e é ímpar, mas o 2002 e o 1999 não são.

Page 27: Eureka 2002

Sociedade Brasileira de Matemática

EUREKA! N°13, 2002

27

b) Já que 2001 é múltiplo de 3, 20012001 terá 32001 como um de seus fatores primos eserá ímpar pois um número ímpar multiplicado por outro número ímpar é igual anúmero ímpar.Portanto 20012001 é legal, já que respeita as condições para um número ser legal.

PROBLEMA 5: Veja a solução do Problema 2 do Nível 2.

SOLUÇÕES – NÍVEL 2

PROBLEMA 1: a) SOLUÇÃO DE ELTON GOMES CORIOLANO (FORTALEZA – CE)Se fizermos a segunda dobradura, teremos a seguinte figura:

A C

B

M

N D

Logo percebemos que CNAN = e que CMAM = . Temos, então, dois triângulos

isósceles: o triângulo ANC e o triângulo AMC. Então os ângulos NAC ˆ e NCA ˆ são

congruentes e os ângulos MAC ˆ e MCA ˆ são congruentes também. Sabendo que AM é

paralelo a NC, pode-se dizer que os ângulos MAC ˆ e NCA ˆ são congruentes, pois estes

são ângulos alternos internos. Assim, NAC ˆ = NCA ˆ = MAC ˆ = MCA ˆ . Portanto, os

ângulos NAM ˆ e NCM ˆ são congruentes. Logo, NCA ˆ é congruente a CMA ˆ . Assim,AMCN é paralelogramo, pois seus ângulos opostos são congruentes. Por este motivo,os lados opostos também serão iguais, ou seja, AM = CN e AN = CM. Dessa forma,AM = CN = AN = CM. Logo, o quadrilátero AMCN é um paralelogramo com todosos lados iguais, ou seja, AMCN é um losango.

A M B

D N C

T

b) SOLUÇÃO DE THIAGO COSTA LEITE SANTOS (SÃO PAULO – SP)

Seja T o centro do retângulo. Como AT é metade de AC , ADAT = , os triângulosATN e ADN são retângulos, logo ADNATN ∆≡∆ , pelo caso especial cateto-

Page 28: Eureka 2002

Sociedade Brasileira de Matemática

EUREKA! N°13, 2002

28

hipotenusa (a hipotenusa comum aos dois triângulos). Analogamente, temosCTMCBM ∆≡∆ e portanto o retângulo ABCD está dividido em 6 triângulos

congruentes. Portanto a área de cada triângulo é igual a 6

1

6

)(=

ABCD e como o

losango AMCN possui 4 dos 6 triângulos, sua área será igual a 3

2

6

14 =⋅ .

PROBLEMA 2: SOLUÇÃO DE RAFAEL DAIGO HIRAMA (CAMPINAS – SP)i) Vamos pintar o retângulo igual a um tabuleiro de damas. Pelo enunciado o númeroanterior (e o posterior) de um número está acima, abaixo, a esquerda ou a direita.Portanto, um número tem sua cor diferente de seus dois vizinhos.

Então, se o primeiro não é pintado, o segundo é, oterceiro não é, o quarto é, etc.Ou seja, os números pares tem uma cor e osimpares outra.

Pintando os dois tabuleiros percebemos que no de Célia o 20 e o 31 estão da mesmacor. Logo o desenho de Célia está errado.

11

31

20

Desenho da Maria

11

31

20

ii) Para completar o desenho, vejamos que o 10 e o 12 devem estar no quadrado àesquerda e no abaixo do 11 pois este não tem mais vizinhos. Igualmente o 19 e o 21estão à direita ou abaixo do 20.O número 12 deve estar abaixo do 11 pois, se não o 9 teria que ficar no lugarmarcado com um círculo, e não haveria lugar para o 8 (pois o único modo deconectar o 12 com o 20 seria como na figura).

2012 11

13 10 19

14 9 18

15 16 17

Logo o 12 está abaixo do 11.

Vamos provar que o 19 está abaixo de 20. Se o 19 estivesse a direita de 20 travaria asaída para o 21 formando uma barreira sem saída. Logo o 19 está abaixo do 20.Com isso, podemos montar algo obrigatório (tudo por falta de outras opções).

Page 29: Eureka 2002

Sociedade Brasileira de Matemática

EUREKA! N°13, 2002

29

2010 11

9 12 19

8 13

7

21

31

1) O 9 tem que estar abaixo do 10.2) O 13 abaixo do 12.3) O 8 abaixo do 9.4) O 7 abaixo do 8.

Temos que o 21 deve alcançar o 31 em 9 espaços, o 13 alcançar o 19 em 5 espaços etemos mais 6 espaços para fazer do 6 ao 1.

Se colocarmos o 6 à direita do 7 teríamos problemas, pois o caminho entre o 21 e o31 (o 30 não pode ficar embaixo pois senão isolaria o 31 de uma parte em brancopois o cordão 29 a 31 impediria e nenhuma parte do cordão pois senão também fariauma área sem alcance).

2010 11

9 12 19

8 13

7

21

31

6

30

22 23

24

25

26

27 29 28

Daria pouco espaço para 11 espaços (14 → 18 e 1→ 6) (O 30 deve ficar na direita do 31 neste casopois abaixo e à esquerda ocorreria o espaço semalcance), 8 no máximo.

Então o 6 é abaixo do 7, o 5 abaixo do 6, o 4 à direita do 5, o 3 à direita do 4 (estes 3por falta de opção) e o 2 à direita do 3 (pois à esquerda do 31 tem o 6 e abaixo tem o4 sobrando os outros dois (acima e à direita para o 30 e o 32).

Logo, teremos:

2010 11

9 12 19

8 13

7

21

31 6 30

22 23

24

25

26

27 29 28

22 23

28

5 4 3 2

29 26

27 24

25

2010 11

9 12 19

8 13

7

21

31

30

32

22 23

26

33

28

18 17

14 15 16

29 27

6 34

5 4 3 2 1

24

25

35

Agora, o cordão 21 → 31 deve ter o 30 acima do 31pois senão forma a mesma área sem alcance como osexemplos. Então o 32 fica à direita do 31 e o 29 àdireita do 30, o 14 à direita do 13, o 28 à direita do29, o 33 à direita do 32, o 1 à direita do 2, o 34 àdireita do 33, o 15 à direita do 14, o 18 à direita do19, o 22, 17, 16, 27 à direita do 21, 18, 15 e 28respectivamente. O 23, 26 e 35 devem ficar à direitado 22, 27 e 34, respectivamente. Como o 23 devechegar ao 26 em 2 espaços, o 24 fica embaixo do 23e o 25, embaixo do 24. Então é só completar com o35 a 42 do único modo possível.

Page 30: Eureka 2002

Sociedade Brasileira de Matemática

EUREKA! N°13, 2002

30

Respostas: i) A diretora falou com Célia. A direitora percebeu que o 20 e o 31 nãopoderiam estar na mesma cor se o tabuleiro fosse pintado como o de damas.ii)

2010 11

9 12 19

8 13

7

21

31

30

32

22 23

26

33

28

18 17

14 15 16

29 27

6 34

5 4 3 2 1

24

25

35

36

41

40

38

37

39

42

Obs. Estendendo-se esse raciocínio é possível demonstrar que esta é a única maneirade se completar o desenho.

PROBLEMA 3: SOLUÇÃO DE THIAGO COSTA LEITE SANTOS (SÃO PAULO – SP)

a b

c

h

m n α.

Temos 222

222

nhb

mha

+=

+=

Para uma equação Pitagórica:x2 = y2 + z2, resolvida em inteiros positivos, temos:

cima) de a (análoga 12

))1( senão (pois 122

2222

+≥

+<<+≥

zy

yxyyz

122

1

2

1

12

2

1

122

22

2

22

2

2

2

+≥≥

−⇔≥

+≥

⇔≥−

+≥⇔ yz

y

zy

yz

zy

yz

Mas se substituirmos y por h e z por m ou n, teremos que estes estarão limitados acertos valores, logo a, b, e c estão limitados a certos valores e, portanto acabou!!!Mas supomos que m e n são inteiros positivos e eles poderiam ser irracionais e asoma de dois irracionais dar um inteiro.Para isto podemos aplicar a lei dos cossenos.

Page 31: Eureka 2002

Sociedade Brasileira de Matemática

EUREKA! N°13, 2002

31

)(;cos 22 hbnb

n −==α

22222222 2cos2 hbccbabccba −⋅−+=⇔−+= α . Assim, se n é irracional,a2 também será, absurdo. Portanto m e n são inteiros.

Obs.: m e n não poderiam ser fracionários, observando as equações iniciais, quemostram que m2 e n2 são inteiros. Note que se n = p/q é um racional não inteiro,então n2 = p2/q2 também não será inteiro, pois se q não divide p então q2 não dividep2.

PROBLEMA 4: SOLUÇÃO DE HENRY WEI CHENG HSU (SÃO PAULO – SP)2001 = 3. 23 . 29Temos que a2 + b2 é sempre ≥ 0, então, para o produto (a + b) (a2 + b2) valer 2001,a + b não pode ser negativo.Como a e b são inteiros, a2 ≥ a e b2 ≥ b, assim a2 +b2 ≥ a + b (a2 < a somente quando0 < a < 1)Os valores possíveis para (a + b) e (a2 + b2) são:1) a + b = 1 e a2 + b2 = 2001 2) a + b = 3 e a2 + b2 = 6673) a + b = 23 e a2 + b2 = 874) a + b = 29 e a2 + b2 = 69Vamos analisar os casos 2) e 3).Em ambos temos a + b ≡ 3(mod 4) e a2 + b2 ≡ 3 (mod 4)Para a + b ≡ 3 (mod 4), podemos ter os seguintes casos:a ≡ 0 e b ≡ 3 ⇒ a2 ≡ 0 e b2 ≡ 1a ≡ 1 e b ≡ 2 ⇒ a2 ≡ 1 e b2 ≡ 0a ≡ 2 e b ≡ 1 ⇒ a2 ≡ 0 e b2 ≡ 1a ≡ 3 e b ≡ 0 ⇒ a2 ≡ 1 e b2 ≡ 0Então, quando a + b ≡ 3 (mod 4), a2 + b2 será congruente a 1 (mod 4). Portanto oscasos 2) e 3) não podem existir. Vamos analisar o caso 1)Para a = 1 e b = 0 e a = 0 e b = 1, a2 + b2 sempre valerá 1.Quando um dos dois for negativo: a = n + 1 e b = – n ou vice-versaa + b = n + 1 – n = 1a2 + b2 = n2 + 2n + 1 + n2 = 2n2 + 2n + 1a2 + b2 = 2001 ⇔ 2n2 + 2n + 1 = 2001 ⇔ 2n2 + 2n – 2000 = 0 ⇔ n2 + n – 1000 = 0n2 + n – 1000 = 0∆ = 1 + 4000 = 4001 como 4001 não é quadrado perfeito, n não é inteiro.Outra maneira: a + b = 1 ⇔ a = 1 – ba2 + b2 = b2 – 2b + 1 + b2 = 2b2 – 2b + 1

Page 32: Eureka 2002

Sociedade Brasileira de Matemática

EUREKA! N°13, 2002

32

2b2 – 2b + 1 = 2001 ⇔ 2b2 – 2b – 2000 = 0 ⇔ b2 – b – 1000 = 0∆ = 1 + 4000 = 4001 (não é quadrado perfeito).Assim, o caso 1) não existe. Vamos analisar o caso 4).a + b = 29 ⇔ b = 29 – aa2 + b2 = a2 + 841 – 58a + a2 = 2a2 – 58a + 841a2 + b2 = 69 ⇔ 2a2 – 58a + 772 = 0 ⇔ a2 – 29a + 386 = 0∆ = 841 – 1544 < 0Como ∆ < 0, a não é inteiro. Assim, o caso 4) não pode ocorrer.Como nenhum caso pode ocorrer, não existem dois números a e b tais que(a + b) (a2 + b2) = 2001.

PROBLEMA 5: SOLUÇÃO DE MARCELA SOBRINHO PEREIRA (FORTALEZA – CE)Pelo enunciado, temos: a + b + c = 0a = – (b + c) ⇒ a3 = – b3 – c3 –3b2c –3bc2 = – b3 – c3 –3bc(b + c),a4 = b4 + c4 + 4b3c + 4c3b + 6c2b2 e a5 = – b5 – c5 –5b4c –5c4b – 10b3c2 –10c3b2.Substituindo na expressão, temos:( ) ( )

( )( ) =+++

+++++−22233

2233442

225

64422)(3

bccbcbbc

bcbccbcbcbbc

=+++

+++++2223322

223344222

)22(25

)64422()(9

bccbcbcb

bcbccbcbcbcb

( ) 25

18

481048

)481048(2

25

95243354266

5243354266

=++++++++++++⋅

bccbcbcbcbcb

bccbcbcbcbcb.

PROBLEMA 6: Veja a solução do problema 5 do nível 3.

SOLUÇÕES – NÍVEL 3

PROBLEMA 1: SOLUÇÃO DE CARLOS STEIN NAVES DE BRITO (GOIÂNIA - GO)Seja a + b = x, a + c = y e b + c = z. (x, y, z > 0)Temos um sistema linear de variáveis a, b e c e

Resolvendo

−+=

−+=→−=+−

−+=→=+

=+−=+−

=+⇔

=+=+=+

2

2

2

xyzc

yzxbxycb

zyxaxba

zcb

xycb

xba

zcb

yca

xba

Page 33: Eureka 2002

Sociedade Brasileira de Matemática

EUREKA! N°13, 2002

33

Chamando x + y + z = 2p. Temos a = p – z, b = p – y e c = p – x.Logo a desigualdade vira:

(a + b)(a + c) ≥ ⇔++ )( cbaabc

⇔−+−+−−−−≥

−+−

−+− ))()()((2 zpypxpxpypzpxpzpypzp

zyx

pxpypzpyx ))()((2 −−−≥⋅ . Basta provar isso.

Temos que:

>⇔>

⇔+>+++>+

0

02

)()(

a

a

cbcaba

zyx

>⇔>

⇔+>+++>+

0

02

)()()(

;

c

c

bacbca

xzy

e

>⇔>

⇔+>+++>+

0

02

)()()(

b

b

cacbba

yzx .

Assim x, y e z podem ser lados de um triângulo, sendo p o semiperímetro.Assim seja α o ângulo desse triângulo entre x e y.

Logo a área A dele é: 2

αysenxA

⋅= ; como senα ≤ 1, temos Aysenxyx =⋅≥⋅⋅22

1 α

Mas A em função dos lados é ,))()(( pxpypzp −−− logo

⇔−−−=≥ pxpypzpAxy

))()((2

s ,))()((2 pxpypzpxy −−−≥ cqd.

PROBLEMA 2: SOLUÇÃO DE DAVI MÁXIMO ALEXANDRINO NOGUEIRA (FORTALEZA - CE)Notação: mdc (x, y) = (x, y).Já que a0 faz parte de (an)n ≥ 0, o próprio deve ser potência de primo.Suponha a0 = pm (p primo). Considere primeiro p = 2: a0 = 2m.a1 é o menor inteiro que satisfaz a1 > a0 e (a1, a0) = 1. Portanto, temosa1 = a0 + 1 = 2m + 1. Se m for ímpar, a1 = 2m + 1 ≡ 0 (mod 3)⇒ 3|a1 ⇒ a1 = 3n, pois todo termo de (an)n ≥ 0 deve ser potência de primo.Logo 2m + 1 = 3n. Se m = 1, 3 = 3n ⇒ n = 1, a1 = 3 e a0 = 2 (I)Se m ≥ 2, segue que 4|2m, e 2m +1 ≡ 3n (mod 4) ⇒ 1 ≡ (–1)n (mod 4) ⇒ n par,digamos n = 2n0. Logo, ficamos com:

Page 34: Eureka 2002

Sociedade Brasileira de Matemática

EUREKA! N°13, 2002

34

1)12(22222213

213)13)(13(2312

0

0

0002 =⇒−=⇒−=⇒

=+

=−⇒+−=⇒=+ − ααβααβ

β

α

n

nnnmnm

e 321 =⇒=+= mαβ e n = 2 80 =⇒ a 91 =a (II)

Agora observemos:(I) 20 =a 31 =a

Então, 52 =a , 73 =a , .114 =a Provaremos por indução:

ai = (i + 1)-ésimo primoSupondo a hipótese válida para j, olhemos o passo indutivo j → j + 1:

1)...,(; 1011 => ++ jjjj aaaaaa . Seja p o (j + 2)-ésimo primo.

Por hipótese, ai = (i + 1)-ésimo primo. Seja jjj axpaax >⇒−++∈ 1,...,2,1

porém, 1)...,( 10 ≠jaaax pois os fatores primos de x pertencem ao produto jaaa ...10

logo, pa j =+1 pois p é o menor inteiro tal que p > aj e (p, a0…aj) = 1 e o resultado

segue.(II): a0 = 8 a1 = 9 ⇒ a2 = 11, a3 = 13, a4 = 17, a5 = 19, a6 = 23, a7 = 25 = 52, a8 = 29,a9 = 31, a10 = 37, a11 = 41, a12 = 43, a13 = 47, a14 = 49 = 72, a15 = 53.Prova analogamente por indução ("mesmo" passo indutivo anterior) que a partir dei ≥ 15, ai = (i + 1)-ésimo primo (indução feita no anexo).Sendo assim, suponha a0 = 2m com m par (m > 0). Temos:a0 = 2m ⇒ a1 = 2m + 1 ⇒ a2 = 2m + 3 (pois, (a2, a1) = (a2, a0) = 1) ⇒ a3 = 2m + 5.Porém, como m é par, a3 ≡ 2m + 5 ≡ (–1)m + 5 ≡ 0 (mod 3) ⇒ 2m + 5 = 3n

Resolvamos então 2m + 5 = 3n.

Como 2m = a0 > 1 ⇒ m > 0. Mas como estamos supondo m par ⇒ m ≥ 2 ⇒ .2|4 m

Logo,2m + 5 = 3n ⇒ 2m + 5 ≡ 3n (mod 4) ⇒ 3n ≡ 1 (mod 4) ⇒ n par, digamos n = 2n0.Logo,

)2323 (pois 523

123

5)23)(23(523352

0000

00

00

00000000 2222

mnmn

mn

mn

mnmnmnnm

+<−

=+

=−⇒

⇒=+−⇒=−⇒=+

42142 0010 =⇒==⇒=⇒=⇒ + anmmm

,51 =⇒ a ,72 =a 93 =a

Prova-se novamente por indução (mesmo passo indutivo da anterior) que para todo,4≥i ai = (i + 1)-ésimo primo.

Page 35: Eureka 2002

Sociedade Brasileira de Matemática

EUREKA! N°13, 2002

35

Agora, suponha .2,0 ≠= ppa m nmpa 211 =+=⇒ pois .1|2 +mp Se n for par,digamos n = 2n0, veja (mod 3):

3)3(mod11)3(mod21 02 =⇒≡+⇒≡+ ppp mnm

sendo assim; resolvemos02213 nm =+

=−

=+⇒+−=⇒

β

α

312

312)12)(12(3

0

0

00

n

nnnm

11,0)13(32233 =⇒==⇒−=⇒=−⇒ − mαββαββα

e ⇒==⇒= 4,32 10 aan 52 =a e para ⇒≥ 2i )1( += iai -ésimo primo (pelomesmo argumento indutivo).

Sendo assim, suponha n ímpar: nmm papa 21, 10 =+== e 1222 +=+= nmpa (já

que (pm, pm + 2) = (pm + 2, pm + 1) = 1) ⇒ 2|3 a (já que n é ímpar) ⇒ ta 32 = logo,

123 =− nt (*)mas já é sabido nosso que as soluções de (*) se dão para: t = 1 e n = 1 ⇒ a2 = 3 e a1 = 2 ⇒ a0 = 1 (Absurdo!)t = 2 e n = 3 ⇒ a2 = 9 e a1 = 8 ⇒ a0 = 7 (III)(III): a0 = 7, a1 = 8, a2 = 9, a3 = 11, a4 = 13, a5 = 17, a6 = 19, a7 = 23, a8 = 25 = 52,a9 = 29 e, para i ≥ 9, ai = (i + 1)-ésimo primo (como antes).Resposta: a0 pode ser 2, 3, 4, 7 ou 8.

Anexo:Para um j > 1, suponha a0, a1, …, aj definidos como disse e a hipótese valendo.Basta ver que no conjunto a0, a1,…,aj aparecem todos os j + 1 primeiros fatoresprimos. Chame p = (j + 2)-ésimo primo. Os candidatos a aj + 1 antes de p seriamaj + 1, aj + 2,…, p – 1. Porém, os fatores primos de qualquer um desses carasaparecem no produto a0 a1,…, aj ⇒ se x ∈ aj + 1,…, p – 1,mdc (x, a0 a1,…, aj) ≠ 1. Logo, x = p (veja que p >aj e mdc (p, a0 a1,… ,aj) = 1).Os outros passos de indução são totalmente análogos.

Page 36: Eureka 2002

Sociedade Brasileira de Matemática

EUREKA! N°13, 2002

36

PROBLEMA 3: SOLUÇÃO DE LARISSA CAVALCANTE QUEIROZ DE LIMA (FORTALEZA - CE) E F B A

D

G

M

C

x

m

m

(m + n)

n

n

2x x

x

x

x

ED ⊥ BC ⇒ ∆ EDB é um triângulo retângulo.Como EF = FB, F é ponto médio de EB ⇒ EF = FB = DF

⇒ Se xBDF =ˆ , temos AFCxxxxFBDBDFEFDxDBF ˆ32ˆˆˆˆ =<=+=+=⇒=portanto D não está dentro do segmento BC ⇒ H, pé da altura relativa ao lado BCestá fora do segmento BC (pois AH // ED e H, D, B estão na reta nessa ordem ⇒ H

∉ DB e como D ∉ CB , temos H ∉ CB ) ⇒ BCA ˆ é um ângulo obtuso.

* CFDCDFDFCxxxAFDAFCDFC ∆⇒=⇒=−=−= ˆˆ23ˆˆˆ é isósceles com CD =CF = m.Seja M o ponto médio de DB ⇒ MF é base média de ∆BDE ⇒ MF // ED ⇒

°== 90ˆˆ BDEBMF ( °= 90ˆFGD pois AD ⊥ CF e G = AD ∩ CF)

⇒ DGMF é um quadrilátero inscritível ⇒ xMGFMDF == ˆˆ e xGMDGFD == ˆˆ

⇒ CMGxGMCMGC ∆⇒== ˆˆ é isósceles com CM = CG = nDM = MB ⇒ MB = DC + CM = m + n e DB = DM + MB = 2(m + n)GF = GC + CF = m + n

Menelaus ∆ FCB e reta ⇒ADG

mnnm

n

m

nm

GF

GC

DC

DB

AB

AF341

)(2

3

21 =⇒=

+⋅+⋅⇒=⋅⋅

2

72

7

2

34

2

32

22)(2 =⋅

=

+

=+

=+=+=m

m

m

mm

m

mm

m

nm

m

nm

DC

DB

.2

7=∴DC

DB

Obs. C não obtuso ⇒ H ∈ CB e portanto

D ∈ o.contradiçã .0ˆ

023ˆˆˆ

<−=⇒=+=+=⇒

DFC

xyxyxDFCCFADFACB

Page 37: Eureka 2002

Sociedade Brasileira de Matemática

EUREKA! N°13, 2002

37

PROBLEMA 4: SOLUÇÃO DE ALEX CORRÊA ABREU (NITERÓI - RJ)Definimos as seqüências (xk), (yk) como sendo respectivamente o máximo e omínimo depois da operação feita k vezes kk senxx =⇒ +1 ou kycos pois a função

seno é crescente e a co-seno decrescente no intervalo considerado e kk senyy =+1 ou

,cos kx mas ⇒>+⇔

−>⇔>

22cos

ππyxysensenxysenx

se kkkk senxxyx =>+ +1 temos, 2

π e kk xy cos1 =+ . Analogamente, se

kkkk yxyx cos temos,2 1 =<+ +π

e .1,1221 ≥∀=+⇒=+ kyxsenyy kkkk mas

temosAssim, . e cos2

22)(2)2()(

11

22222

kkkk

kkkkkkkkkk

senyyyx

yxyxyxyxyx

==

⇒<≤+⇒=+≤++=+

++

π

1cos...cos...coscoscos 1999

1199920002001 vezes

sensensensenysensensenyyx ==== já que

,1 1cos sen< pois .2

14

ππ <<

PROBLEMA 5: SOLUÇÃO DE EINSTEIN DO NASCIMENTO JUNIOR (FORTALEZA - CE)

Lema 1: Sabemos que um quadrilátero é inscritível se e somente se as mediatrizesdos lados desse quadrilátero são concorrentes.Lema 2: Os pontos médios dos lados de um quadrilátero qualquer, convexo ou não,formam um paralelogramo.

Tome agora um quadrilátero convexo ABCD, com lados opostos não paralelos.

C B N

M P

Q D A

O1

O'1

Sejam M, N, P, Q os pontos médios de AB,BC, CD, DA.Trace agora as mediatrizes de BC e AD, quese encontram em O1.

Chame .EMPNQ =∩Provaremos que as alturas relativas a BC ea AD se encontram no ponto simétrico a O1

em relação a E.Chame O'1 o simétrico de O1 em relação aE.

Page 38: Eureka 2002

Sociedade Brasileira de Matemática

EUREKA! N°13, 2002

38

Pelo Lema 2, E é ponto médio de NQ e E é ponto médio de MP e além disso pordefinição de O'1, E é ponto médio de O'1O1.Então temos que NO1QO'1 é um paralelogramo!Daí: NO'1 // O1Q ⇒ NO'1 ⊥ AD ⇒ NO'1 é a altura em relação a AD.QO'1 // NO1 ⇒ QO'1 ⊥ BC ⇒ QO'1 é a altura em relação a BC.Logo O'1 é o encontro das alturas relativas a BC e AD.Fazendo o mesmo para os lados AB e CD podemos concluir que:O simétrico em relação a E do encontro de mediatrizes de lados opostos é igual àinterseção das alturas destes lados opostos.Chame O2 o encontro das mediatrizes de AB e CD.O'2 será o simétrico em relação a E de O2 e consequentemente o encontro das alturasrelativas a AB e CD.Note que: O'1 ≡ O'2 ⇔ O1 ≡ O2.Então: O'1 ≡ O'2 ⇔ O1 ≡ O2 ⇔ ABCD é inscritível.Segue que O'1 ≡ O'2 ⇔ ABCD é inscritível.Logo as 4 alturas tem um ponto em comum se e somente se o quadrado forinscritível.

PROBLEMA 6: SOLUÇÃO DE HUMBERTO SILVA NAVES (SÃO PAULO - SP)

Vamos introduzir o conceito de energia para as pedras:

=

x

xpos

E)(

4

3, onde pos(x) é a posição de x.

O que acontece se realizarmos um movimento? Vamos mostrar que a energia semprediminui a cada momento:Movimento tipo A:

p – 1 p p + 1 p – 1 p p + 1 ⇒

EEEEpppp

<

⋅=

+

−=

−+

12

5

4

3

4

3

4

3

4

3'

11

Movimento tipo B:

p – 1 p p + 1 p + 2 p – 1 p p + 1 ⇒

p + 2

Page 39: Eureka 2002

Sociedade Brasileira de Matemática

EUREKA! N°13, 2002

39

EEEEpppp

<

−=

+

+

−=

+−

48

5

4

3

4

3

4

3

4

32'

21

Obs. O número 4

3 não foi escolhido ao acaso, foi escolhido um número q tal que:

21 qq +< e qq 21 3 <+Considere um copo de posição p, onde p é tal que

04

3E

p

>

, onde E0 é a energia inicial do sistema.

Como a energia, a cada movimento, sempre diminui, qualquer que sejam osmovimentos que se faça, nenhuma pedra ficará numa posição menor ou igual a p. Ouseja, existe uma "barreira" à esquerda para as pedras.Estamos agora capazes de resolver o problema (a primeira parte):* Dada uma configuração inicial das "n" pedras, é impossível realizar uma seqüênciainfinita de movimentos.Demonstração: Vamos demonstrar (*) por indução:Base: Para n = 1 é verdadeiro!Passo indutivo: Suponha, por absurdo, que seja possível realizar uma seqüênciainfinita de movimentos. Sabemos que a posição da pedra mais à esquerda nãoaumenta a cada movimento e como existe uma barreira à esquerda, então a partir deum certo ponto a pedra mais a esquerda não mais será movimentada, e só com osrestantes (o número de pedras restantes é no máximo n – 1) é impossível realizaruma seqüência infinita de movimentos, o que é um absurdo!Logo por indução, (*) é verdadeiro para todo n ∈ *.Vamos resolver a segunda parte do problema:Sabemos que dada uma configuração inicial, independente das escolhas dosmovimentos sempre chegamos a uma configuração onde é impossível mover(configuração parada). Suponha por absurdo que a partir de uma configuração inicialse chegue a duas configurações paradas distintas A e B.Seja k' a posição da pedra mais à direita das configurações A e B e k = k' + 2.Considere o seguinte invariante: (não varia a cada movimento)

∑ −=x

xposkFI )( , onde Fn é o n-ésimo número de Fibonacci.

Lembramos que F1 = 1, F2 = 1 e Fn + 2 = Fn + 1 + Fn, para todo n ≥ 1)Sabemos que ,BA II = pois I é invariante, isto é, permanece o mesmo depois de cadamovimento.De fato, Fk – (p – 1) = Fk – p + 1 = Fk – p + Fk – p – 1 = Fk – p + Fk – (p + 1), donde I não mudaapós um movimento do tipo A, e =++=+ −−−−−+−−− 21)2()1( pkpkpkpkpk FFFFF

Page 40: Eureka 2002

Sociedade Brasileira de Matemática

EUREKA! N°13, 2002

40

= pkF −2 , donde I não muda após um movimento do tipo B.

Algoritmo:1- Seja x a pedra mais à esquerda de A e y a pedra mais à esquerda de B.Devemos ter pos(x) = pos(y), pois se fosse pos(x) ≠ pos(y) (assumimos sem perda degeneralidade que pos(x) > pos(y)), teríamos:

∑∈

−−−−−− ++++≤=At

xposkxposkxposktposkA FFFFFI 24)(2)()()( ...

se )(xposk − for par e

∑∈

−−−− +++≤=At

xposkxposktposkA FFFFI 32)()()( ... caso contrário:

Mas 121222 1... ++ <−=++ kkk FFFF e 2222123 1... +++ <−=++ kkk FFFF (como seprova facilmente por indução).logo ,1 )(1)(1)( ByposkxposkxposkA IFFFI ≤≤<−≤ −+−+− um absurdo!

2- Seja A: = A – x e B: = y.3- Vá para o 1.Pronto! Demonstramos que A e B são a mesma configuração, o que é um absurdo! Aconfiguração final independe da escolha dos movimentos.

Page 41: Eureka 2002

Sociedade Brasileira de Matemática

EUREKA! N°13, 2002

41

XXIII OLIMPÍADA BRASILEIRA DE MATEMÁTICAProblemas e Soluções da Primeira Fase – Nível Universitário

PROBLEMA 1

Seja xexf −=)( sen x. Calcule f (2001)(0). (Denotamos por f (n)(x) a derivada de ordemn no ponto x; assim, f (2)(x) = f '' (x).)

PROBLEMA 2Seja s(n) a soma dos algarismos de n. Assim, por exemplo, s(77) = 14 e s(2001) = 3.Diga se existe um inteiro positivo n com s(n) = 10 e s(n2) = 100. Se não existir,demonstre este fato. Se existir, dê um exemplo.

PROBLEMA 3O centro de massa de uma lata cilíndrica de refrigerante tem a mesma posiçãoquando a lata está vazia ou cheia. Se a massa da lata vazia é m e a massa dorefrigerante dentro da lata cheia é M, determine a fração de refrigerante que deve serdeixado na lata para que seu centro de massa fique o mais baixo possível.

PROBLEMA 4Um ratinho ocupa inicialmente a gaiola A e é treinado para mudar da gaiolaatravessando um túnel sempre que soa um alarme. Cada vez que soa o alarme oratinho escolhe qualquer um dos túneis incidentes a sua gaiola com igualprobabilidade e sem ser afetado por escolhas anteriores. Qual a probabilidade de queapós o alarme soar 23 vezes o ratinho ocupe a gaiola B?

A B C

D E F

PROBLEMA 5Seja A uma matriz n × n com a1, j = ai, 1 = 1 (para quaisquer i e j, 1 ≤ i, j ≤ n) e

1,,1,1,1 ++++ ++= jijijiji aaaa (para quaisquer i e j, 1 ≤ i, j < n). Assim,

=

63 25 7 1

25 13 5 1

7 5 3 1

1 1 1 1

A. Calcule det(A).

PROBLEMA 6

Seja xn uma seqüência de números reais definida por ,2

21

nnn

xxx −=+ .0≥n

Para quais valores de x0 a seqüência converge? Para que valor?

Page 42: Eureka 2002

Sociedade Brasileira de Matemática

EUREKA! N°13, 2002

42

SOLUÇÕES – NÍVEL UNIVERSITÁRIO

SOLUÇÃO DO PROBLEMA 1

−−=−−= −−

42)cos()('

πxsenexsenxexf xx

Em geral, se u = x + a, onde a é uma constante, a derivada de xe− sen u é igual

a .4

2

−− − πusene x

Logo .2

244

)2()('' 2

−=

−−−= −− πππ

xsenexsenexf xx

Se ,4

)2()()(

−−= − πk

xsenexf xkk para k ∈ , teremos

−−−⋅−= −+

442)2()1( ππk

xsenef xkk .4

)1()2( 1

+−−= −+ πk

xsene xk

Logo, por indução,

−−= −

4)2()()( πn

xsenexf xnn (n ∈ ) e

−−= −

4

2001)2()( 2001)2001( π

xsenexf x

−−= −

42 2 1000 π

xsene x

). (cos2 1000 xsenxe x −= −

SOLUÇÃO DO PROBLEMA 2Existem muitos inteiros n com as propriedades pedidas. A menor solução é1101111211. Algumas outras são 10111111111, 11011111111, 200220000202 e

∑=

=9

0

2 .10j

j

n

A única condição necessária é que, ao calcular n2 pelo algoritmo usual não deveocorrer nenhum 'vai um'. Mais precisamente, se a expansão decimal de n é

0...)(),10( axaxppn kk ++== com 100 <≤ ia então temos .10)()1( == nsp

Temos 02

22 ...))(()( bxbxpxq k

k ++== com2

0110 )10(,... nqaaaaaab jjjj =+++= − e .100)1( =q

Para que )1()( 2 qns = precisamos apenas que cada bj seja menor do que 10.Exemplos de soluções podem ser facilmente obtidos tomando os algarismos de npequenos e espalhados.

Page 43: Eureka 2002

Sociedade Brasileira de Matemática

EUREKA! N°13, 2002

43

SOLUÇÃO DO PROBLEMA 3Se há fração λ de refrigerante na lata, [ ]1,0∈λ , a massa total de refrigerante será

Mλ e o centro de massa do refrigerante (sem contar a lata) tem altura h2

λ, onde h é

a altura total da lata. Como o centro de massa da lata vazia tem altura 2

h, a altura do

centro de massa é ( )./22

)( mMh

mh

Mf +

⋅+⋅= λλλλ

Temos

))(2)(()(2

)(' 22

MmMMmMmM

hf +−⋅+

+= λλλ

λλ

).2()(2

222

mMMmMmM

h −++

= λλλ

As raízes de mMMmM −+ λλ 222 são 02

322

1 <+−−=M

mMmMMmλ e

.12

2

2

322

2 <++

=−+=++−=Mmmm

m

M

mMmm

M

mMmMMmλ Assim,

0)(' <λf para 20 λλ <≤ e 0)(' >λf para ,12 ≤< λλ e portanto )(λf é mínimo

para .2

2 M

mMmm −+== λλ

SOLUÇÃO DO PROBLEMA 4Seja an a probabilidade de que após n apitos o ratinho esteja na coluna central (B ouE). Temos a0 = 0 (o ratinho não começa na coluna central). Claramente após o apitosoar um número par de vezes o ratinho estará em A, C ou E e após um número ímparde vezes em B, D ou F. Assim, queremos calcular a23.Se, antes de soar o alarme, o ratinho está na coluna central ele tem 1/3 deprobabilidade de permanecer lá (independentemente da gaiola onde o ratinho estavaser B ou E). Por outro lado, se ele não está na coluna central ele tem probabilidade1/2 de ir para lá (novamente independentemente da gaiola onde o ratinho começou).Assim,

nnnn aaaa6

1

2

1)1(

2

1

3

11 −=−+=+ ou ).

7

3(

6

1

7

31 −−=−+ nn aa

Page 44: Eureka 2002

Sociedade Brasileira de Matemática

EUREKA! N°13, 2002

44

A seqüência 7

3−= nn ab é portanto uma progressão geométrica de razão 6

1− com

.7

30 −=b Assim

232367

3

⋅=b e .

67

)16(3

67

3

7

323

23

2323 ⋅+=

⋅+=a

SOLUÇÃO DO PROBLEMA 5

Fazendo operações em linhas (subtraindo a primeira linha da segunda, a segunda daterceira e assim por diante) temos

=

38 12 2 0

18 8 2 0

6 4 2 0

1 1 1 1

det)det(A

Fazendo agora operações em colunas (subtraindo a primeira coluna da segunda, asegunda da terceira e assim por diante) temos

.

13 5 1

5 3 1

1 1 1

det2

26 10 2 0

10 6 2 0

2 2 2 0

0 0 0 1

det)det( 1

=

= −

nA

Esta última matriz é a versão (n – 1) × (n – 1) de A. De fato, sua entrada (i, j) é

.))()((2

1,,1,,11,1 jijijijiji aaaaa =−−− ++++

Chamando o valor de det(A) para matrizes n × n de bn temos portanto

,11 =b nn

n bb 21 =+ donde .2 2

)1( −

=nn

nb

SOLUÇÃO DO PROBLEMA 6

Seja .2

)( 2 xxxf −= Temos 0)( =⇔= xxxf ou .

2

3=x Para 2

3>x temos

.)( xxf > Assim, se 2

30 >x temos

2

30 >≥ xxn para todo n e, por indução,

nnn xxfx >=+ )(1 para todo n. Assim, nesse caso, (xn) é crescente, e portanto

Lxnn =∞→lim para algum ∈L ou .lim +∞=∞→ nn x No primeiro caso, teríamos

Page 45: Eureka 2002

Sociedade Brasileira de Matemática

EUREKA! N°13, 2002

45

),()(lim)(limlim 1 LfxfxfxL nnnnnn ==== ∞→∞→+∞→ mas ,2

3lim 0 >≥= xxL n

donde LLf >)( , absurdo. Assim, se .lim,2

30 +∞=> ∞→ nn xx

Se ,2

3)(,1 010 >=−< xfxx donde também temos .lim +∞=∞→ nn x

Se ,2

3

2

1 <<− x com ,0≠x temos ( ) .xxf < Por outro lado, para todo

.2

1

16

1

16

1)

4

1()(,1 2

11 −>−≥−−==≥ −− nnn xxfxn Assim, se

nnn xxfxx ≤=<<− + )(,2

31 10 para todo .1≥n Portanto, existe

.2

3lim <= ∞→ nn xc Temos portanto

.)(,)()(limlim0 1 cfcfxfxc nnnn −∈==≤ ∞→+∞→ Temos que ,)( ccf =− com

c > 0, implica 2

1=c , e, como 16

1−≥nx para todo ,1≥n se 2

1lim =∞→ nn x teríamos

,2

1lim =∞→ nn x donde ,0

2

1)(limlim

2

11 =

=== ∞→+∞→ fxfx nnnn absurdo. Como

ccf <)( se 0 < c < ,2

3 temos necessariamente c = 0, o portanto .0lim =∞→ nn x

Se 10 −=x ou 2

30 =x temos

2

3=nx para todo ,1≥n donde .2

3lim =∞→ nn x

Assim, xn converge se ,2

31 0 <<− x quando 0lim =∞→ nn x e se ,

2

3,10

−∈x

quando .2

3lim =∞→ nn x Em qualquer outro caso, .lim +∞=∞→ nn x

Page 46: Eureka 2002

Sociedade Brasileira de Matemática

EUREKA! N°13, 2002

46

XXIII OLIMPÍADA BRASILEIRA DE MATEMÁTICAProblemas e Soluções da Segunda Fase – Nível Universitário

PROBLEMA 1:São dados um ponto O e uma reta r no plano. Para cada ponto P de r, seja rp a retaperpendicular a OP passando por P. Prove que o conjunto rPrp ∈ é o conjunto de

todas as retas tangentes a uma parábola.

PROBLEMA 2:Sejaε um número real positivo arbitrário. Com centro em todos os pontos do planocom coordenadas inteiras, traça-se um círculo de raioε . Prove que toda retapassando pela origem intercepta uma infinidade desses círculos.

PROBLEMA 3:Definimos SL(2, ) como o conjunto das matrizes 2 × 2 com coeficientes inteiros e

determinante 1. Seja A ∈ SL(2, ) uma matriz tal que existe n > 0 inteiro com An = I.

Prove que existe X ∈ SL(2, ) tal que X–1AX é igual a uma das matrizes:

. 1 1

1 0 ;

0 1

1 0 ;

1 1

1 0 ;

1 0

0 1

−−

±

−±

−±

±

PROBLEMA 4:Caminhando sobre os segmentos unitários da figura abaixo, determine quantospercursos distintos existem de A até B sem passar duas vezes por um mesmo ponto.

A B 1

PROBLEMA 5:

Para todo real u, seja ∫ +−=π

0

2 .) cos 21( ln)( dxuxuuI

a) Prove que ).( 2

1)()( 2uIuIuI =−= b) Calcule I(u) para todo u ∈ .

PROBLEMA 6:Seja D o conjunto dos pontos p em 2 com .1≤p Seja f : D → D uma função

sobrejetora satisfazendo qpqfpf −≤− )()( para quaisquer p, q ∈ D. Prove que f

é uma isometria, isto é, que qpqfpf −=− )()( para quaisquer p, q ∈ D.

(Observação: ),( yx denota 22 yx + .)

Page 47: Eureka 2002

Sociedade Brasileira de Matemática

EUREKA! N°13, 2002

47

SOLUÇÕES – NÍVEL UNIVERSITÁRIO

PROBLEMA 1: SOLUÇÃO DE EMANUEL DE SOUZA CARNEIRO (FORTALEZA - CE)Suponha, sem perda de generalidade, que o ponto O seja (0,1) e a reta r seja o eixo x.Seja P um ponto sobre r, P(p, 0), logo:(x, y) ∈ rp ⇔ (x – p, y) ⋅ (p, – 1) = 0 ⇔ xp – p2 – y = 0 (Equação da reta rp).Buscamos agora uma parábola do tipo y = ax2 + bx + c, de modo que estas retas rp

sejam tangentes a ela.Derivando, obtemos a equação da reta tangente a essa parábola no ponto (x0, y0).

baxxx

yy+=

−−

00

0 2

ou seja: )2)(()( 00020 baxxxcbxaxy +−=++−

0)2( 200 =−+−+⇔ ycaxbaxx

tome a = 1/4, b = c = 0, daí teremos: .04

1

2

1 200 =−−

yxxx

Esta será uma idéia: em rp | P ∈ r basta tomar p = x0 /2.

Reciprocamente, cada rp ∈ rp | P ∈ r é tangente à parábola 2

4

1xy = no ponto

(2p, p2), pois a equação da reta tangente a essa parábola nesse ponto é:

ppfpx

py ==−−

)2('2

2

⇔ 02 =−− ypxp , que é a equação da reta rp.

Logo o conjuntorp | P ∈ r é o conjunto das retas tangentes à parábola .4

2xy =

PROBLEMA 2: SOLUÇÃO DE EMANUEL DE SOUZA CARNEIRO (FORTALEZA - CE)Para provar a questão, mostraremos uma lema equivalente ao lema de Kronecker.

Lema: Seja δ um número irracional. Dado ε > 0 arbitrário, existe a ∈ * tal queεδ <a , onde x= x – [x] indica a parte fracionária de x.

Prova: Seja n ∈ tal que εε )1(1 +≤< nn . Divida o intervalo [0, 1) em (n + 1)

intervalos do tipo ),)1(,[ εε +ii ;0 ni ≤≤ o último intervalo é [nε, 1).

0 ε 2ε …

nε 1 (n + 1) ε

Observe agora os números δ, 2δ, 3δ,…;(n + 2)δ . Como a função parte fracionáriavai dos reais em [0,1), pelo princípio da casa dos pombos, dois dentre os (n + 2)números δ , 2δ , 3δ ,…,(n +2) δ , cairão num mesmo intervalo [iε, (i + 1)ε).

Page 48: Eureka 2002

Sociedade Brasileira de Matemática

EUREKA! N°13, 2002

48

Sejam j, os dois números tais que:jδ , δ ∈ [i ε, (i + 1) ε).Suponha, sem perda de generalidade, que jδ ≥ δAssim:

[ ][ ] )(][][)(

inteiro

δδδδδδδδδδδ

−+−=−⇒

+=+=

jjjjjj

Logo εεεδδδ =−+<−=− iijj )1()( e −j é o inteiro procurado

Obs: Não podemos ter kδ = 0, k ∈ *, pois isso implicaria kδ = n ∈ ⇒ δ = n/k

∈ , absurdo!Vamos agora resolver o problema.Seja dado ε > 0. Uma reta passando pela origem tem a forma y = mx

Primeiro caso: Se m ∈ , nesse caso a reta passa por infinitos "Lattice points" (pontosde coordenadas inteiras), pois se m = p/q os pontos da forma (x, y) = (kq, kq . p/q) =(kq, kp), k = 1, 2, 3… estão todos na reta, então ela interceptará todos os círculos quetêm centros nesses pontos.

Segundo caso: Se m for irracional.Os pontos da reta são da forma (x, mx), logo, pela observação feita após o lema, estareta não contém nenhum ponto de coordenadas inteiras.

Suponha que ela intersecte somente uma quantidade finita de círculos, digamos C1,C2,…, Cn de centros P1, P2, …, Pn

Observe que para qualquer outro ponto no "Lattice" P (fora os centros) teremos:d(P, r) > ε.Seja d a menor das distâncias, d1(P1, r), d2(P2, r),…, dn(Pn , r)onde di(Pi, r) é a distância do centro Pi à reta r.(Note que di (Pi, r) > 0 ⇒ d > 0).Logo teríamos dentro todos os pontos de coordenadas inteiras no plano, um deles Pi

que minimizaria a distância à reta r, com d(Pi, r) = d > 0.Isso contradiz o lema, vejamos:Pelo lema existe um inteiro a, de modo que am < d/2.Seja [ ]amb = , o ponto Q = (a, am) ∈ r e sua distância a (a, b) é menor ou igual quea distância de (a, b) a r.

Page 49: Eureka 2002

Sociedade Brasileira de Matemática

EUREKA! N°13, 2002

49

x

y

r

(a, am)

(a, b)

d '

Logo d' ≤ d((a, b), (a; am)) = |am – b| =am < d/2.Absurdo, pois d era a distância mínima ⇒a reta corta uma infinidade de círculos.

PROBLEMA 3: SOLUÇÃO DE CARLOS YUZO SHINE (SÃO PAULO – SP)

Seja

=

dc

baA uma matriz de SL(2,) tal que An = I para algum inteiro positivo n.

Seja m o menor inteiro positivo tal que Am = I. Sejam λ1 e λ2 os autovalores de A, ouseja, as raízes da equação det(A – xI) = 0 ⇔ x2 – (a + d)x + 1 = 0. (I)Se λ1 = λ2, temos que o discriminante da equação (I) é nulo, logo a + d = 2 oua + d = –2.Se a + d = 2, temos

IAdc

ba

bcddac

dabbcaA

dc

baA −=

−=

++++

=⇒

= 2

122

212

)(

)(2

22

Por indução, pode-se mostrar que Ak = kA – (k – 1)I.Desta forma, Am = I ⇔ mA – (m – 1)I = I ⇔ A = I.Se a + d = –2, temos, analogamente, que Ak = (–1)k + 1(kA + (k – 1)I) e portanto Am = I⇔ (–1)m + 1(mA + (m – 1)I) = I ⇔ A = –I e m é par (na verdade, m = 2).Se λ1 ≠ λ2, então a matriz A é diagonalizável em , ou seja, existe uma matriz P tal

que A = PDP–1, sendo

=

2

1

0

0

λλ

D . Neste caso, temos que Ak = PDkP–1, com

=

k

kkD

2

1

0

0

λλ

.

Assim, Am = I ⇔ PDmP–1 = I ⇔ Dm = I ⇔ λ1m = λ2

m = 1, ou seja, λ1 e λ2 são raízesm-ésimas da unidade. Assim, como a + d = λ1 + λ2, –1 ≤ a + d ≤ 1. Como a + d éinteiro, temos os seguintes casos:

(i) a + d = –1. Neste caso, (I) ⇔ x2 + x + 1 = 0 e portanto λ1 e λ2 são raízescúbicas da unidade. Portanto m = 3.

(ii) a + d = 0. Neste caso, (I) ⇔ x2 + 1 = 0 e portanto λ1 e λ2 são raízes quartasda unidade. Portanto m = 4.

Page 50: Eureka 2002

Sociedade Brasileira de Matemática

EUREKA! N°13, 2002

50

(iii) a + d = 1. Neste caso, (I) ⇔ x2 – x + 1 = 0 e portanto λ1 e λ2 são raízes sextasda unidade. Portanto m = 6.

Observando as matrizes dadas, temos

III =

−=

−=

−− 643

11

10;

01

10;

11

10

Assim, basta provarmos que existem inteiros x, y, z e w com xw – yz = 1 e

±=

xz

ywW

wz

yxA ,

sendo W uma das três matrizes acima. Note que

Iwz

yx

wz

yx

xz

yw⋅

=

−det

Caso (i) Tomamos

−−

=11

10W . Assim

−−−++−−−++±=

xwxzywzwwz

xyyxyzxzywA

22

22

.

Veja que a + d = – 1 é equivalente a xw – yz = 1. A condição det A = 1 é equivalentea ad – bc = 1 ⇔ a(–1 – a) – bc = 1 ⇔ bc = –(a2 + a + 1). Assim, devemos ter

)(

)(

)(

22

22

zwwzc

xyyxb

yzxzywa

++±=−−−±=

++±= (II)

Isso pode ser verificado fatorando a2 + a + 1 = (a – ω)(a – ω ) em inteiros deEisenstein. Considere um fator x – yω de a – ω e seja z – wω o fator de a – ω tal que

(x – yω)(z – wω) = a – ω. Note que (x – yω )(z – wω ) = a – ω . As normas dosfatores são x2 + y2 + xy e z2 + w2 + zw e portanto sempre existem x, y, z e w tais que|b| = x2 + y2 + xy e |c| = z2 + w2 + zw. Substituindo o valor de a de (II) na equaçãooriginal verifica-se que de fato bc = – (a2 + a + 1). Reciprocamente, a outra raizdesta última equação é –1 – a = d = –xz – yw – xw. Mas para resolver isso bastatrocar x e y por z e w, respectivamente, e trocar o sinal.

Obs. Usamos implicitamente a existência e unicidade da fatoração para inteiros deEisenstein para obter x, y, z, w como acima.

Page 51: Eureka 2002

Sociedade Brasileira de Matemática

EUREKA! N°13, 2002

51

Caso (ii) Agora tomamos

−=

01

10W , com

−−+−−+±=

xzywwz

yxxzywA

22

22

.

Novamente, é óbvio que a condição a + d = 0 é satisfeita. A condição det A = 1 éequivalente a ad – bc = 1 ⇔ bc = –(a2 + 1). O sistema correspondente agora é

)(

)(

)(

22

22

wzc

yxb

xzywa

+±=−−±=+±=

(III)

Agora usamos inteiros de Gauss! Considere a fatoração a2 + 1 = (a + i)(a – i) e sejamx + yi e z + wi fatores de a + i com (x + yi)(z + wi) = a + i. Note que (x – yi)(z – wi) =a – i. Então existem sempre x, y, z e w tais que |b| = x2 + y2 e |c| = z2 + w2.Novamente, substituindo o valor de a de (III) na equação original vemos que de fatobc = – (a2 + 1). A outra raiz é oposta ao valor de a de (III), mas é só trocar o sinal.

Obs. Aqui usamos implicitamente a existência e unicidade de fatoração para inteirosde Gauss para obter x, y, z, w como acima.

Caso (iii) Tomamos dessa vez

−=

11

10W . Temos nesse caso

+−−−++−−−+±=

xwxzywzwwz

xyyxyzxzywA

22

22

. Mas esse caso é análogo ao caso (i)!

Tome y = –y’ e w = –w’ e obtemos a mesma matriz do caso (i).

Logo existe sempre uma matriz X tal que A = XWX–1, onde W é uma das matrizesdadas no enunciado.

PROBLEMA 4: SOLUÇÃO DE JULIANA ABRANTES FREIRE (RIO DE JANEIRO – RJ)Vamos numerar os pontos assim:

0 = A B = 10

1 3 5 7 9

2 4 6 8 B (6)

A (6) C (6)

F(n) = O número de percursos que chegam em n, sem passar por n + 2.A(n) = O número de percursos que chegam em n vindos de n – 1.B(n) = O número de percursos que chegam em n vindos de n – 2.C(n) = O número de percursos que chegam em n vindos de n + 1.A(6), B(6) e C(6) estão ilustrados na figura acima.

Page 52: Eureka 2002

Sociedade Brasileira de Matemática

EUREKA! N°13, 2002

52

Note que não há mais caminhos relevantes para o problema além destes: Qualquercaminho indo a 6 vindo de 8, passou obrigatoriamente por 6 ou 7 ou ambos. Sepassou por 6, este caminho não pode porque passaria por 6 duas vezes. Mesmo senão passou por 6, este caminho não pode: 7 e 8 já foram usados, então este caminhonão pode chegar até B.Então F(n) = A(n) + B(n) + C(n).B(n) = F (n – 2), porque nenhum caminho chegando a n – 2 passou por n, como noexemplo descrito anteriormente. EntãoB(n) = A(n – 2) + B(n – 2) + C(n – 2).A(n) = B(n – 1) + A(n – 1):

B (n – 1)

n

A(n) A (n – 1)

C (n – 1)Porque todos os C(n – 1) passaram por n (mais que isto,eles são os caminhos que vieram para n – 1 diretamentede n) e nenhum A(n – 1) ou B(n – 1) passou por n porqueeles não teriam como passar por n e voltar para n – 2 e

n – 3, respectivamente, sem repetir pontos e mantendo a possibilidade do caminhochegar a B.C(n) = B (n – 1) + A(n – 1), porque estes são os caminhos que chegam a n + 1 sempassar por n: eles vão direto de n – 1 para n + 1.Para chegar ao ponto 1:

B (1)

2

C(1) A (1)

0

1 3 B(n) = 0, não existe caminho.A(1) = 1 (caminho direto )C(1) = 1 (caminho ).

Para chegar ao ponto 2:

2

C(2) A (2)

0

1 3

B(2) 4

Os caminhos possíveis são:

Ou seja A(2) = B(2) = C(2) = 1.

A(3) = A(2) + B(2) = 2 A(5) = A(4) + B(4) = 7B(3) = A(1) + B(1) + C(1) = 2 B(5) = A(3) + B(3) + C(3) = 6C(3) = A(2) + B(2) = 2 C(5) = A(4) + B(4) = 7

A(4) = A(3) + B(3) = 4 A(6) = A(5) + B(5) = 13B(4) = A(2) + B(2) + C(2) = 3 B(6) = A(4) + B(4) + C(4) = 11C(4) = A(3) + B(3) = 4 C(6) = A(5) + B(5) = 13

Page 53: Eureka 2002

Sociedade Brasileira de Matemática

EUREKA! N°13, 2002

53

A(7) = A(6) + B(6) = 24 A(8) = A(7) + B(7) = 44B(7) = A(5) + B(5) + C(5) = 20 B(8) = A(6) + B(6) + C(6) = 37C(7) = A(6) + B(6) = 24 C(8) = A(7) + B(7) = 44

A(9) = A(8) + B(8) = 81B(9) = A(7) + B(7) + C(7) = 68C(9) = A(8) + B(8) = 81

C(10) = 0 porque não existe ponto 11.Mas ainda valeA(10) = A(9) + B(9) = 149B(10) = A(8) + B(8) + C(8) = 125F(10) = A(10) + B(10) + C(10) = 149 + 125 + 0 = 274.

PROBLEMA 5: SOLUÇÃO DE MARCIO ASSAD COHEN (RIO DE JANEIRO – RJ)

∫ +−=π

0

2 )cos21ln()( dxuxuuI

a) fazendo xt −=π ; dt = – dx:

∫ ∫ ∫ =−+−−=++=+−−−=0

0 0

222 ))(cos)(21ln() cos21ln())cos(21ln()(π

π ππ dxuxudtutudtutuuI

I(– u). Logo, I(u) = I(–u).Note agora que, como cosx = 2cos2x/2 –1:

=−+=−++=+− 2/cos4)1(2/cos421cos21 2222224242 xuuxuuuuxu

)2/cos21)(2/cos21( 22 uxuuxu +−++

Logo, ∫∫ +−+++=ππ

0

22

0

2 )2/cos21ln()2/cos21(ln)( dxuxudxuxuuI

Fazendo t = x/2 nessas últimas integrais vem:

.)cos21ln()cos21ln(2)(2/

0

22/

0

22

+−+++= ∫ ∫

π πdtutudtutuuI (II)

Agora, note que ∫∫ =+−=++=

0

/2-

2

2/

2 )cos21ln( )cos21ln(π

π

πdxuxudtutu

t-x

∫ ∫ +−=+−−=−=

0

2/

2/

0

22 )cos21ln()cos21ln(π

πdxuxudxuxu

xt (III)

De (II) e (III) vem

=

+++++= ∫ ∫

2/

0 2/

222 )(2)cos21ln()cos21ln(2)(π π

πuIdxuxudxuxuuI

Logo, I(u) = 2

1I(u2).

Page 54: Eureka 2002

Sociedade Brasileira de Matemática

EUREKA! N°13, 2002

54

b) Fazendo u = 1, vem I(1) = 2I(1) ∴ I(1) = 0. Em geral, I(u) = :),(2

1 2 xuIk

k∀⋅

I(u) = )...(2

1

4

1)(

2

1

2

1)(

2

1 842 uIuIuI ⋅=⋅= (é uma indução simples).

Supondo 0 ≤ u ≤ 1 inicialmente:

)1ln(2)1ln()cos21ln()(1cos0

2

0

2 udxudxuxuuIx +=+≤+−=⇒≤ ∫∫ πππ

Logo, xu

uIk

k

∀+

≤ ,2

)1ln(2)(

2π pois .)1ln(2

2

1)(

2

1 22 kk

uuIkk

+⋅≤ π

Fazendo k → ∞ e substituindo t = 2k, deve-se ter .0)1ln(2

lim)( =

+≤

∞→ t

uuI

t

t

π

Logo, I(u) = 0 se u ∈ [ –1, 1] (lembrando já que I(u) = I(– u)).

Se |u| > 1, faço a substituição ω1=u : Então )1( <ω :

∫ ∫ =

+−=

+−=

π π

ωωω

ωωω 0 0 2

2

2

1cos2ln

1cos21ln

1dx

xdx

xI

∫ ∫ −=−=−+−=

π π

ωπωπωωωωω 0 0

22 .ln2ln2)()ln()cos21ln(1

IdxdxxI

Logo, se ,1>u temos: I(u) = uuIu

ln2)(1

ln2 ππ =∴−

Concluindo:

>

≤=

.1 se ,ln2

1 se zero,)(

uu

uuI

π

PROBLEMA 6: SOLUÇÃO DE BRUNO FERNANDES CERQUEIRA LEITE (SÃO PAULO – SP)Fato 1: Seja 0 = (0, 0). Então f(0) = 0.Prova: Dx ∈∀ , temos .10 ≤−x Logo devemos ter !,1)0()( Dxfxf ∈∀≤− Como

a função é sobrejetora, Dy ∈∀ Dx ∈∃ 0 com .)( 0 yxf = Logo, Dy ∈∀ ,

,1)0( ≤− fy ou seja, f(0) dista no máximo 1 de qualquer ponto do disco D. Logo

f(0) = 0.

Fato 2: Sejam ryxyxBr <+= 22),( , ryxyxBr ≤+= 22),( . Então, se p ∈ Br,

.)( rBpf ∈ Se .)(, rr BpfBp ∈∈

Page 55: Eureka 2002

Sociedade Brasileira de Matemática

EUREKA! N°13, 2002

55

Prova: [ ] [ ].0 rrpBBp rr ≤<−⇔∈∈ Mas [ ].00)()0()( rrppffpf ≤<−≤−=−

Logo ).()( rr BBpf ∈∈Fato 3: Se f(p) está no bordo de D ( | f (p) – 0| = 1) então p está no bordo de D.Prova: f(p) e – f(p) são diametralmente opostos, logo .2))(()( =−− pfpf Se p não

estivesse no bordo de D, ,2, <−∈∀ qpDq absurdo pois deveríamos ter

.,,)()( Dqpqpqfpf ∈∀−≤−Fato 4: Se f(p) e f(q) são diametralmente opostos, então p e q também são.Prova: Se p e q não fossem diametralmente opostos, ,)()(2 qfpfqp −=<−absurdo.Fato 5: Se p e q são opostos tais que f(p) e f(q) são diametralmente opostos, e se x estáentre p e q então f(x) está entre f(p) e f(q). Além disso, nesse caso

,)()( xqqfxf = xppfxf =)()( e 0)0()( xfxf =Se A, B, C são pontos distintos, dizemos que B está entre A e C (e denotamos

A → B → C) se )ACBCAB =+ ). onde ( BAAB −=Prova: Sabemos, pelo fato 4, que p e q são diametralmente opostos. Se x = 0, f(x) = 0e o lema fica trivial. Suponhamos, sem perda de generalidade, x ≠ 0 e p → x → 0.

Então .00 pxpx =+ Como pxxfpf ≤)()( e ,0)0()( xfxf ≤10)0()()()( =+≤+ xpxfxfxfpf , por outro lado, pela desigualdade triangular,

)()( xfpf + .1)0()()0()( =≥ fpffxf Logo f(x) está entre f(p) e 0 e portanto está

entre f(p) e f(q), e temos ,)()( pxxfpf = xxff 0)()0( = e .)()( qxxfqf =Fato 6: Se p está no bordo de D ( |p – 0| = 1) então f(p) está no bordo de D (o bordode D é B , só para facilitar a notação). Além disso, a restrição de f a B (que temimagem B ) é uma composição de rotação com espelhamento.Prova: Seja p ∈ D tal que f(p) = (1, 0). Pelo fato 3, p ∈ B . A imagem inversa de

(–1, 0), pelo fato 4, é – p, isto é, f(– p) = (–1, 0). (– p ).B∈ Sejam q e – q as imagensinversas de (0, 1) e (– 1, 0).Então q e – q B∈ e é bem fácil ver que p, q, – p e – q formam um quadrado (ou nãoteríamos ).,,)()( Dnmnmnfmf ∈∀−≤−

Seja DDf →:~

a composição de rotação com espelhamento que coincide com f nos

pontos p, q, – p e – q. Vamos mostrar que .ˆ),(~

)( Dxxfxf ∈∀=Note que f

~ é uma bijeção, e que DDf ˆˆ:

~ 1 →− também é uma composição derotação com espelhamento.

Page 56: Eureka 2002

Sociedade Brasileira de Matemática

EUREKA! N°13, 2002

56

Sejam m, – m ∈ D . Existem n, – n ∈ B tais que f(n) = m e f(– n) = – m. Devemoster pnpfnfm −≤−=− )()()0,1( , .)()()0,1( pnpfnfm +≤−−=−− Isso já

implica que )(~ 1 mfn −= ou )(

~ 1 mfn −−= , pois |)0,1(| ||ˆ −≥−∈ mpxDx e

)(~

),(~

|)0,1( | || 11 mfmfmpx −− −=−−≥+ (de fato |)0,1(| |)(~

| |)(~

| −=−=− mpfmpmf

e .|)0,1(| )(~

| |)(~

| 1 −−=−−=+− mpfmpmf Como, além disso,|| |)()(| |)1,0(| qnqfnfm −≤−=− e || |)()(| |)1,0(| qnqfnfm +≤−−=−− , sobra

apenas a possibilidade ).(~ 1 mfn −= Como 1~ −f é sobrejetiva, dado Dx ˆ∈ existe

Dm ˆ∈ tal que )(~ 1 mfx −= , e portanto teremos ).(

~)( xfmxf ==

Agora estamos em condições de terminar a prova: de fato segue dos fatos 5 e 6 que

ff~= e logo é uma composição de rotação com espelhamento, e portanto preserva

distâncias. Com efeito, ff~= em D

~, e f leva diâmetros pq com Dpq ˆ∈−= em

diâmetros )(~

)(~

qfpf (e ),ˆ)(~

),(~

Dqfpf ∈ e tanto f quanto f~

restritos ao diâmetro

pq preservam as distâncias aos extremos, e portanto preservam distâncias, logocoincidem.

Page 57: Eureka 2002

Sociedade Brasileira de Matemática

EUREKA! N°13, 2002

57

XXIII OLIMPÍADA BRASILEIRA DE MATEMÁTICAResultado – Nível 1 (5a. e 6a. Séries)

NOME CIDADE – ESTADO PRÊMIOEduardo Fischer Encantado – RS OuroRaphael Rodrigues Mata Salvador – BA OuroGuilherme R. Nogueira de Souza São Paulo – SP OuroAndré Linhares Rodrigues Fortaleza – CE OuroAndré Martins Costa Aranha Rio de Janeiro – RJ OuroLuiz Müller Vitória – ES OuroRafael Bandeira Lages Teresina – PI Prata

Felipe Gonçalves Assis Campina Grande – PB PrataRenato Rebouças de Medeiros Fortaleza – CE PrataThaís Viveiro São Paulo – SP PrataAdriano Jorge Braun Vieira Neto Fortaleza – CE PrataJaques Deivinson da Silva Castello Serra – ES PrataEnzo Haruo Hiraoka Moriyama São Paulo – SP PrataPriscilla Yu Chen Kashiwakura São Paulo – SP PrataJefferson Quesado Neto Fortaleza – CE PrataWilliam Vasconcelos de Morais Fortaleza – CE PrataSophia Cherem Lopes Belo Horizonte – MG BronzeArthur Rodrigues de Oliveira Sobral S. J. dos Campos – SP BronzePedro Paulo Gondim Cardoso Salvador – BA Bronze

Regina Reis da Costa Alves Rio de Janeiro – RJ BronzeWeslen Costa Timóteo Paulista – PE BronzeAndré Rodrigues Salerno Goiânia – GO BronzeCaroline Goulart Campos Rio de Janeiro – RJ BronzeEdson Augusto Bezerra Lopes Fortaleza – CE BronzeLuiz Felipe Bruzzi Curi Belo Horizonte – MG BronzeBernardo de Oliveira Veiga Rio de Janeiro – RJ BronzeFelipe Alves Tomé Fortaleza – CE BronzeLuiza Cristina Maia e Silva Recife – PE BronzeIgor Ribeiro Azevedo Belo Horizonte – MG BronzeMariana Nasser Brolezzi Santo André – SP Menção HonrosaPaulo Alexandre Pavoni Curitiba – PR Menção HonrosaPaulo André Carvalho de Melo Rio de Janeiro – RJ Menção HonrosaGuilherme Pereira Barbosa Belo Horizonte – MG Menção HonrosaGustavo Sampaio Sousa Fortaleza – CE Menção HonrosaDennis G. de Macedo Bragagnolo Curitiba – PR Menção HonrosaPedro Nogueira Machado Rio de Janeiro – RJ Menção HonrosaYuriy Thallickson Bincovski Curitiba – PR Menção HonrosaGil Henriques Vassouras – RJ Menção HonrosaCássio Kendi Takamori S. J. dos Campos – SP Menção HonrosaIuri Lima Ribeiro Fortaleza – CE Menção HonrosaFranco Veronez Ribeiro Vitória – ES Menção HonrosaMateos Kruchelski Tschá Curitiba – PR Menção HonrosaKleber Varela dos Santos Jaboatão dos Guararapes – PE Menção Honrosa

Núbia Martins Domingues Belo Horizonte – MG Menção HonrosaEduardo Tadafumi Sato Mogi das Cruzes – SP Menção Honrosa

Marco Aurélio Buono Carone Belo Horizonte – MG Menção HonrosaThalles Melo de Oliveira Lopes Goiânia – GO Menção HonrosaMichel Ricardo Nigri Rio de Janeiro – RJ Menção HonrosaRaquel Pereira Martins Rio de Janeiro – RJ Menção Honrosa

Page 58: Eureka 2002

Sociedade Brasileira de Matemática

EUREKA! N°13, 2002

58

Resultado – Nível 2 (7a. e 8a. Séries)

NOME CIDADE – ESTADO PRÊMIOThiago Costa Leite Santos São Paulo – SP OuroHenry Wei Cheng Hsu São Paulo – SP OuroRafael Daigo Hirama Campinas – SP OuroRodrigo Aguiar Pinheiro Fortaleza – CE OuroMarcus Edson Barreto Brito Fortaleza – CE PrataDaniela Satie Kondo São Paulo – SP PrataTelmo Luis Correa Junior Santo André – SP PrataAlan Hideki Uchida São Paulo – SP PrataFelipe Rodrigues Nogueira de Souza São Paulo – SP PrataDiogo dos Santos Suyama Belo Horizonte – MG PrataRicardo Mizoguchi Gorgoll São Paulo – SP PrataKaroline Matias Morais Fortaleza – CE PrataPaulo Roberto Sampaio Santiago Salvador – BA PrataMarcela Sobrinho Pereira Fortaleza – CE PrataThomás Yoiti Sasaki Hoshina Rio de Janeiro – RJ Prata

Guilherme Rodrigues Salerno Goiânia – GO BronzeHector Kenzo Horiuti Kitahara São Paulo – SP BronzeMauro Cardoso Lopes São Paulo – SP BronzeAndré Lucas Ribeiro dos Santos Pindamonhangaba – SP BronzeRenata Sayuri Takehara S. J. dos Campos – SP BronzeHenrique Castro Noronha Valinhos – SP BronzeMatheus Migliolo Coelho Limeira – SP BronzeLucas de Freitas Frenay Santo André – SP BronzeRafael Marini Silva Vila Velha – ES BronzeAndré Slepetys São Paulo – SP BronzeLuiza Fontana Barbosa Curitiba – PR BronzeJefferson Fonlin Tsai São Paulo – SP BronzeDeborah Regina Fujisawa Okuno São Paulo – SP BronzeFelipe Paupitz Schlichting Florianópolis – SC BronzeElton Gomes Coriolano Fortaleza – CE BronzeÁlison Santos Xavier Fortaleza – CE Menção HonrosaMarcus Vinícius Martins da Costa Belo Horizonte – MG Menção HonrosaRodrigo Viana Soares Fortaleza – CE Menção HonrosaLucas M. Pereira Castello Branco Fortaleza – CE Menção HonrosaLarissa Rodrigues Ribeiro Fortaleza – CE Menção HonrosaThiago Jorge Marinho Vieira Fortaleza – CE Menção HonrosaCincinato Furtado Leite Neto Fortaleza – CE Menção HonrosaRafael Kitayama Shiraiwa São Paulo – SP Menção HonrosaAnderson Hoshiko Aiziro São Paulo – SP Menção HonrosaGuilherme Alonso Daud Patavino Santos – SP Menção HonrosaVitor Humia Fontoura Salvador – BA Menção HonrosaAndré Schultz Santa Bárbara D'Oeste – SP Menção HonrosaGabriel Tavares Bujokas São Paulo – SP Menção HonrosaFrancisco Bruno de Lima Holanda Fortaleza – CE Menção HonrosaGustavo Eufrásio Farias Fortaleza – CE Menção HonrosaKatja Stephanie Ried Valinhos – SP Menção HonrosaRaul Máximo Alexandrino Nogueira Fortaleza – CE Menção HonrosaMarcos Vainer Loeff São Paulo – SP Menção HonrosaAntonia Taline de Souza Mendonça Fortaleza – CE Menção HonrosaJúlio Vitório dos Santos Ferreira Rio de Janeiro – RJ Menção Honrosa

Page 59: Eureka 2002

Sociedade Brasileira de Matemática

EUREKA! N°13, 2002

59

Resultado – Nível 3 (Ensino Médio)

NOME CIDADE – ESTADO PRÊMIOHumberto Silva Naves São José dos Campos – SP OuroDavi Máximo Alexandrino Nogueira Fortaleza – CE Ouro

Larissa Cavalcante Queiroz de Lima Fortaleza – CE OuroCarlos Stein Naves de Brito Goiânia – GO OuroAlex Corrêa Abreu Niterói – RJ OuroDaniel Pinheiro Sobreira Fortaleza – CE PrataEinstein do Nascimento Júnior Fortaleza – CE PrataGuilherme Fujiwara São Paulo – SP PrataThiago Barros Rodrigues Costa Fortaleza – CE PrataRafael Tajra Fonteles Teresina – PI PrataEduardo Famini Silva Salvador – BA PrataRodrigo Roque Dias São Paulo – SP Prata

Fábio Dias Moreira Rio de Janeiro – RJ PrataDaniel Pessôa Martins Cunha Fortaleza – CE BronzeYuri Gomes Lima Fortaleza – CE BronzePaulo Ribeiro de Almeida Neto Ananindeua – PA BronzeThiago da Silva Sobral Fortaleza – CE BronzeGermanna de Oliveira Queiroz Fortaleza – CE BronzeBernardo Freitas Paulo da Costa Rio de Janeiro – RJ Bronze

Samuel Barbosa Feitosa Fortaleza – CE BronzeIsaac Newton Ferreira Santa Rita Nova Iguaçu – RJ BronzeJosé Luiz Gomes Junior Belém – PA BronzeAyran Ayres Barbosa Loriato Vitória – ES BronzeFernanda Maria de Oliveira Nicacio Fortaleza – CE BronzeHenrique Chociay Pinhais – PR BronzeJoão Alfredo Castellani Fajardo Freire Salvador – BA BronzeRafael da Silva Faria Rio de Janeiro – RJ BronzeIsrael Franklim Dourado Carrah Fortaleza – CE Menção HonrosaArtur Duarte Nehmi São Paulo – SP Menção HonrosaLucas de Melo Pontes e Silva Fortaleza – CE Menção HonrosaDiogo Luiz Duarte Rio de Janeiro – RJ Menção Honrosa

Estillac B. Filho Belém – PA Menção HonrosaAlex Cardoso Lopes São Paulo – SP Menção HonrosaArthur M. Rocha de Azevedo Scalercio Belém – PA Menção HonrosaDiego Silva Dias Belém – PA Menção HonrosaMartha Priscilla Araújo de Moraes Fortaleza – CE Menção HonrosaRicardo Monteiro da Silva Lanna Belo Horizonte – MG Menção HonrosaFernando Souza Martins S. J. dos Campos – SP Menção HonrosaMaurício Richartz Curitiba – PR Menção HonrosaLéo Tsukui Belém – PA Menção HonrosaVitor Gabriel Kleine Mogi das Cruzes – SP Menção Honrosa

Page 60: Eureka 2002

Sociedade Brasileira de Matemática

EUREKA! N°13, 2002

60

Resultado – Nível Universitário

NOME CIDADE – ESTADO PRÊMIOEmanuel Augusto de Souza Carneiro Fortaleza – CE OuroCarlos Yuzo Shine São Paulo – SP OuroDaniel Massaki Yamamoto São Paulo – SP OuroFabrício Siqueira Benevides Fortaleza – CE PrataDiêgo Veloso Uchôa Teresina – PI PrataFrederico Vale Girão Fortaleza – CE PrataBruno Fernandes Cerqueira Leite São Paulo – SP PrataMarcio Afonso Assad Cohen Rio de Janeiro – RJ PrataLucas Heitzmann Gabrielli São Paulo – SP PrataChristian Iveson São Paulo – SP BronzeDaniel Nobuo Uno São Paulo – SP BronzeGiuliano Boava Florianópolis – SC BronzeVinícius José Fortuna Campinas – SP BronzeLeonardo Augusto Zão Nilópolis – RJ BronzeLeandro de Mattos Ferreira Rio de Janeiro – RJ BronzeRodrigo Villard Milet Rio de Janeiro – RJ BronzeTertuliano Franco Santos Franco Salvador – BA BronzeAleksander Medella Campos da Silva Rio de Janeiro – RJ BronzeArnaldo João do Nascimento Junior Duque de Caxias – RJ BronzeArtur Radoman de Oliveira Rio de Janeiro – RJ BronzeBruno Germano Borics Rio de Janeiro – RJ BronzeThiago Afonso de André São Paulo – SP BronzeJuliana Abrantes Freire Rio de Janeiro – RJ BronzeFelipe Duarte Cardozo de Pina Rio de Janeiro – RJ BronzeFernando Prado Rocha Goiânia – GO Menção HonrosaCamilo Marcantonio Junior Rio de Janeiro – RJ Menção Honrosa

Dúlio Matos Leite de Carvalho e Silva Rio de Janeiro – RJ Menção HonrosaDiogo Diniz Pereira da Silva e Silva Campina Grande – PB Menção HonrosaIlan Lobel Rio de Janeiro – RJ Menção HonrosaRafael Pellizzer Soares Jundiaí – SP Menção HonrosaAnderson Rodrigues Ferreira Rio de Janeiro – RJ Menção HonrosaRafael de Freitas Lemos S. J. dos Campos – SP Menção HonrosaBruno Martins Reboredo Rio de Janeiro – RJ Menção Honrosa

Daniele Véras de Andrade Rio de Janeiro – RJ Menção Honrosa

Page 61: Eureka 2002

Sociedade Brasileira de Matemática

EUREKA! N°13, 2002

61

AGENDA OLÍMPICA

XXIV OLIMPÍADA BRASILEIRA DE MATEMÁTICA

NÍVEIS 1, 2 e 3Primeira Fase – Sábado, 8 de junho de 2002

Segunda Fase – Sábado, 14 de setembro de 2002Terceira Fase – Sábado, 19 de outubro de 2002 (níveis 1, 2 e 3)

Domingo, 20 de outubro de 2002 (níveis 2 e 3 - segundo dia de prova).

NÍVEL UNIVERSITÁRIOPrimeira Fase – Sábado, 14 de setembro de 2002

Segunda Fase – Sábado, 19 e Domingo, 20 de outubro de 2002♦

VIII OLIMPÍADA DE MAIOmaio de 2002

XIII OLIMPÍADA DE MATEMÁTICA DO CONE SUL22 a 28 de junho de 2002

Fortaleza – CE, Brasil♦

XLIII OLIMPÍADA INTERNACIONAL DE MATEMÁTICA18 a 31 de julho de 2002Glasgow, Reino Unido

♦XVII OLIMPÍADA IBEROAMERICANA DE MATEMÁTICA

30 de setembro a 5 de outubro de 2002El Salvador

♦V OLIMPÍADA IBEROAMERICANA DE MATEMÁTICA UNIVERSITÁRIA

outubro de 2002

♦♦ ♦

Page 62: Eureka 2002

Sociedade Brasileira de Matemática

EUREKA! N°13, 2002

62

COORDENADORES REGIONAIS

Amarísio da Silva Araújo (UFV) Viçosa – MG

Alberto Hassen Raad (UFJF) Juiz de Fora – MG

Benedito Tadeu Vasconcelos Freire (UFRN) Natal – RN

Carlos Frederico Borges Palmeira (PUC-Rio) Rio de Janeiro – RJ

Claudio Arconcher (Colégio Leonardo da Vinci) Jundiaí – SP

Claus Haetinger (UNIVATES) Lajeado – RS

Cleonor Crescêncio das Neves (UTAM) Manaus – AM

Élio Mega (Colégio Etapa) São Paulo – SP

Rosângela Souza (Colégio Singular) Santo André – SP

Florêncio Ferreira Guimarães Filho (UFES) Vitória – ES

Gisele de Araújo Prateado Gusmão (UFGO) Goiânia – GO

Ivanilde Fernandes Saad (UC. Dom Bosco) Campo Grande– MS

Jacqueline Fabiola Rojas Arancibia (UFPB) João Pessoa – PB

João Benício de Melo Neto (UFPI) Teresina – PI

João Francisco Melo Libonati (Grupo Educacional Ideal) Belém – PA

Irene Nakaoka (UEM) Maringá – PR

José Carlos Pinto Leivas (UFRG) Rio Grande – RS

José Cloves Saraiva (UFMA) São Luis – MA

José Gaspar Ruas Filho (ICMC-USP) São Carlos – SP

José Luiz Rosas Pinho (UFSC) Florianópolis – SC

José Vieira Alves (UFPB) Campina Grande – PB

Marcelo Rufino de Oliveira (Grupo Educacional Ideal) Belém – PA

Licio Hernandes Bezerra (UFSC) Florianópolis – SC

Luzinalva Miranda de Amorim (UFBA) Salvador – BA

Marcondes Cavalcante França (UFC) Fortaleza – CE

Pablo Rodrigo Ganassim (Liceu Terras do Engenho) Piracicaba – SP

Reinaldo Gen Ichiro Arakaki (INPE) SJ dos Campos – SP

Ricardo Amorim (Centro Educacional Logos) Nova Iguaçu – RJ

Roberto Vizeu Barros (Colégio Acae) Volta Redonda – RJ

Sérgio Cláudio Ramos (IM-UFRGS) Porto Alegre – RS

Silvio de Barros Melo (UFPE) Recife – PE

Tadeu Ferreira Gomes (UEBA) Juazeiro – BA

Tomás Menéndez Rodrigues (U. Federal de Rondônia) Porto Velho – RO

Valdenberg Araújo da Silva (U. Federal de Sergipe) São Cristovão – SE

Wagner Pereira Lopes (Escola Técnica Federal de Goiás) Jataí – GO

Page 63: Eureka 2002

CONTEÚDO

AOS LEITORES 2

VIII OLIMPÍADA DE MAIO 3Enunciados e Resultado Brasileiro

XIII OLIMPÍADA DE MATEMÁTICA DO CONE SUL 6Enunciados, Soluções e Resultado Brasileiro

XLIII OLIMPÍADA INTERNACIONAL DE MATEMÁTICA 17Enunciados e Resultado Brasileiro

ARTIGOS

MUROS, PRÉDIOS E ESCADAS 19Cícero de Oliveira Holmer

INTEIROS DE GAUSS E INTEIROS DE EISENSTEIN 23Guilherme Fujiwara

SEQÜÊNCIAS ARITMÉTICO-GEOMÉTRICAS 32José Paulo Carneiro & Carlos Gustavo Moreira

O PRINCÍPIO DA INVARIÂNCIA 35Marcelo Rufino de Oliveira

TORNEIO DAS CIDADES 43Provas

OLIMPÍADAS AO REDOR DO MUNDO 48

SOLUÇÕES DE PROBLEMAS PROPOSTOS 53

PROBLEMAS PROPOSTOS 59

AGENDA OLÍMPICA 61

COORDENADORES REGIONAIS 62

Page 64: Eureka 2002

Sociedade Brasileira de Matemática

EUREKA! N°14, 2002

2

AOS LEITORES

É com grande alegria que anunciamos que, mais uma vez, os 6 estudantes daequipe brasileira obtiveram medalhas na IMO. Isto mostra a nossa grande evoluçãocom a nova OBM e as atividades extras que apareceram com ela: a Eureka, aSemana Olímpica, as semanas de treinamento antes das competições internacionais.Mas o que nos traz maior satisfação é saber que esses 6 jovens são a ponta de umiceberg. Basta observar o número de pessoas que resolvem os vários problemas quetrazemos a cada nova edição da Eureka. São estudantes, professores, profissionaisliberais, enfim, amantes da Matemática que provam que no nosso país há muitaspessoas de boa vontade e de grande competência.

Falando mais um pouco sobre problemas, nesse número publicamos algumasprovas do Torneio das Cidades, uma competição que se caracteriza pelaoriginalidade de suas questões, algumas das quais já entraram para o folclorematemático. Ela possui duas modalidades, Sênior (2a e 3a séries EM) e Júnior (8a

série EF e 1a série EM); níveis O (iniciante) e A (avançado).

Para encerrar, os agradecimentos. Mais uma vez, o professor Carlos Shine eos estudantes Alex Lopes, Felipe de Souza, Henry Hsu, Rodrigo Yamashita eGuilherme Fujiwara fizeram uma leitura cuidadosa das versões prévias desta edição.

Os editores.

Page 65: Eureka 2002

Sociedade Brasileira de Matemática

EUREKA! N°14, 2002

3

VIII OLIMPÍADA DE MAIOEnunciados e Resultado Brasileiro

PRIMEIRO NÍVEL

PROBLEMA 1Um grupo de homens, alguns dos quais acompanhados pelas esposas, gastaram 1000dólares num hotel. Cada homem gastou 19 dólares e cada mulher, 13 dólares.Determine quantas mulheres e quantos homens estavam no hotel.

PROBLEMA 2Uma folha de papel retangular (branca de um lado e cinza do outro) foi dobrada trêsvezes, como mostra a figura abaixo:

1 2 3

O retângulo 1, que ficou da cor branca após a primeira dobra, tem 20cm a mais deperímetro que o retângulo 2, que ficou branco após a segunda dobra, e este por suavez tem 16cm a mais de perímetro que o retângulo 3, que ficou branco após aterceira dobra. Determine a área da folha.

PROBLEMA 3Mustafá comprou um tapete. O vendedor mediu o tapete com uma régua quesupostamente media um metro. Como o resultado foi que o tapete tinha 30 metros delargura e 20 metros de comprimento, o vendedor cobrou 120000 rupias. QuandoMustafá chegou a sua casa mediu novamente o tapete e percebeu que o vendedortinha cobrado 9408 rupias a mais. Quantos centímetros mede a régua usada pelovendedor?

PROBLEMA 4Num banco só o diretor conhece o segredo do cofre, que é um número de cincodígitos. Para proteger este segredo são dados a cada um dos dez empregados dobanco um número de cinco dígitos. Cada um destes números tem numa das cincoposições o mesmo dígito que o segredo e nas outras quatro posições um dígitodiferente do que tem o segredo nesse lugar. Os números de proteção são:

07344, 14098, 27356, 36429, 45374, 52207, 63822, 70558, 85237, 97665.

Qual é o segredo do cofre?

Page 66: Eureka 2002

Sociedade Brasileira de Matemática

EUREKA! N°14, 2002

4

PROBLEMA 5Encontre o máximo número de caixinhas de 3 × 5 × 7 que podem ser colocadasdentro de uma caixa de 11 × 35 × 39. Para o número encontrado, indique comocolocar essa quantidade de caixinhas dentro da caixa.

SEGUNDO NÍVEL

PROBLEMA 1Utilizando cubinhos brancos de lado 1 foi montado um prisma (sem buracos).As faces do prisma foram pintadas de preto. Sabe-se que os cubinhos que ficaramcom exatamente 4 faces brancas são 20 no total. Determine quais podem ser asdimensões do prisma. Encontre todas as possibilidades.

PROBLEMA 2Seja k um número inteiro positivo fixo, k ≤ 10.Dada uma lista de dez números, a operação permitida é: escolher k números da lista,e somar 1 a cada um deles. Obtém-se assim uma nova lista de dez números.Se inicialmente temos a lista 1, 2, 3, 4, 5, 6, 7, 8, 9, 10, determine os valores de kpara os quais é possível, mediante uma seqüência de operações permitidas, obteruma lista que tenha os dez números iguais. Indique a seqüência para cada caso.

PROBLEMA 3Num triângulo ABC, retângulo em A e isósceles, seja D um ponto do lado AC (D ≠ Ae D ≠ C) e seja E o ponto do prolongamento do lado BA tal que o triângulo ADE éisósceles. Se P é o ponto médio do segmento BD, R é o ponto médio do segmentoCE e Q o ponto onde se cortam as retas ED e BC, demonstre que o quadriláteroARQP é um quadrado.

PROBLEMA 4Os vértices de um polígono regular de 2002 lados estão numerados de 1 a 2002, nosentido horário. Dado um inteiro n, 1 ≤ n ≤ 2002, pinta-se de azul o vértice n, logo,seguindo o sentido horário, contam-se n vértices começando no seguinte de n, epinta-se de azul o número n. E assim sucessivamente, a partir do vértice que segueao último vértice que há sido pintado, contam-se n vértices, pintados ou sem pintar, eo número n é pintado de azul. Quando o vértice que tem que ser pintado já é azul, oprocesso pára. Denotamos P(n) ao conjunto de vértices azuis que se obtém com esteprocedimento quando se começa pelo vértice n. Por exemplo, P(364) está formadopelos vértices 364, 728, 1092, 1456, 1820, 182, 546, 910, 1274, 1638 e 2002.Determine todos os inteiros n, 1 ≤ n ≤ 2002, tais que P(n) tem exatamente 14vértices.

Page 67: Eureka 2002

Sociedade Brasileira de Matemática

EUREKA! N°14, 2002

5

PROBLEMA 5Dados x e y inteiros positivos, consideramos um quadriculado de x × y, que tempintados de vermelho os (x + 1) ⋅ (y + 1) pontos que são vértices de quadradinhos.Inicialmente há uma formiga em cada um dos pontos vermelhos. Num instante dado,todas as formigas começam a caminhar pelas linhas do quadriculado, todas com amesma velocidade. Cada vez que chegam num ponto vermelho, giram 90° emalguma direção.Determine todos os valores de x e y para os quais é possível que as formigascontinuem movendo-se indefinidamente de maneira que em nenhum momento háduas ou mais formigas num mesmo ponto vermelho. (Não interessam as possíveiscoincidências em pontos das linhas do quadriculado que não são vermelhos.)

RESULTADO BRASILEIRO

PRIMEIRO NÍVEL (ATÉ 13 Anos)Eduardo Fischer Encantado – RS Medalha de OuroPedro Nogueira Machado Rio de Janeiro – RJ Medalha de PrataAndré Márcio de Lima Curvello Goiânia – GO Medalha de PrataKatja Stephanie Ried Valinhos – SP Medalha de BronzeEnzo Haruo Hiraoka Moriyama São Paulo – SP Medalha de BronzeAnderson Gleryston Silva Sousa Campina Grande – PB Medalha de BronzeMariana Nasser Brolezzi Santo André – SP Medalha de BronzeArthur Rodrigues de Oliveira Sobral S. José dos Campos – SP Menção HonrosaCássio Kendi Takamori S. José dos Campos – SP Menção HonrosaDiogo Bonfim Moraes Morant de Holanda Rio de Janeiro – RJ Menção Honrosa

SEGUNDO NÍVEL (ATÉ 15 Anos)Fábio Dias Moreira Rio de Janeiro – RJ Medalha de OuroGuilherme Rodrigues Salerno Goiânia – GO Medalha de PrataTelmo Luis Correa Júnior Santo André – SP Medalha de PrataThiago Costa Leite Santos São Paulo – SP Medalha de BronzeAndré Rodrigues Salerno Goiânia – GO Medalha de BronzeHenry Wei Cheng Hsu São Paulo – SP Medalha de BronzeRodrigo Aguiar Pinheiro Fortaleza – CE Medalha de BronzeRafael Marini Silva Vila Velha – ES Menção HonrosaLarissa Rodrigues Ribeiro Fortaleza – CE Menção HonrosaDouglas Bokliang Ang Cunha S. José dos Campos – SP Menção Honrosa

Page 68: Eureka 2002

Sociedade Brasileira de Matemática

EUREKA! N°14, 2002

6

XIII OLIMPÍADA DE MATEMÁTICA DO CONE SULEnunciados, Soluções e Resultado Brasileiro

A XIII Olimpíada de Matemática do Cone Sul foi realizada na cidade deFortaleza, Ceará no período de 22 a 28 de junho de 2002.

A equipe brasileira foi liderada pelos professores Yoshiharu Kohayakawa(São Paulo – SP) e Luciano Guimarães Castro (Rio de Janeiro – RJ).

O Resultado da Equipe Brasileira

BRA 1 Alex Corrêa Abreu OuroBRA 2 Israel Dourado Carrah BronzeBRA 3 Larissa Cavalcante Queiroz de Lima OuroBRA 4 Rafael Daigo Hirama Ouro

PRIMEIRO DIADURAÇÃO: 4 horas e meia.

PROBLEMA 1:Os alunos da turma de Pedro praticam a soma e a multiplicação de números inteiros.A professora escreve os números de 1 a 9 em nove fichas, uma para cada número, eas coloca em uma urna. Pedro retira três fichas e deve calcular a soma e o produtodos três números correspondentes. Ana e Julián fazem o mesmo, esvaziando a urna.Pedro informa à professora que retirou três números consecutivos cujo produto é 5vezes a soma. Ana informa que não tem nenhum número primo, mas sim doisconsecutivos e que o produto desses três números é 4 vezes a soma dos mesmos.Quais números retirou Julián?

SOLUÇÃO DE ISRAEL FRANKLIM DOURADO CARRAH (FORTALEZA – CE):Diremos que Pedro escolheu os números P1, P1 + 1 e P1 + 2, Ana retirou os númerosA1, A2 e A3. Logo, temos que:

)1(53)33(5)21(5)2()1( 11111111 +⋅⋅=+⋅=++++⋅=+⋅+⋅ PPPPPPPP e como

(P1 + 1) é um número positivo ⇒ ⇒=⋅=+⋅ 1553)2( 11 PP 1P e )2( 1 +P sãodivisores de 15 (obviamente P1 + 2 > P1). Assim, temos duas possibilidades:

==→=+=

113152

1.1

11

1

PP

Pa Absurdo!

=→=+=

352

3.2

11

1

PP

Pa Ok!

Portanto, Pedro escolheu os números 3, 4 e 5.

Page 69: Eureka 2002

Sociedade Brasileira de Matemática

EUREKA! N°14, 2002

7

Como Ana escolheu 3 números que não são primos ⇒ os possíveis númerosretirados por Ana são 1, 6, 8 e 9. (pois 2 e 7 são números primos e 3, 4 e 5 sãonúmeros que já foram retirados por Pedro.)Mas temos também que Ana retirou dois números consecutivos e dentre (1, 6, 8 e 9)os únicos dois números consecutivos são 8 e 9 ⇒ Ana escolheu os números 8 e 9.Assim: A2 = 8, A3 = 9 e A1 ⋅ 8 ⋅ 9 = 4 ⋅ (A1 + 8 + 9) ⇒ A1 ⋅ 18 = A1 + 17 ⇒ 17A1 = 17⇒ A1 = 1. Logo, Ana escolheu os números 1, 8 e 9.Portanto, Julián retirou os números restantes: 2, 6 e 7.

PROBLEMA 2:De um triângulo ABC, retângulo em A, conhecemos: o ponto T de tangência dacircunferência inscrita em ABC com a hipotenusa BC, o ponto D de interseção da

bissetriz interna do ângulo B com o lado AC e o ponto E de interseção da bissetriz

interna do ângulo C com o lado AB. Descreva uma construção com régua ecompasso para obter os pontos A, B e C. Justifique.

SOLUÇÃO DE LARISSA CAVALCANTE QUEIROZ DE LIMA (FORTALEZA – CE): A

E D N

T W Y C

B

. .

.

β β θ

θ

BC = a, AC = b, AB = c; 2

cbap

++=

⇒ CT = p – c; BT = p – bsejam Y tal que EY ⊥ BC e W tal queDW ⊥ BC

°==

=∆≡∆

90ˆˆ

bissetriz) ( ˆˆ

comum

*

CYEEAC

CEYCEECA

EC

AECEYC

===

⇒bCYAC

EYAE

*

=

°==∆≡∆

bissetriz) (ˆˆ

90ˆˆ

comum

DBWBDDBA

DABDWB

BD

ADBDWB

===

⇒cWBAB

ADDW

Page 70: Eureka 2002

Sociedade Brasileira de Matemática

EUREKA! N°14, 2002

8

* WT = WB – TB = c – (p – b) = (c + b + a) – a – p = 2p – p – a = p – a* YT = YC – CT = b – (p – c) = c + b – p = p – a

⇒ T é ponto médio de WY. *DEYW é um trapézio retângulo ( )°== 90ˆˆ EYWYWD .

Construção do ∆ ABC.Dados D e E, é fácil obter com régua e compasso o ponto N, ponto médio de DE.Sabemos que os pontos D, E, Y e W do ∆ABC formam um trapézio retângulo, sendoT o ponto médio de YW ⇒ NT será base média e NT//EY//DW ⇒

BCNTYTN ⊥⇒°= 90ˆ ⇒ Conhecemos já a reta NT, com compasso, marcamos.';' TNTNNTN =∈

A mediatriz de NN'é perpendicular a NN' em T, portanto coincide com a reta BC ⇒conhecemos agora a reta BC .

* ⇒°= 90ˆTYE para encontrar Y, encontramos M1, ponto médio de ET e contruímoscom compasso a circunferência Γ1 de centro M1 passando por E e T. Onde Γ1

encontrar a reta BC será o ponto Y (note que será no segundo ponto de encontro

com BC , o primeiro é T). Analogamente, encontramos o ponto W no encontro da

circunfrência Γ2 de diâmetro DT e da reta BC .

Dessa maneira, encontramos EY e .DW Com o compasso, encontramos Γ3 de raioEY e centro E, e Γ4 de raio DW e centro D. ',43 AA=Γ∩Γ A será o ponto que está"acima"de DE ( supondo o ponto T "abaixo"de DE).

Encontramos então as retas AE e AD .

BCAEB ∩= e BCADC ∩=∴ Encontramos então ∆ABCobs: Se Y coincidir com T, temos que W coincidirá com T e p – a = 0 ou seja,

acbacbaacba =+⇔=−++⇔=−++

0202

Absurdo!

TY ≠⇒ e .TW ≠

PROBLEMA 3:Arnaldo e Bernardo jogam uma Super Batalha Naval. Cada um tem um tabuleiro n xn. Arnaldo coloca barcos em seu tabuleiro (pelo menos um mas não se sabequantos). Cada barco ocupa as n casas de uma linha ou de uma coluna e os barcosnão podem se superpor nem ter um lado comum. Bernardo marca m casas(representando tiros) em seu tabuleiro. Depois que Bernardo marcou as m casas,Arnaldo diz quais dentre elas correspondem a posições ocupadas por barcos.Bernardo ganha se, a seguir, descobre quais são as posições de todos os barcos de

Page 71: Eureka 2002

Sociedade Brasileira de Matemática

EUREKA! N°14, 2002

9

Arnaldo. Determine o menor valor de m para o qual Bernardo pode garantir suavitória.

SOLUÇÃO DE ALEX CORRÊA ABREU (NITERÓI – RJ):Suponha n ≥ 4(I) Primeiro vemos que obviamente tem que marcar casas em todas as linhas ecolunas, pois se uma linha não tiver nenhuma casa marcada obviamente podemos terum barco em uma linha não adjacente portanto Bernardo não saberá se ali tem ounão um barco. De modo análogo para as colunas.

(II) Agora suponha que uma casa está marcada, se na linha e na coluna dessa casanão tiver mais nenhuma outra casa marcada, pode vir a calhar de arnaldo tercolocado apenas um barco no tabuleiro e ser exatamente nessa linha, portantoBernardo não sabe se o barco está na vertical ou na horizontal pois só vai ter umacasa onde sabe que o barco está. Então para cada casa marcada existe outra na linhaou na coluna.

k

k1

k2

Considere agora que Arnaldo marcou um barco na linha i⇒ se uma casa é marcada na linha i, para identificar se obarco está na horizontal ou na vertical precisamos de maisuma casa adjacente. Se esta estiver pintada o barco estána vertical (baseado no desenho) e na horizontal casocontrário.

O problema consiste em pintar blocos 1 × k tais que cadalinha e cada coluna tem interseção com um deles, agorase tivermos um 1 × k se k > 3 podemos dividir e de fatoapenas melhoramos as coisas então os blocos são 1 × 2 e1 × 3.

M(n) é o mínimo procurado para um tabuleiro n × n.

Page 72: Eureka 2002

Sociedade Brasileira de Matemática

EUREKA! N°14, 2002

10

A

A

1

3 Se n ≥ 3 retire as 3 primeiras linas e as 3 primeirascolunas então m(n) = m(3) + m (n – 3) pois precisamos dem(n – 3) pois se não tiver nenhum barco nas 6 fileiras quesaíram e m(3) se não tiver nas que ficaram pois seconsiderarmos só a região A.temos que colocar 3 em cada no mínimo, o que dá mais 6contra 4 de m(3) ⇒ (também porque podemos diminuir asintersecções) ⇒

+=+−=++=+−=+−=+

==

34)5()1(4)23(

24644)4()3()1()13(

4)3()3(

kmkkm

kkmmkkm

kkmkm

é fácil ver que m(3) = 4 pois 3 obviamente não é e m(4) = 6, m (5) = 7 como ao lado.

SEGUNDO DIADURAÇÃO: 4 horas e meia.

PROBLEMA 4:Seja ABCD um quadrilátero convexo tal que suas diagonais AC e BD sãoperpendiculares. Seja P a interseção de AC e BD e seja M o ponto médio de AB.Mostre que o quadrilátero ABCD é inscritível se, e somente se, as retas PM e CD sãoperpendiculares.

SOLUÇÃO DE ISRAEL FRANKLIM DOURADO CARRAH (FORTALEZA – CE):

Primeiramente vejamos quando PM e CD são perpendiculares

A M

B

D C K

90° – θ

90° – θ θ

P 90° – θ

θ . .

θ 2θ

Page 73: Eureka 2002

Sociedade Brasileira de Matemática

EUREKA! N°14, 2002

11

Seja .KCDMP =∩ Como no ABP∆ , retângulo em P, M é o ponto médio da

hipotenusa .MBMAPMAB ==⇒ Assim, seja

⇒=⇒=⇒= θθθ 2ˆˆˆ PMABPMDBAθθ −°==⇒−°= 90ˆˆ90ˆ MPAKPCAPM e como

.ˆ90ˆ θ=⇒°= DCPCKP Logo, ⇒== θDCADBA ˆˆ O quadrilátero ABCD éinscritível!Vejamos agora se ABCD é inscritível:

A M

B

D C K

90° – θ

90° – θ

P 90° – θ

θ

θ

Do mesmo modo como M é o ponto médio da hipotenusa AB do triângulo retângulo

.MBMAPMAPB ==⇒Logo, se θθθ −°=⇒−°==⇒= 90ˆ90ˆˆˆ KPCAPMPABDBA e como ABCD é

inscritível CDMPCKPDBADCA ⊥⇒°=+−°−°=⇒==⇒ 90)90(180ˆˆˆ θθθ .Portanto, ABCD é inscritível .CDPM⊥⇔

PROBLEMA 5:Considere o conjunto A = 1, 2,…, n. Para cada inteiro k, seja rk a maior quantidadede elementos distintos de A que podemos escolher de maneira que a diferença entredois números escolhidos seja sempre diferente de k. Determine o maior valor

possível de rk, onde .2

1n

k ≤≤

Page 74: Eureka 2002

Sociedade Brasileira de Matemática

EUREKA! N°14, 2002

12

SOLUÇÃO DE RAFAEL DAIGO HIRAMA (CAMPINAS – SP):Vamos analizar casos pequenos:1, 2, 3, 4, 5, 6, 7, 8 k = 1 rk = 4 1, 2, 3, 4, 5, 6, 7, 8, 9 k = 1 rk = 5

k = 2 rk = 4 k = 2 rk = 5k = 3 rk = 5 k = 3 rk = 6k = 4 rk = 4 k = 4 rk = 5

1, 2, 3, 4, 5, 6, 7, 8, 9, 10k = 1 rk = 5k = 2 rk = 6k = 3 rk = 6k = 4 rk = 6k = 5 rk = 5

Isso me deu idéia para um lema!

Lema: Para n = m ⋅ k (m inteiro positivo maior que 1) temos que

+⋅=

2

1mkrk

Prova: Podemos dividir os números em casas de congruências módulo k. Porexemplo o 0.Seus componentes serão k, 2k, 3k,...,mk. Como a diferença entre dois deles deve serdiferente de k, não podemos escolher dois números consecutivos nessa seqüência.Sempre deve haver um "ausente" ou mais entre dois "presentes". Para m par temos

que só poderemos ter 2

m escolhidos pois caso tenhamos 1

2+m

escolhidos teremos

12

−m não escolhidos, mas para separar os escolhidos (para não serem consecutivos)

deveríamos ter pelo menos 2

mnão escolhidos. Absurdo.

Para m ímpar teremos 2

1+mescolhidos que podem ser espaçados pelos 1

2

1 −+mnão

escolhidos. Do mesmo modo, para 12

1 ++mdeveríamos ter

2

1+mespaços mas só

teriam sobrado 2

31

2

1 −=

++− mm

m não escolhidos que não são suficientes.

Page 75: Eureka 2002

Sociedade Brasileira de Matemática

EUREKA! N°14, 2002

13

Como temos k casas de congruência com m números cada e pelo fato de

par é se

2

ímpar é se 2

1

2

1

+

=

+

mm

mm

mtemos que .

2

1

+⋅= m

krk

Agora precisamos ver como transformar o lema em algo que seja mais versátil aonosso problema, ou seja, não devemos ter de usar o fato n = m ⋅ k.

Analizando mais casos pequenos estou conjecturando que o rk máximo é .3

2

n

Vamos provar que 3

2nrk ≤ para todo k.

Suponha o contrário, que há 3

2nrk > . Logo vamos provar primeiro que em uma casa

de congruência módulo k com j termos o aproveitamento máximo de termos é 3

2do

total j de termos. Temos a seguinte regra: se x foram escolhidos então pelo menos x –

1 não podem ter sido. Logo devemos provar que .323

2xj

j

x ≥⇒≤ Sabemos que

,1−+≥ xxj portanto vale [ ] .23243)1(232 ≥⇔≥−⇔≥−+⇐≥ xxxxxxxjPara 2≥x já está provado. Mas se escolhermos x = 1 necessariamente j = 2 pois se j= 1, ou seja, se há somente um número entre 1 e n com congruência módulo k

significa que 2k > n, então 2

nk > o que contradiz o enunciado.

Nesse caso o aproveitamento é 2

1que é menor que .

3

2

Chamando o aproveitamento para a casa de congruência i de ai e o número de termosnessa casa de congruência de bi temos

nbbbbbababar kkkk 3

2)...(

3

2... 3212211 =++++≤+++=

)... pois ( 21 nbbb k =+++ portanto nrk 3

2≤

como rk é inteiro rk máximo é .3

2

n

Falta provar a existência de tal rk. Se faz assim:

Page 76: Eureka 2002

Sociedade Brasileira de Matemática

EUREKA! N°14, 2002

14

Divide-se n por 3 e arredonda-o para cima. Esse é o nosso k. .3

= n

k É obvio que

para n ≥ 2, .2

nk ≤ ( se n =1 k não existe)

Se n = 3k então nrk 3

2= e é máximo

Se n = 3k – 1n + 1 = 3k então teríamos, em relação ao caso acima a perda de um termo, a

escolher. O 3k rk máximo é

=−=−+= nn

nrk 3

2

3

1

3

21

3

)1(2

Se n = 3k – 2n + 2 = 3k do mesmo modo perdemos 2 em relação ao primeiro caso

=−=−+= nn

nrk 3

2

3

2

3

22

3

)2(2.

Portanto ao montar o caso n = 3k escolhe-se:

1, 2, 3,..., k, 2k, 2k + 1,..., 3k totalizando 2k termos n3

2

O caso n = 3k – 1 e n = 3k – 2 retira-se o 3k; e o 3k e o 3k – 1 respectivamente.

Resposta:

≤≤→=

3

2 é máximo 1

2

11 ter podemos não 1

nrn

kkn

k

Obs: x maior inteiro menor ou igual a x

xxx ≤<−1

x inteiro

x menor inteiro maior ou igual a x

1+<≤ xxx

x inteiro.

PROBLEMA 6:Dizemos que um inteiro n, n, > 1, é ensolarado se ele é divisível pela soma dos seus

fatores primos. Por exemplo, 90 é ensolarado pois 10532 e 53290 2 =++⋅⋅=divide 90. Mostre que existe um número ensolarado com pelo menos 200210 fatoresprimos distintos.

Page 77: Eureka 2002

Sociedade Brasileira de Matemática

EUREKA! N°14, 2002

15

SOLUÇÃO DE RAFAEL DAIGO HIRAMA (CAMPINAS – SP):Vamos ver casos pequenos: 2 ⋅ 3 ⋅ 5 = 30 e é ensolarado pois 2 + 3 + 5 = 10|30Olha só que interessante: se escolhemos alguns números primos e a soma delespuder ser escrita como um produto qualquer deles o produto de todos esses primosvezes a soma deles é um número ensolarado, aliás o mmc é ensolarado.Vamos ver até onde isso vai:2 + 3 + 5 + 7 = 17 droga! 17 é primo, vamos adaptar2 + 3 + 5 + 7 + 17 = 34 e 34 = 2 ⋅ 17mmc (34, 2 ⋅ 3 ⋅ 5 ⋅ 7 ⋅ 17) = 2 ⋅ 3 ⋅ 5 ⋅ 7 ⋅ 17 que é ensolarado2 + 3 + 5 + 7 + 11 = 28 e 28 = 22 ⋅ 7mmc ( 28, 2 ⋅ 3 ⋅ 5 ⋅ 7 ⋅ 11) = 22 ⋅ 3 ⋅ 5 ⋅ 7 ⋅ 11 que é ensolarado2 + 3 + 5 + 7 + 11 + 13 = 412 + 3 + 5 + 7 + 11 + 13 + 41 = 82 e 82 = 2 ⋅ 412 ⋅ 3 ⋅ 5 ⋅ 7 ⋅ 11 ⋅ 13 ⋅ 41 é ensolarado.

Prova geral: p1 + p2 + p3 + p4 + ... + pn = x, nnpppx ααα ...21

21= (ou seja, não tem fatores

primos além dos p1 a pn, mas iα pode ser 0)

mmc(p1p2p3...pn), x) = ....2121 yppp n

n =βββ Como yx | e y só tem fatores p1, p2, ..., pn,y é ensolarado.

Percebendo os meus testes podemos ver um modo de adaptar se tivermos a somadesses primos um número primo diferente dos anteriores. E se tivermos mais?

......... e ... 3214321321432121321+++

+++=+++<<<< nnnnnnnnnn ppppppppppppppp αααααααα

)1......(... 213212

113211121 +=++++ ++

+−

+++nnn

nnnnnn ppppppppppp αααααα

este número é com certeza menor que a soma inicial logo terá um limite para seusfatores primos.Fazendo:

......... 321321321321121+++

++++ =++++ nnnnnnnn pppppppppp ββββββ

)1......(... 32143213

121432122121 +=+++++ +++

+−

+++++nnn

nnnnnnn ppppppppppppp βββββββ

Calma, podemos evitar tudo isso se escolhermos os n primeiros primos:2 + 3 + 5 +...+ pn = x ≤ n ⋅ pn. Se tivesem dois primos pi e pj em x tal que i, j > nteremos pipj ≤ x ≤ n ⋅ pn.Mas perceba que pk > k para todo k ∈ (isso acontece porque a seqüência dos kpercorre todos os naturais enquanto a dos pk "pula" vários naturais).ContinuandoVamos provar que n ⋅ pn < pi ⋅ pj, não o contrário. nnji pppp ⋅>⋅

Page 78: Eureka 2002

Sociedade Brasileira de Matemática

EUREKA! N°14, 2002

16

nji pnpp ⋅>⋅∴Logo em x só pode haver um fator primo diferente dos 2, 3, 5,..., pn. Usaremos pi edo mesmo modo vemos que x não é divisível duas vezes por pi (é só fazer j = i)2 + 3 + 5 +...+pn = m ⋅ pi ≤ n ⋅ pn

i

n

p

pnm

⋅≤

Olhe só, como 1<i

n

p

p porque pn < pi porque n < i, m < n só que n < pn ⇒ m < pn

Como m e pn são inteiros m + 1 ≤ pn

Agora pronto:2 + 3 + 5 +...+ pn = m ⋅ pi

2 + 3 + 5 +...+ pn + pi = (m + 1) ⋅ pi

mas como m + 1 ≤ pn , m + 1 pode ser escrito como produto dos primos 2, 3, 5,..., pn,

ou seja inin pppp nααα ...32...532 21=+++++ .

Como inin pppp nααα ...32...532 21=+++++

mmc ininin pppppp nn ⋅⋅⋅=⋅⋅⋅⋅⋅ )1,max()1,max()1,max( ...32)...532,...32( 2121 αααααα aliás,

como m + 1 ≤ pn αn= 0 ou 1, ou seja max(αn , 1) = 1Como queremos 102002 primos distintos, se

primos primeiros 10 os

10

2002

2002...32 p+++ não for fatorável nos

primos 200210,...,3,2 p ele será da forma m ⋅ pi , i > 102002

.102002pm <

Com isso ,)1(...32 200210 ii pmpp ⋅+=++++ que é fatorável em ipp ,,...,5,3,2 200210,

já que .1 200210pm ≤+

Logo pelo menos um entre 2002102002

21

10...32

ααα p⋅⋅ ou ipp ⋅⋅⋅ 2002102002

21

10...32

βββ , com

,200210...32 p

ip +++ com iα e iβ suficientemente grandes são ensolarados, ou seja, há umnúmero ensolarado com 102002 ou 102002 + 1(ou ambos) fatores primos distintosAlias, esse método prova que para todo t inteiro positivo existe pelo menos umnúmero ensolarado com t fatores primos ou pelo menos um número ensolarado com t+ 1 fatores primos (ou ambos).

Obs.

≥≥

=xyy

yxxyx

se ,

se ,),max(

Page 79: Eureka 2002

Sociedade Brasileira de Matemática

EUREKA! N°14, 2002

17

XLIII OLIMPÍADA INTERNACIONAL DE MATEMÁTICAEnunciados e Resultado Brasileiro

A LXIII Olimpíada Internacional de Matemática foi realizada na cidade deGlasgow, Reino Unido no período de 18 a 31 de julho de 2002.

A equipe brasileira foi liderada pelos professores Edmilson Motta (SãoPaulo – SP) e Ralph Costa Teixeira (Niterói – RJ).

O Resultado da Equipe Brasileira

BRA 1 Alex Corrêa Abreu BronzeBRA 2 Larissa Cavalcante Queiroz de Lima PrataBRA 3 Guilherme Issao Camarinha Fujiwara BronzeBRA 4 Yuri Gomes Lima BronzeBRA 5 Davi Máximo Alexandrino Nogueira BronzeBRA 6 Thiago da Silva Sobral Bronze

PRIMEIRO DIADURAÇÃO: 4 horas e meia.

PROBLEMA 1Seja n um inteiro positivo. Seja T o conjunto de pontos (x; y) no plano onde x e y sãointeiros não negativos e x + y < n. Cada ponto de T é pintado de vermelho ou azul.Se um ponto (x; y) é vermelho, então todos os pontos (x'; y') com x' ≤ x e y' ≤ ytambém são. Um conjunto X é um conjunto de n pontos azuis com abcissas todasdistintas, e um conjunto Y é um conjunto de n pontos azuis com ordenadas todasdistintas. Prove que o número de conjuntos X é igual ao número de conjuntos Y.

PROBLEMA 2Seja BC um diâmetro do círculo Γ de centro O. Seja A um ponto de Γ tal que

.1200 00 <∠< AOB Seja D o ponto médio do arco AB que não contém C. A reta quepassa por O e é paralela a DA encontra a reta AC em J. A mediatriz de OA corta Γem E e F. Prove que J é o incentro do triângulo CEF.

PROBLEMA 3Encontre todos os pares de inteiros m, n ≥ 3 tais que há infinitos inteiros positivos a

para os quais 1

12 −+−+

aa

aan

m

é inteiro.

Page 80: Eureka 2002

Sociedade Brasileira de Matemática

EUREKA! N°14, 2002

18

SEGUNDO DIADURAÇÃO: 4 horas e meia.

PROBLEMA 4Seja n inteiro maior que 1. Os divisores positivos de n são d1, d2,…,dk , onde

nddd k =<<<= ...1 21

Seja D = d1d2 + d2d3 +…+dk – 1 dk.(a) Prove que D < n2.(b) Encontre todos os valores de n para os quais D é um divisor de n2.

PROBLEMA 5Encontre todas as funções f de em tais que

)()())()())(()(( yzxtfztxyftfyfzfxf ++−=++

para todo x, y, z, t ∈ .

PROBLEMA 6Sejam nΓΓΓ ,...,, 21 círculos de raio 1 no plano, onde n ≥ 3. Seus centros são O1,O2,…,On, respectivamente.Suponha que não exista reta que intercepte mais que dois dos círculos. Prove que

∑≤<≤

−≤nji ji

n

OO1

.4

)1(1 π

Page 81: Eureka 2002

Sociedade Brasileira de Matemática

EUREKA! N°14, 2002

19

MUROS, PRÉDIOS E ESCADASCícero de Oliveira Holmer, São Paulo – SP

♦ Nível Avançado.

Há um clássico problema de máximos e mínimos cujo enunciado envolve um prédio(tão alto quanto se queira) e um muro de altura h, à uma distância d deste prédio.Pretende-se colocar uma escada, apoiada no muro, a partir do solo e alcançando oprédio, conforme o esquema:

h

d

Pergunta-se então o seguinte: Qual é o comprimento mínimo da escada?Vamos montar um modelo, considerando um triângulo retângulo ABC e umretângulo APQR inscrito neste triângulo:

B P

Q

A

R

C

θ

Sendo ,aPQ = bQR = e ,)ˆ( θ=CBAm temos:

.cosθθ

b

sen

aQCBQBC +=+= Assim, )(θfBC = e

θθ

θθθ

22 cos

cos.)('

senb

sen

af

⋅+−=

Para termos BC mínimo, é preciso que ,0)(' =θf isto é, 0cos

cos22

=⋅+⋅−θθ

θθ senb

sen

a

cos

cos

cos3

3

3

22 b

atg

b

asen

sen

asenb =⇔=⇔⋅=⋅⇔ θθθ

θθ

θθ

(I).

Pelo teorema de Pitágoras, =+++=⇔+= 222222 )()( RCARPABPBCACBABC

Page 82: Eureka 2002

Sociedade Brasileira de Matemática

EUREKA! N°14, 2002

20

.)( 2

2

θθ

tgbabtg

a ⋅++

+= Como BC deve ser mínimo, de (I), temos:

=

⋅++

+⋅=

⋅++

+=2

3 22

3 2

2

3

2

3

2 baabbab

abab

b

a

aBC

.33

22

33 23 2

33 23 223 23 22

3 23 2223 23 2

+=⇔

+=+⋅⋅⋅+⋅⋅⋅+=

=⋅⋅+⋅⋅⋅+++⋅⋅⋅+⋅⋅=

baBCbabbabbaaa

babbaaabbabbaa

Portanto, o comprimento mínimo da escada deve ser .3

3 23 2

+ dh

Vamos agora considerar uma situação com valores numéricos (talvez você possaaproveitar melhor o que vem a seguir tendo em mãos papel, caneta e, se possível,uma boa calculadora).A partir de um triângulo retângulo bem conhecido, de lados 3, 4 e 5, e outrotriângulo semelhante, por exemplo o de lados 9, 12 e 15, podemos montar a figura:

B P

Q

A

C

3 m 5 m

4 m 8 m

10 m

9 m

Formulamos, então, o seguinte problema: Se o muro tem 3 metros de altura, adistância do muro ao prédio é igual a 8 metros e a escada tem 15 metros decomprimento, poderíamos afirmar que a distância do pé da escada ao muro é igual a4 metros?

Page 83: Eureka 2002

Sociedade Brasileira de Matemática

EUREKA! N°14, 2002

21

Vejamos:

O menor comprimento possível da escada é de ( )333 23 2 4983 +=+

metros.

Pode-se verificar que ( ) ,154933 <+ e isto quer dizer que há duas maneiras

distintas de posicionarmos a escada e, portanto, existem duas distâncias possíveis dopé da escada ao muro. Vamos então, novamente, montar um modelo:

B P

Q

A

C

3

X 8

15

Uma solução possível, claro, é x = 4 metros. Busquemos a outra solução:

,~ ABCPBQ ∆∆ logo 8

15

15

8

+⋅=⇔+=⇔=

x

xBQ

x

BQ

x

BC

BA

BQ

BP

No BPQ∆ temos

05761441521698

153 234

22222 =+⋅+⋅−⋅+⇔−

+⋅=⇔−= xxxx

x

xxBQx

Aplicando-se o algoritmo de Briot-Ruffini, obtemos:

1 1

16 20

–152 –72

144 –144

576 0

4

Assim, a outra solução é raiz de .01447220 23 =−−+ xxx

É possível mostrar que essa equação tem duas raízes reais negativas e uma raiz realpositiva, que é aproximadamente 4,3274534… e pode ser escrita como

Page 84: Eureka 2002

Sociedade Brasileira de Matemática

EUREKA! N°14, 2002

22

⋅ 53

154154

1567arccos

cos1543

4 (ver por exemplo [3] para um método de

resolução de equações do terceiro e quarto graus).

Assim, as possíveis distâncias do pé da escada ao muro, são de 4 metros e de

...3274534,453

154154

1567arccos

cos.1543

4 =

−⋅ metros.

Referências Bibliográficas

[1] Piskunov N., Cálculo Diferencial e Integral, Tomos I e II, Ed. Mir 1977[2] Demidovitch B., Problemas e Exercícios de Análise Matemática, Ed. Mir 1978[3] Moreira, C.G., Uma solução das equações do terceiro e quarto graus, RPM 25,pp. 23-28.

Page 85: Eureka 2002

Sociedade Brasileira de Matemática

EUREKA! N°14, 2002

23

INTEIROS DE GAUSS E INTEIROS DE EISENSTEINGuilherme Fujiwara, São Paulo – SP

♦ Nível Avançado.

Vamos abordar nesse artigo a aritmética de dois conjuntos de inteiros algébricos: osInteiros de Gauss e os Inteiros de Eisenstein.

1. INTEIROS DE GAUSSDefinimos o conjunto [i] dos inteiros de Gauss como [i] = a + bi | a, b ∈ ,onde (i2 = –1). A seguir veremos as duas coisas mais importantes de sua aritmética, oteorema da fatoração única e os primos.

1.1 NormaVamos definir uma função N: [i] → + chamada norma, tal que ∀ z ∈ [i], N(z) = z

⋅ z sendo z o conjugado complexo de z. Observe que como ,baab ⋅= então

)()()( abNababbababbaabNaN =⋅=⋅⋅⋅=⋅=⋅ , ou seja, a norma émultiplicativa.

1.2. UnidadesAs unidades em [i], analogamente a , são todos os elementos z ∈ [i] quepossuem inverso, ou seja, que ][' iz ∈∃ tal que .1'=⋅ zz Segue que se z = a + bi é

uma unidade, então ⇔=+⇔=⇒⋅=⋅= 11)()'()()'(1 22 bazNzNzNzzN

0,1 =±=⇔ ba ou ,0=a ib ±=±=⇔±= ou 11 , e como esses quatro teminverso, todas as unidades são 1± e .i± Observe então que ][ix ∈ é unidade

.1)( =⇔ xN

1.3. DivisibilidadeDizemos que para a, b ∈ [i], a|b (lê-se a divide b) se ][ic ∈∃ tal que b = ac.

1.4. Divisão EuclidianaVamos ver como funciona a divisão euclidiana. A divisão Euclidiana é a existênciade 0],[,],[, ≠∈∀∈ bibairq tal que a = bq + r, sendo ).()(0 bNrN <≤ Parademonstrá-la, basta dividir:

, , wizbyixa +=+= onde x, y, z, w ∈ .

Page 86: Eureka 2002

Sociedade Brasileira de Matemática

EUREKA! N°14, 2002

24

iwz

xwyz

wz

ywxz

wz

ywiyzixwixz

wiz

wiz

wiz

yix

wiz

yix

b

a222222

2

+−+

++=

+−+−⇔

−−⋅

++=

++=

Tomamos m e n como os inteiros mais próximos de 22 wz

ywxz

++

e 22 wz

xwyz

+−

,

respectivamente. Note que 2

1,

2222≤

+−−

++−

wz

xwyzn

wz

ywxzm Se q = (m + ni), então:

+−+−

+−=

−=−= in

wz

xwyzm

wz

xwyzbq

b

abbqar

2222

)(2

)(

2

1

2

1)()(

22

bNbN

bNrN <=

+

≤⇒

1.5. Lema de EuclidesA partir da divisão euclidiana podemos demonstrar o lema de Euclides, ou seja, se pé um primo de Gauss (ou seja, não pode ser escrito como o produto de dois inteirosde Gauss cujas normas são maiores que 1), então sendo a, b ∈ [i], p|ab ⇒ p|a oup|b. Para demonstrá-lo, vamos fazer sucessivas divisões euclidianas, sendo a0 = a ea1 = p. Seja ak + 2 o resto da divisão euclidiana de ak por ak+1. Temos então asdivisões:

11

112

4332

3221

2110

+−

−−−

+=+=

+=+=+=

nnnn

nnnn

aaqa

aaqa

aaqa

aaqa

aaqa

Observe que como )()(0 1 kkk aNaNa <⇒≠ + , podemos tomar n tal que N(an +1) =0, ou seja, an + 1 = 0.Logo an|an – 1. Observe que 1| +kn aa e .|| 1−⇒ knkn aaaa Logo nn aa | e ,| 1−nn aa

então indutivamente, ,0 ,| nkkaa kn ≤≤∀ particularmente aaan =0| e

.| 1 paan = Tomando as j + 1 primeiras equações e realizando substituições

adequadas, temos que aj = xj a1 + yj a0 = xj p +yj a; particularmente .aypxa nnn +=

Page 87: Eureka 2002

Sociedade Brasileira de Matemática

EUREKA! N°14, 2002

25

Voltando ao lema, veja que se p|a então o lema está certo. Se p não divide a, então,como ,| pan an|a e an = xnp + yna, então an ∈ 1; – 1; i; – i e temos:

,|)(1 bpabybpxabaypxa nnnnnn ⇒+=⇔+= − pois ,| abp o que conclui ademonstração.

1.6. Fatoração únicaA fatoração única é uma das propriedades mais usadas em problemas envolvendonúmeros inteiros. Vamos prová-la para os inteiros de Gauss. Primeiramenteprovaremos que todo inteiro z de Gauss com norma maior que 1 pode ser escritocomo o produto de um ou mais primos de Gauss. Se N(z) = 2, como 2 é primo e anorma é multiplicativa, então z é primo, portanto está provado. Considere N(z) > 2.Se z é primo a fatoração é imediata. Se z não é primo, então z = a ⋅ b ⇒ N(z) = N(a) ⋅N(b), onde N(a), N(b) > 1, portanto N(a), N(b) < N(z). Podemos supor, por indução,que se N(x) < N(z), então x é fatorável. Logo a e b são fatoráveis, e portanto z.Para provar que esta fatoração é unica, basta considerar as duas fatorações p1p2…pn eq1q2…qm . Suponha, por indução, que p1p2…pn = εq1q2…qm, sendo ε uma unidade,implica que a seqüência (pi) é uma permutação (a menos que sejam multiplicaçõespor unidades) da (qi). Se maxn; m = 1, então o resultado é imediato. Supondo queele vale se maxn'; m'< maxn; m, pelo lema de Euclides, vemos que para algum i,pn|qi. Sem perda de generalidade, i = m. Como pn e qm são primos, então qm = ε'pn,onde ε' é uma unidade. Logo p1p2…pn = εq1q2…qm

⇔ p1p2…pn – 1 = ....' 121 −mqqqεε Por indução, p1, p2,...,pn-1 é uma permutação (amenos que sejam multiplicações por unidades) de q1, q2, …, qm, portanto a fatoraçãoúnica está provada.

1.7. Números primosVamos agora ver quem são os números primos em Z[i]. Observe que se N(π) é primoem , então π é um primo de Gauss (pois se π fatora então N(π) fatora).

Observe que todo primo π divide N(π), portanto ele deve dividir ao menos um fatorprimo em de N(π). Se π dividir ao menos dois números distintos (absolutamente) x

e y primos em , como sempre é possível tomar a, b ∈ tal que ax + by = 1,teríamos π|1, um absurdo. Logo todo primo de Gauss divide exatamente um primointeiro positivo (e seu oposto negativo) em . Seja esse primo inteiro positivo p.Temos três casos:Se p é par, então p = 2. Sendo π = a + bi, então a2 + b2 = 2 ⇔ π = ± 1 ± i , e obtemosos quatro primos 1 + i, 1 – i, –1 + i e –1 –i. Observe que eles são dois a dois um amultiplicação por uma unidade do outro.

Page 88: Eureka 2002

Sociedade Brasileira de Matemática

EUREKA! N°14, 2002

26

Se p ≡ 3 (mód. 4), como x ∈ ⇒ x2 ≡ 0 ou 1 (mód. 4), então, se existisse π = c +di, c, d ∈ Z, 1 < N(π) < p2 tal que p = πϕ, é facil ver que, como p é um primo inteiroϕ = c – di , logo p = c2 + d2 ≡ 0, 1 ou 2 (mod.4), absurdo, pois p = 4k + 3. Logo p éum primo de Gauss.Se p ≡ 1 (mód. 4), então, sendo x = 1 × 2 ×…× ( p – 1)/2, então:

2

)1(...21

2

)1(...212 −××××−×××≡ pp

x

)1(1)1()2(...2

)1(

2

)1(...21 −×≡−×−××+×−×××≡ ppp

pp

).(1 pmód−≡

Logo ).)((1| 2 ixixxp −+=+ Como π é um primo de Gauss que divide p, então π∈ Z, π|x + i ou π|x – i ⇒ π|1, absurdo. Portanto π ∉ [i] tal que p = πϕ. Seja π = a +bi e ϕ = c + di, a, b, c, d ∈ Z. Como p é primo em z, então mdc(a; b) = mdc(c;d) = 1.Temos p = (a + bi)(c + di) = ac – bd + (bc + ad)i. Como p ∈ Z, então bc = –ad ⇒ (a

= c e b = – d) ou (a = –c e b = d) ⇔ ϕ = ± .π Como p > 0, então ,)( pN =⇒= ππϕlogo π é primo (e π e seu conjugado são únicos primos de Gauss que dividem p).Portanto vimos que os números primos em [i] são:

(1) O primo 1 + i e seus produtos pelas unidades.(2) Os primos p em tal que p ≡ 3 (mod. 4) e seus produtos pelas unidades.

(3) Para cada primo p em + tal que p ≡ 1 (mod. 4), os primos a + bi, a – bi e seusprodutos pelas unidades, sendo a2 + b2 = p.

1.8. Ternas pitágoricasAgora que já vimos a aritmética básica dos inteiros de Gauss, vamos começar comum resultado simples e interessante. Vamos achar as soluções da equação a2 + b2 =c2, sendo a, b, c ∈ . Seja m = mdc(a; b), a' = a/m e b' = b/m. Temos então m2 (a2 +

b2) = c2 ⇒ m|c. Seja então c' = c/m, temos a'2 + b'2 = c'2, mdc(a';b';c') = 1.Note que a'2 + b'2 = c'2 ⇔ (a'+ b'i)(a'– b'i) = c'2. Observe que se d = mdc(a' + b'i; a'–b'i), então d|2a' e d|2b' ⇒ d|2. Se d não divide 1, então d|a'2 + b'2 ⇒ a' e b' sãoímpares, o que é um absurdo, basta ver congruência módulo 4. Portanto d|1 ⇔ a' +b'i e a' + b'i são primos entre si, logo ambos são quadrados perfeitos. Observetambém que quaisquer a'e b' primos entre si tais que a' + b'i e a'– b'i são quadradosperfeitos são soluções da equação. Portanto a'e b' formam uma solução se e somentese existem x, y, z, w ∈ tal que:

Page 89: Eureka 2002

Sociedade Brasileira de Matemática

EUREKA! N°14, 2002

27

xyb

yxayixiba

yixiba

yixiba

wiziba

yixiba

2'

')(''

)(''

)(''

)(''

)('' 222

2

2

2

2

=−=

⇔+=+⇔−=−

+=+⇔

+=−

+=+

Veja então que a' e b' são primos entre si se e só se x e y são primos entre si. Logo assoluções são a = (x2 – y2) ⋅ d, b = 2xy ⋅ d, ou vice-versa, e conseqüentemente c = (x2 +y2) ⋅ d, para x, y, d ∈ , sendo x e y primos entre si.

1.9. O número de representações de um inteiro como a soma de dois quadradosProvaremos agora o seguinte

Teorema. Dado n ∈ , o número de pares a, b ∈ tais que n = a2 + b2 é igual aquatro vezes a diferença entre o número de divisores da forma 4k + 1 de n e onúmero de divisores da forma 4k + 3 de n.Podemos expressar n da forma:

mmmmi

kk qqqqppn ββββααα )(...)(...2 11

11 ×=Sendo pi primos de Gauss inteiros (da forma 4k + 3) e os pares de conjugados qi ≠

iq primos de Gauss (N(qi) da forma 4k + 1) e esses primos diferem dois a dois por

mais do que uma multiplicação por uma unidade.Sendo n = a2 + b2 = (a + bi)(a – bi), então, pelo teorema da fatoração única e amultiplicidade do conjugado, temos:

,)(...)(...)1( 111

1

1122

1mmm

k

mmk qqqqppibia γβγγβγαα

αε −−×+=+ onde ii βγ ≤≤0 e ε é

uma unidade.Portanto o número de representações de m como a soma de dois quadrados será 0 sealgum iα for ímpar e será 4(β1 + 1)…(βm + 1) se todos αi forem pares, sendo o fator4 pois há 4 escolhas possíveis para a unidade.Observe agora que a fatoração de n em primos inteiros será:

mkmk qNqNppn ββααα )(...)(...2 11

11 ×=Onde pi serão primos da forma 4k + 3 e N(qi) serão primos da forma 4k + 1. Observeagora que um divisor ímpar de n será da forma:

,)(...)(... 1111

mk bm

bak

a qNqNppd ×= onde mmkk bbaa ββαα ≤≤≤≤ ,...,,,..., 1111

Note que se a1 + … + ak é par, então d é da forma 4k + 1, se for ímpar é da forma4k + 3. Portanto, conseguimos verificar que se algum αi for ímpar, o número de d’sda forma 4k + 3 será igual ao número de d’s da forma 4k + 1, e se todos os αi forempares, a diferença entre esses números será (β1 + 1)…(βm + 1), o que termina ademonstração do teorema.

Page 90: Eureka 2002

Sociedade Brasileira de Matemática

EUREKA! N°14, 2002

28

1.10. Problemas

Problema 1. Determine todos os pares x, y ∈ tal que y3 = x2 + 1

Problema 2. Sejam x, y, z ∈ tais que xy = z2 + 1. Prove que existem inteiros a, b, c,d tais que x = a2 + b2, y = c2 + d2 e z = ac + bd.

Problema 3. Prove que existem duas seqüências inteiras (an) e (bn) infinitas e

estritamente crescentes tais que ak(ak + 1) divide 12 +kb para todo natural k.

2. INTEIROS DE EISENSTEINVamos agora ver os Inteiros de Eisenstein. Definimos o conjunto [ω] dos inteiros

de Eisenstein como [ω] = a + bω| a, b ∈ , sendo ,2

3

2

1 i+−=ω donde ω2 + ω

+ 1 = 0. Para ζ = a + bω ∈ [ω] definiremos a norma como N(ζ) = ζζ ⋅ = a2 – ab +b2. Observe que essa norma segue as mesmas propriedades da norma dos inteiros deGauss (é inteira não negativa e multiplicativa).

2.1. UnidadesAs unidades em [ω] são definidas como os seus elementos que possuem inverso,ou seja u, tal que ∃ u – 1 tal que u ⋅ u – 1 = 1 ⇒ N(u) = 1 ⇔ u = ±1, ±ω, ±(1 + ω), everificamos que esses quatro números tem inverso, portanto u é unidade se, e só seN(u) = 1.

Obs. Note que . )1( 2ωω ±=+±

2.2. Divisão EuclidianaPara provar a existência de divisão Euclidiana entre a, b ∈ [ω], b ≠ 0. Sejam α e βtais que:

βωα +=b

a

Tomando q = c + dω, tais que c e d são respectivamente os inteiros mais próximos deα e β. Portanto:

)(4

1

4

1

4

1)())())(())(((

)()(

22 bNbNddccbN

rNqbbqar

<

++≤−+−−−−=

=⇒−+=−=

ββαα

βωα

Portanto existe a divisão Euclidiana.

Page 91: Eureka 2002

Sociedade Brasileira de Matemática

EUREKA! N°14, 2002

29

2.3. Teorema da fatoração ÚnicaNote que, para os inteiros de Gauss, provamos o lema de Euclides e a fatoraçãoúnica, usando somente o fato de que existe divisão Euclidiana, portanto, seguindo osmesmos passos para provar o lema de Euclides e a fatoração única, provaremos afatoração única para os inteiros de Eisenstein.

2.4. PrimosTudo é muito parecido com os inteiros de Gauss: N(π) é primo em ⇒ π é primo

em [ω]; todo primo π em [ω] divide exatamente um primo inteiro positivo. Ademonstração desses dois fatos é exatamente igual que foi dada na seção de inteirosde Gauss. Seja p o inteiro positivo primo que o primo, π em Z[ω] divide. Temos trêscasos:• Se p é da forma 3k, então p = 3, e obtemos π = ±(1 – ω) ou ±(2 + ω).• Se p é da forma 3k + 2, como a2 – ab + b2 só é da forma 3k ou 3k + 1 (verifiquevocê mesmo), então p é um primo de em Z[ω] tal que N(π) = p2.• Se p é da forma 3k + 1, pela lei da reprocidade quadrática*:

13

1)1(3

32

13

2

1

=

−⇒=−=

−−⋅

pp

pp

Portanto existe x inteiro tal que )2)(2(223)1(| 2222 ωω −−=+−=+− xxxxxp , e

como p não divide 2, então p não é primo em Z[ω] e existem π e ψ tal que .p=πψComo p é um primo inteiro, então πψ = , logo π e ψπ = são primos em [ω] e

p=⋅ππ .

Portanto os primos em [ω] são:(1) O primo 1 – ω e suas multiplicações por unidades.(2) Os primos inteiros da forma 3k + 2 e seus produtos pelas unidades, que também

são primos em [ω].

(3) Para todo primo inteiro p da forma 3k + 1, os primos π e π tal que p=ππ e seus

produtos pelas unidades são primos em [ω].

*A lei de reciprocidade quadrática de Gauss diz o seguinte: dados a ∈ e p ∈ primo que não divide

a, definimos

=

contrário. caso ,0

.mod. quadrático resíduo é se 1, pa

p

a Para p, q ∈ primos ímpares com p > 0

vale sempre .)1( 2

1

2

1

−=

qp

p

q

q

p

Page 92: Eureka 2002

Sociedade Brasileira de Matemática

EUREKA! N°14, 2002

30

2.5. Exemplo

Ache todos os a, b, c ∈ *+ lados de um triângulo com um ângulo de 60o.

Vamos supor, sem perda de generalidade, que o ângulo de 60o é entre os lados demedidas a e b. Pela lei dos co-senos, temos:

))(()60cos(2 22222 ωω babaabbaabbac −+=−+=°−+=

Observe a semelhança deste problema com o das ternas pitagóricas.

Seja m = mdc(a; b), a = a'm, b = b'm. Segue que cmcm ⇔22 , e teremos c = c'm.

Logo babacabbac '''' 222222 −+=⇔−+= ', e temos mdc(a'; b'; c') = 1. Seja d talque ω'' bad + e .'' ωbad − Segue que '2ad e '2bd , logo 2d . Se d não divide 1,

então '2''''22 22 abbaad ⇒+−⇒ e '2 b , absurdo, logo 1d , e portanto:

2

222222

2'

')2()(''

yxyb

yxayxyyxyxba

−=

−=⇔−+−=+=+ ωωω

Portanto as soluções são myxybmyxa )2(,)( 222 −=−= e myxyxc )( 22 +−= ,

para todo x, y, m ∈ + com x > y, e as permutações de a, b e c.

Outro bom exemplo de aplicação dos inteiros de Eisenstein é o problema 6 da IMOde 2001:

Sejam a, b, c, d inteiros com a > b > c > d > 0. Considere que

))(( cadbcadbbdac +−+−++=+

Prove que ab + cd não é primo.

Primeiramente vamos mostrar por que usar inteiros de Eisenstein:

2222

2222

))((

cacadbdb

caccdbcacaadabcdaddbdbcabbdbbdac

cadbcadbbdac

+−=++⇔

⇔−+−−+−+++−+++−+=+⇔

⇔+−+−++=+

Aí vemos por que usar inteiros de Eisenstein.

( )( ) ( )( )22 ωωωω cacadbdb ++=−−

Observe que como a, b, c > 1, então mdc(b; d) > 1 ⇒ ab + cd não é primo, logopodemos supor que b e d são primos entre si. Analogamente supomos que mdc(a; c)= mdc(a; d) = mdc(b; c) =1.

Page 93: Eureka 2002

Sociedade Brasileira de Matemática

EUREKA! N°14, 2002

31

Seja π um primo em ][ω tal que ωπ db − . Vamos provar que π não divide

πω ⇔+ ca não divide ,2ωca + e segue que ωω cadb +− . Suponha então que

ωπ ca + . Veja que ),)(()( ωωπ cadbN +− e temos

ωωω )())(( adcdbccdabcadb −+++=+−

Como ,)( ∈πN e ))(()( ωωπ cadbN +− , então cdabN +)(π e, supondo ab +

cd primo, teríamos )(πNcdab =+ . Mas nesse caso segue que

,'2 kkk cadbdb πεωπεωεπω =+⇒⋅=−⇒=− sendo ε e 'ε unidades.

Se ,/' εεµ = então ).(2 ωµωω cadbddb +=−=++ Considerando o fato demdc(b; d) = mdc(a; c) = mdc(b; c) = 1 e que a > b > c > d > 0, temos que isto é umabsurdo (a verificação fica para o leitor, basta considerar as 6 possibilidades para

)µ . Logo ωω cadb +− , e analogamente .ωω dbca −+ Portanto

)( ωυω cadb +=− , onde υ é uma unidade. Novamente, basta verificar todas aspossibilidades para υ e verificar que isto é um absurdo. Portanto ab + cd não éprimo.

2.6. ProblemasDeixamos aqui mais alguns problemas para o leitor:

Problema 1.(a) Prove que, para cada inteiro n, o número de soluções inteiras de x2 – xy + y2 = n é

finito e divisível por 6.(b) Determine todas as soluções inteiras de x2 – xy + y2 = 727.

Problema 2.Mostre que a equação diofantina x3 + y3 + z3 = 0 só tem soluções triviais, ou seja, taisque xyz = 0.

Problema 3.Prove que se n é um inteiro positivo tal que a equação x3 – 3xy2 + y3 = n tem soluçõesem inteiros (x; y), então ela tem pelo menos três soluções.

Page 94: Eureka 2002

Sociedade Brasileira de Matemática

EUREKA! N°14, 2002

32

SEQÜÊNCIAS ARITMÉTICO-GEOMÉTRICASJosé Paulo Carneiro & Carlos Gustavo Moreira

♦ Nível Intermediário.

A) Seqüências Aritmético-Geométricas ( à la Zé Paulo)Uma progressão aritmética (PA) é uma seqüência tal que cada termo é igual aoanterior adicionado de uma constante (a razão), isto é, seu termo geral (an) satisfaz àrelação de recorrência .1 raa nn += − Daí se deduz, como é conhecido, que

rnaan )1(1 −+= .Analogamente, uma progressão geométrica (PG) é uma seqüência tal que cada termoé igual ao anterior multiplicado por uma constante (a razão), isto é, seu termo geral(an) satisfaz à relação de recorrência qan = .1−na Daí se deduz, como é conhecido,

que 11

−= nn qaa .

Existe um outro tipo de seqüência que aparece freqüentemente, que é uma espécie demistura de uma PA e uma PG. É aquela cujo termo geral (an) satisfaz a relação derecorrência qan = ,1 ran +− e que vamos chamar de seqüência aritmético-geométrica (à la Zé Paulo) (SAG), de "razão geométrica" q e "razão aritmética" r.(só vamos considerar os casos em que r ≠ 0 e q ≠ 1, para não recair numa PA ounuma PG.)Observemos que, uma vez conhecido o primeiro termo a1 e a relação de recorrência

qan = ,1 ran +− conhecem-se sucessivamente a2, a3, etc., e, em princípio, todos osan. Mas permanece o interesse em determinar uma expressão explícita para o termogeral de uma SAG em função de n, uma vez conhecidos r, q e a1.Um exemplo de SAG aparece na solução do célebre problema da Torre de Hanói(ver [2]), onde são dados três pinos A, B e C, e se quer determinar o número mínimode movimentos necessários para se mover do pino A para um dos dois outros pinosuma pilha de n discos de tamanhos desiguais, de modo que nunca um disco maiorfique em cima de um disco menor. Se an for o número procurado, podemosraciocinar que, inicialmente, vão ser necessários an – 1 movimentos para mover os n –1 discos superiores para o pino B, digamos; em seguida, um movimento para movero maior de todos os discos para o pino C; e finalmente, mais an – 1 movimentos paramover a pilha restante para C e completar a operação. Portanto, an satisfaz à relaçãode recorrência an = an – 1 + 1 + an – 1 = 2an – 1 + 1, além da condição inicial a1 = 1. Apartir daí, podem ser determinados sucessivamente:a2 = 2 × 1 + 1 = 3, a3 = 2 × 3 + 1 = 7, e assim por diante.Uma seqüência constante pode ser considerada uma PA de razão 0 ou uma PG derazão 1.

Page 95: Eureka 2002

Sociedade Brasileira de Matemática

EUREKA! N°14, 2002

33

Uma pergunta interessante é: uma SAG pode ser uma seqüência constante? É claroque, se c for este valor constante, isto ocorrerá se e só se: c = qc + r, ou seja:

.1 q

rc

−=

Vamos agora determinar uma expressão explícita para o termo geral de uma SAGem função de n. Para isto, consideremos duas SAGs quaisquer, de termos gerais,respectivamente, an e bn, que tenham a mesma razão aritmética r e a mesma razãogeométrica q, e consideremos a sua diferença dn = an – bn . Como an = q an – 1 + r ebn = q bn – 1 + r, então dn = an – bn = q (an – 1 – bn – 1) = q dn – 1. Mas isto mostra que(dn) é uma PG de razão q e, portanto: dn = d1 q

n – 1, ou seja: an – bn = (a1 – b1)qn – 1 , ou

ainda: an = bn + (a1 – b1)qn – 1. Como an e bn eram SAGs quaisquer, esta fórmula

indica como qualquer SAG pode ser obtida de outra que tenha a mesma razãoaritmética e a mesma razão geométrica q, podemos tomar bn constante e igual a

,1 q

r

− obtendo: 1

1 11−

−+−

= nn q

q

ra

q

ra ,

que é a expressão que se procurava para an.Por exemplo, no caso da Torre de Hanói, an = 2an – 1 + 1, com a1 = 1. Portanto:

.12221

11

21

1 1 −=

−−+

−− nn

Vale a pena acrescentar que, se a SAG tiver uma infinidade de termos e |q| < 1, então

qn – 1 tende a zero, quando n tende a infinito. Portanto, an tende a .1 q

r

Por exemplo: a seqüência infinita definida por: ,5

2 1−+= n

n

aa com a1 = 1 tem

termos: ;...,125

63;

25

13;

5

3;1 ou, em decimais: 1,000; 0,600; 0,520; 0,504;….

Como ,5

2

5

11 += −nn aa esta seqüência é uma SAG com

5

2=r e ,5

1=q de modo

que seu termo geral é ,5

1

2

1

2

11−+=

nna e o seu limite é .5,02

1 =

B) Seqüências Aritmético-Geométricas ( à la Gugu )Na seção anterior definimos seqüências aritmético-geométricas (à la Zé Paulo)generalizando as recorrências de PA's e PG's. Vamos adotar agora um ponto de vistaalternativo, generalizando a fórmula do termo geral de PA's e PG's.

Page 96: Eureka 2002

Sociedade Brasileira de Matemática

EUREKA! N°14, 2002

34

Lembremos que em uma PA de termo inicial a0 = a e razão r temos an = a + nr,∈∀ n e numa PG de termo inicial a0 = a e razão q, temos an = aqn. Fazemos,

inspirados nessas fórmulas, a seguinte definição:

Definição: Uma progressão Aritmético-Geométrica (PAG) (à la Gugu) é umaseqüência (an) cujo termo geral satisfaz a fórmula an = (a + nr)qn, ∈∀ n .Note que se r = 0 nossa PAG é uma PG, e se q = 1 nossa PAG é uma PA.É interessante obter uma fórmula para a soma dos n primeiros termos de uma PAG

(sn = ∑−

=

1

0

n

jna ), a qual generalizaria as fórmulas para a soma dos n primeiros termos de

PG's (a princípio generalizaria também de PA's, mas suporemos q ≠ 1. Se fizermos qtender a 1 a fórmula tenderá à fórmula da soma dos n primeiros termos de uma PA).

Temos sn = ∑−

=

+1

0

.)(n

j

jqjra Então ∑ ∑−

=

=

− =++=+=− 1

1

2

0

1 ))1(()(n

j

njn qraqjra

q

as

∑ ∑−

=

=

−−

−+−+−

−=++=2

0

2

0

11

))1((1

)1()(

n nn

n

n

qrnasq

qrqrarq

e, portanto,

( ) ( )=

+−−

−−+−

=⇒−

−−−−+=

− −−

1)

1)1((

11

1)1(

1 11

1

q

r

q

aq

q

rrna

q

qs

q

qr

q

aqrna

q

qs n

n

nn

n

.)1(

)(

11 21

−+−+

−+−

= −

q

aqarq

q

qrnra

q

q n

Note que se r = 0 a expressão acima se reduz a ,1

)1(

)1(

)1(

1 2 −−=

−−⋅+

− q

qa

q

qa

q

aq nn

que é a

fórmula da soma dos n primeiros termos de uma PG.Notemos finalmente que SAG's e PAG's satisfazem recorrências lineareshomogêneas (ver [1]): Para SAG's, temos ,112 nnnn qaarqaa −==− +++ logo

nnn qaaqa −+= ++ 12 )1( , e para PAG's, notamos que se nnn qab /= , temos bn =

a + nr, e =⇒−=⇒−=⇒−==− ++−

++

+++++ 211

22

12112

22 nn

nnn

nn

nnnnnnn aq

a

q

a

q

abbbbbrbb

.2 21 nn aqqa −= + Os polinômios característicos dessas recorrências são

respectivamente )1)(()1(2 −−=++− xqxqxqx e x2 – 2qx + q2 = (x – q)2.

Referência:[1] Héctor Soza Pollman, Equações de Recorrência, Eureka! No. 9, pp. 33-40.[2] Carlos Yuzo Shine, A torre de Hanói, Eureka! No. 11, pp. 17-23.

Page 97: Eureka 2002

Sociedade Brasileira de Matemática

EUREKA! N°14, 2002

35

O PRINCÍPIO DA INVARIÂNCIAMarcelo Rufino de Oliveira, Belém – PA

Artigo baseado em aula ministrada no II Teorema, Fortaleza – CE

♦ Nível Intermediário.

Uma das principais estratégias para resolução de problemas de olimpíadas é aprocura por invariantes. O fundamento do Princípio da Invariância é simples: buscapelo que se mantém constante quando uma operação permitida é realizada. Entre asprincipais formas de invariantes destacam-se três, que serão apresentadas a seguiratravés de exemplos resolvidos.

1. Expressões ou Valores Numéricos Invariantes

Exemplo 1.1: Começando com o conjunto 3, 4, 12, é permitido apagar dois númerosa e b e escrever em seus lugares 0,6.a – 0,8.b e 0,8.a + 0,6.b. É possível chegar aoconjunto 4, 6, 12?

Resolução:Repare que (0,6.a – 0,8.b)2 + (0,8.a + 0,6.b)2 = a2 + b2, implicando que a soma dosquadrados dos números dos conjuntos obtidos é invariante. Como 32 + 42 + 122 = 132

e 42 + 62 + 122 = 142 então não é possível chegar ao conjunto 4, 6, 12.

Exemplo 1.2: (Maio-99) Em cada um dos 10 degraus de uma escada existe uma rã.Cada rã pode, de um pulo, colocar-se em outro degrau, mas quando uma rã faz isso,ao mesmo tempo, uma outra rã pulará a mesma quantidade de degraus em sentidocontrário: uma sobe e outra desce. Conseguirão as rãs colocar-se todas juntas nummesmo degrau?

Resolução:Numeremos os degraus de 1 a 10 e associemos a cada rã o número do degrau queocupam. O somatório inicial destes valores é S = 1 + 2 + … + 10 = 55.Perceba agora que este somatório é invariante, pois quando uma rã sobe uma certaquantidade x de degraus, temos outra rã que desce x, fazendo com que a soma dasnumerações destas duas rãs não se altere. Caso todas as rãs ocupem um mesmodegrau (digamos y), então todas as suas numerações são iguais a deste degrau, ouseja, teremos 10y = 55, que não possui solução inteira. Deste modo, é impossível quetodas as rãs ocupem um mesmo degrau.

Page 98: Eureka 2002

Sociedade Brasileira de Matemática

EUREKA! N°14, 2002

36

Exemplo 1.3: As seguintes operações são permitidas com a equação quadráticaax2 + bx + c: a) trocar a e c; b) trocar x por x + t, onde t é um número real. Repetindoestas transformações é possível transformar x2 – x – 2 em x2 – x – 1?

Resolução:Mostraremos que é invariante o valor do discriminante de todas as equações obtidaspela aplicação das operações permitidas.Inicialmente temos ∆0 = b2 – 4ac.Aplicando a primeira operação: ax2 + bx + c → cx2 + bx + a (1)Para esta equação temos ∆1 = b2 – 4ca = ∆0

Aplicando a segunda operação:ax2 + bx + c → a(x + t)2 + b(x + t) + c = ax2 + (b + 2at)x + at2 + bt + c (2)∆2 = (b + 2at)2 – 4a(at2 + bt + c) = b2 + 4abt + 4a2t2 – 4a2t2 – 4abt – 4ac ⇒∆2 = b2 – 4ac = ∆0 = ∆1

Como o discriminante de x2 – x – 2 é 9 e o discriminante de x2 – x – 1 é 5, atransformação é impossível.

2. Restos de Divisões Invariantes

Exemplo 2.1: (Torneio das Cidades-85) Todo membro de uma seqüência, iniciando dosegundo, é igual a soma entre o termo precedente e a soma dos seus dígitos. Oprimeiro número é 1. É possível que 123456 pertença à seqüência?

Resolução:Perceba que: a2 = 2 = 3.0 + 2 a3 = 4 = 3.1 + 1 a4 = 8 = 3.2 + 2a5 = 16 = 3.5 + 1 a6 = 23 = 3.7 + 2 a7 = 28 = 3.9 + 1 ...Aparentemente os restos da divisão por 3 dos termos são alternadamente 1 e 2.Vamos demonstrar isto. Seja S(n) a soma dos dígitos de n. Sabemos que n e S(n)deixam o mesmo resto na divisão por 3:i) Se an = 3k1 + 1 ⇒ an + 1 = an + S(an) = 3k1 + 1 + 3k2 + 1 = 3k3 + 2.ii) Se an = 3k1 + 2 ⇒ an + 1 = an + S(an) = 3k1 + 2 + 3k2 + 2 = 3k3 + 1.Deste modo, se n é par então an = 3k + 2 e se n é ímpar então an = 3k + 1(invariante). Como 123456 é divisível por 3 então não pertence à seqüência.

Exemplo 2.2: (Leningrado-85) Três cangurus estão alinhados em uma estrada. A cadasegundo um dos cangurus salta. É permitido que um canguru salte por cima de umoutro canguru, mas não de dois cangurus de uma só vez. Prove que depois de 1985segundos, os cangurus não podem voltar a ocupar a posição relativa inicial.

Page 99: Eureka 2002

Sociedade Brasileira de Matemática

EUREKA! N°14, 2002

37

Resolução:Existem seis posições para os cangurus: 123 132 312 321 231 213Note que as posições sublinhadas somente podem ser alcançadas através de umposição anterior em negrito e vice-versa (invariante). Assim, depois de um númeroímpar de pulos somente as posições sublinhadas (132, 321 e 213) podem seralcançadas, fazendo com que depois de 1985 pulos não seja possível que os cangurusvoltem a ocupar a posição inicial (123).

Exemplo 2.3: (Pará-2001) Um tabuleiro 4x4 possui, inicialmente, todas as casaspintadas de branco. Uma operação permitida é escolher um retângulo consistindo de3 casas e pintar cada uma das casas da seguinte forma:– se a casa é branca então pinta-se de preto;– se a casa é preta então pinta-se de branco.Prove que, aplicando várias vezes a operação permitida, é impossível conseguirmosque todo o tabuleiro fique pintado de preto.

Resolução:Distribuimos as letras a, b e c no tabuleiro da seguinte forma:

c

c

c c

c

bb

bb

b

a

a

a

a

a

a

Note que as letras estão alternadas tanto nas linhas quanto nas colunas. Estaalternância faz com que toda vez que um retângulo com 3 casas seja selecionado,então exatamente uma letra a, uma letra b e uma letra c são selecionadas.Sejam: A a quantidade de casas brancas com a letra a, B a quantidade de casasbrancas com a letra b e C a quantidade de casas brancas com a letra c. No iníciotemos: A = 6 B = 5 C = 5. Toda vez que selecionamos um retângulo formado detrês casas, estamos somando a cada valor de A, B e C os valores + 1 ou – 1. Percebaque se todas casas ficarem pretas, então teremos A = 0, B = 0 e C = 0. Entretanto,note que iniciando de A = 6, B = 5 e C = 5, e alterando simultaneamente por + 1 ou –1 estes valores, sempre teremos entre os valores de A, B e C dois números ímpares eum par ou dois pares e um ímpar (invariante), fazendo com que a situação A = 0, B= 0 e C = 0 seja impossível.

Exemplo 2.4: (OBM Jr.-95) Temos um tabuleiro 1995 × 1995. A cada uma de suas19952 casas associamos um dos números + 1 ou – 1. Em seguida, associamos a cada

Page 100: Eureka 2002

Sociedade Brasileira de Matemática

EUREKA! N°14, 2002

38

linha o produto dos números das casas desta linha, e a cada coluna o produto dosnúmeros das casas de cada coluna.i) Se T é a soma dos números associados às linhas, colunas e casas, prove que T édiferente de 0.ii) Se S é a soma dos números associados às linhas e às colunas, prove que S édiferente de 0.

Resolução: i) Como temos 1995 colunas e 1995 linhas, então as somas dos números associadosàs linhas e colunas são números ímpares, pois são soma ou subtração de 1995números ímpares. Em relação às casas temos o mesmo raciocínio, pois são ao todo19952 casas, cujo valor de cada casa pode ser igual a 1 ou – 1. Somando todos estesvalores teremos também um valor ímpar. Assim, T é a soma de três valores ímpares,sendo também ímpar, nunca podendo assumir o valor zero.ii) Inicialmente notemos que qualquer disposição no tabuleiro pode ser alcançadapartindo de uma configuração inicial na qual todas as casas possuem valor 1 ealterando-se os sinais desejados. Com o tabuleiro contendo somente 1's, temos queas somas das linhas e colunas é 1995, fazendo uma soma total de 3990, queobviamente é o maior valor possível. Quando trocamos um 1 por um – 1, notamosque as somas das linhas e colunas passam a ser 1993, fazendo S = 3986. Assim, ficaevidente que uma alteração de um sinal em uma casa do tabuleiro altera o valor dasoma da linha e da coluna a qual pertence esta casa em ± 2, e por conseqüência alteraa soma total em 0 ou ± 4 (invariante).Deste modo, a soma total pode ser escrita da forma S = 3990 – 4k.Caso S = 0, teríamos 4k = 3990, que não possui solução inteira, absurdo.

Exemplo 2.5: (Hong Kong-97) Cinco números 1, 2, 3, 4, 5 estão escritos em umquadro negro. Um estudante pode apagar dois dos números a e b no quadro eescrever os números a + b e ab nos seus lugares. Se esta operação é repetidaindefinidamente, podem os números 21, 27, 64, 180, 540 aparecer no quadro negroao mesmo tempo?

Resolução:Não é possível. Note que no início existe somente um número divisível por 3 e nofinal existem quatro números divisíveis por 3. Observemos o que acontece com osrestos da divisão por 3 dos números no quadro quando fazemos uma operação:i) se a = 3x e b = 3y ⇒ a + b = 3z e ab = 3wii) se a = 3x + 1 e b = 3y ⇒ a + b = 3z + 1 e ab = 3wiii) se a = 3x + 2 e b = 3y ⇒ a + b = 3z + 2 e ab = 3wiv) se a = 3x + 1 e b = 3y + 1 ⇒ a + b = 3z + 2 e ab = 3w + 1

Page 101: Eureka 2002

Sociedade Brasileira de Matemática

EUREKA! N°14, 2002

39

v) se a = 3x + 1 e b = 3y + 2 ⇒ a + b = 3z e ab = 3w + 2vi) se a = 3x + 2 e b = 3y + 2 ⇒ a + b = 3z + 1 e ab = 3w + 1Portanto, a única forma de aumentar os divisíveis por 3 é escolher a = 3x + 1 e b =3y + 2. Consequentemente também acrescentamos um número da forma 3k + 2. Poroutro lado, na situação final o único número que não é divisível por 3 é 64, que é daforma 3k + 1, contradição, pois este número deveria ser da forma 3k + 2 (O caso iii)mostra que sempre haverá um número da forma 3k + 2 após termos 4 deles da forma3k).

3. Tendências de Crescimento ou Decrescimento Invariantes

Exemplo 3.1: Um total de 2000 pessoas estão divididas entre os 115 quartos de umamansão. A cada minuto, até que todas não estejam em um mesmo quarto, umapessoa anda para um quarto com um número igual ou maior de pessoas do que oquarto que ocupava. Prove que eventualmente todas as pessoas vão estar em ummesmo quarto.

Resolução:Seja ai a quantidade de pessoas no quarto i, 1 ≤ i ≤ 115.

Considere a expressão I = 2115

22

21 ... aaa +++ .

Digamos que uma pessoa sai de um quarto que possui n pessoas e vai para um quartoque possui m pessoas (m ≥ n). A variação de I é:∆I = ((m + 1)2 + (n – 1)2) – (m2 + n2) = 2(m – n + 1) > 0Assim, toda vez que uma pessoa troca de quarto o valor de I cresce (tendência decrescimento invariante). Entretanto note que o valor de I não pode crescerindefinidamente, uma vez que o número total de pessoas é finito, implicando queuma hora todas as pessoas estarão em um mesmo quarto.

Exemplo 3.2: (2a Lista de Preparação para a Cone Sul-2001) Existem inicialmente nnúmeros 1 em um quadro negro. Em cada passo é permitido apagar quaisquer dois

números a e b e escrever o número ba

ab+

2 . Esta operação é feita n – 1 vezes. Prove

que o último número não é menor que n

1 .

Resolução:Suponha que depois de k operações temos os seguintes números escritos no quadro:

a1, a2, ..., an – k. Considere a expressão Ik = 22

221

1...11

knaaa −+++ .

Page 102: Eureka 2002

Sociedade Brasileira de Matemática

EUREKA! N°14, 2002

40

Depois de uma operação a variação de I vale:

∆I = 22

2

22

21111

2111

2 bababaabba −−

+=−−

+ ⇒ ∆I =

22 21

211

baab−−

Pela desigualdade entre as médias aritmética e geométrica:

abbaba 111

2

11

22

22=≥

+ ⇒ 0

21

211

22≤−−

baab

Desta forma ∆I ≤ 0, ou seja, I nunca cresce.

Como I0 = n e In – 1 = 21

1a

então In – 1 ≤ I0 ⇒ na

≤21

1 ⇒ a1 ≥n

1 .

Exemplo 3.3: (St. Petersburg-96) Vários inteiros positivos distintos estão escritos emum quadro negro. Uma operação permitida é apagar dois inteiros distintos e escreverem seus lugares o máximo divisor comum e o mínimo múltiplo comum destesnúmeros. Prove que, depois da aplicação de operações permitidas várias vezes, osnúmeros eventualmente vão parar de mudar.

Resolução:Suponha que a e b (a > b) são dois números escritos no quadro. SejamD = mdc (a, b) e L = mmc (a, b). Consequentemente: D < a, D ≤ b, L ≥ a, L > b.Como ab = DL ⇒ ab + b2 = DL + b2 (1)Como (L – b)(D – b) ≤ 0 ⇒ DL – Lb – bD + b2 ≤ 0 ⇒DL + b2 ≤ bL + bD (2)Aplicando (2) em (1): ab + b2 ≤ bL + bD ⇒ a + b ≤ D + L (3)Assim, a soma S dos números escritos no quadro nunca decresce. Repare tambémque a igualdade em (3) implica (L – b)(D – b) = 0, ou seja, D = b ⇒ L = a.Por outro lado, como mdc (x1, x2, ..., xm) ≤ mmc (x1, x2, ..., xm) ≤ x1x2...xm, então,depois de um número qualquer de operações realizadas, cada número no quadro émenor ou igual que o produto de todos os números inicialmente escritos. Portanto, Sé menor ou igual a n vezes o produto dos números inicialmente escritos. Como pode-se fazer um número infinito de operações e S possui ao mesmo tempo aspropriedades de ser limitada e nunca decrescer, então a partir de um certo momentoS fica constante, fazendo com que D = b e L = a, implicando que eventualmente osnúmeros vão parar de mudar.

Page 103: Eureka 2002

Sociedade Brasileira de Matemática

EUREKA! N°14, 2002

41

Exercícios:

01. Um círculo é dividido em seis setores. Os números 1, 0, 1, 0, 0, 0 são escritos emsentido horário. É permitido aumentar em 1 dois números vizinhos. É possível queem algum momento todos os números sejam iguais?

02. Divide-se um círculo em 10 setores e coloca-se uma ficha em cada setor. Ummovimento consiste em selecionar duas fichas e mover cada uma para um setoradjacente. Prove que, depois de uma seqüência arbitrária de movimentos, éimpossível que todas as fichas localizem-se em um mesmo setor.

03. (Rio Grande do Norte-99) A professora desafia André e Thiago com o seguintejogo, em que eles jogam alternadamente. Ela escreve no quadro-negro os inteiros de1 a 50. Uma jogada consiste em escolher dois dos números escritos, apagar essesnúmeros, substituindo-os pela soma (Por exemplo, se André escolheu 8 e 23, apaga-os e escreve 31). Depois de algum tempo, vai restar no quadro negro um úniconúmero. Se esse número é par, o ganhador é André, caso contrário, o ganhador éThiago. Quem vence o jogo: André ou Thiago?

04. (Torneio das Cidades-93) Três pilhas de caroços são dadas sobre uma mesa. Épermitido adicionar ou remover de uma pilha um número de caroços que é igual asoma do número de caroços das outras duas pilhas. Por exemplo [12, 3, 5] podetornar-se [12, 20, 5] pela adição de 17 = 12 + 5 para a pilha de 3 ou tornar-se [4, 3, 5]pela remoção de 8 = 3 + 5 caroços da pilha com 12. É possível, iniciando com pilhaspossuindo 1993, 199 e 19 caroços, conseguir uma pilha vazia depois de umaseqüência de operações permitidas?

05. (Torneio das Cidades-85) Na ilha de Camelot vivem 13 camaleões roxos, 15verdes e 17 amarelos. Quando dois de cores distintas se encontram, mudamsimultaneamente para a terceira cor. Poderia dar-se a situação na qual todos tenhama mesma cor?

06. (Rússia-78) São dadas 3 máquinas que produzem cartões com pares de númerosnaturais. A primeira, sendo dado o cartão com (a, b), produz um novo cartão com(a + 1, b + 1); a segunda, sendo dado o cartão com (a, b), produz novo cartãocom (a/2, b/2), se ambos a e b são pares e nada no caso oposto e a terceira máquina,sendo dados os cartões (a, b) e (b, c), produz um novo cartão com (a, c). Todas asmáquinas retornam também os cartões iniciais. Suponha que foi dado o cartão inicial(5, 19). É possível obter:a) (1, 50)?

Page 104: Eureka 2002

Sociedade Brasileira de Matemática

EUREKA! N°14, 2002

42

b) (1, 100)?c) Suponha que foi dado o cartão inicial (a, b) (a < b). Nós queremos obter o cartão(1, n). Para quais n isto é possível?

07. Em um quadro negro estão escritos n números. A cada minuto apaga-se doisnúmeros a e b e escreve-se o número (a + b)/4. Repetindo esta operação n – 1 vezes,existirá somente um número no final. Prove que se inicialmente existirem n 1's noquadro, então o último número não é menor que 1/n.

08. (Leningrado-89) Vários (mas não menos que 2) números não nulos são escritosem um quadro negro. É possível apagar dois dos números, A e B, e então escrevernos seus lugares os números A + B/2 e B – A/2. Prove que o conjunto de númerosno quadro negro, depois de um número qualquer de operações, não pode coincidircom o conjunto inicial.

09. (2a Lista de Preparação para a Cone Sul-96) 119 anões vivem em uma aldeia com120 pequenas casas. Uma casa é dita super-habitada se 15 anões ou mais vivem lá.Todo dia, os anões de uma casa super-habitada têm uma discussão e se mudam paraoutras (distintas) casas da aldeia. Algum dia, necessariamente, esse processo seencerrará?

Bibliografia:

[1] A. Engel, Problem-Solving Strategies, Springer-Verlag, 1998.[2] P. J. Taylor, Tournament of the Towns 1984-1989, Australian InternationalCentre for Mathematics Enrichment, 1992.[3] D. Fomin, A. Kirichenko, Leningrad Mathematical Olympiads 1987-1991, MathProPress, 1994.

Page 105: Eureka 2002

Sociedade Brasileira de Matemática

EUREKA! N°14, 2002

43

XXII TORNEIO DAS CIDADESPrimavera 2001 – Nível O – Júnior

PROBLEMA 1O número natural n pode ser trocado por ab se a + b = n, onde a e b também sãonúmeros naturais. O número 2001 pode ser obtido a partir de 22 através destastrocas?

PROBLEMA 2Um dos segmentos que liga os pontos médios dos lados de um triângulo é maior queuma das medianas do triângulo. Prove que o triângulo é obtusângulo.

PROBLEMA 3Vinte quilogramas de queijo estão à venda em uma mercearia e os fregueses estãoem fila para comprar esse queijo. Após algum tempo, tendo acabado de atender umdos fregueses, a vendedora calcula (corretamente) a quantidade total de queijo jávendida e anuncia o número de fregueses para os quais há queijo, na quantia exata,se cada freguês comprar uma porção cuja massa é exatamente igual a quantidademédia comprada pelos anteriores.Pode ocorrer de a vendedora poder declarar, após cada um dos 10 primeirosfregueses ter feito sua compra, que há queijo na quantia exata para os próximos 10?Se isso puder ocorrer, quanto queijo haverá ainda na mercearia após os primeiros 10fregueses terem feito suas compras?(A quantidade média de uma seqüência de compras é a massa total de queijo vendidadividida pelo número de vendas.)

PROBLEMA 4a) Há 5 triângulos de papel idênticos sobre uma mesa. Cada um pode ser transladadoem qualquer direção. É verdade, então, que qualquer um deles pode ser coberto pelosoutros 4?b) Há 5 triângulos equiláteros de papel idênticos sobre uma mesa. Cada um pode sertransladado em qualquer direção. Prove que, então, qualquer um deles pode sercoberto pelos outros 4.

PROBLEMA 5Sobre um tabuleiro 15 × 15 são colocados quinze cavalos de modo que em cadafileira (linha ou coluna) do tabuleiro haja exatamente um cavalo. Então,simultaneamente, cada cavalo faz um movimento segundo as regras do xadrez.Prove que após os movimentos haverá dois cavalos em uma mesma fileira dotabuleiro.

Page 106: Eureka 2002

Sociedade Brasileira de Matemática

EUREKA! N°14, 2002

44

Primavera 2001 – Nível O – Sênior

PROBLEMA 1Um ônibus que percorre um trajeto de 100 km é equipado com um computador, queprevê quanto tempo falta para chegar ao destino final. Esta previsão é feitaassumindo que a velocidade média do ônibus na parte restante do trajeto será amesma da parte já percorrida. Quarenta minutos após a partida do ônibus, ocomputador previu que o tempo restante de viagem seria de uma hora. E este tempoprevisto permaneceu inalterado pelas próximas 5 horas.

Isso pode de fato ocorrer? Caso possa, quantos quilômetros o ônibus percorreunestas 5 horas?(A velocidade média do ônibus é o número de quilômetros percorridos dividido pelotempo gasto para percorrê-los.)

PROBLEMA 2A representação decimal do número natural a consiste de n dígitos, enquanto que arepresentação decimal de a3 consiste de m dígitos. Pode n + m ser igual a 2001?

PROBLEMA 3No triângulo ABC o ponto X está sobre o lado AB, enquanto o ponto Y está sobre olado BC. Os segmentos AY e CX interceptam-se no ponto Z. Sabe-se que AY = YC eAB = ZC. Prove que os pontos B, X, Z e Y estão sobre uma circunferência.

PROBLEMA 4Duas pessoas jogam sobre um tabuleiro 3 × 100. Elas jogam alternadamente: aprimeira coloca dominós 1 × 2 sobre o tabuleiro, a segunda coloca dominós 2 × 1. Aperdedora é aquela que, na sua vez, não puder colocar dominó.Qual dos dois jogadores têm estratégia vencedora? Descreva-a.

PROBLEMA 5Nove pontos são desenhados sobre a superfície de um tetraedro regular de aresta 1cm. Prove que entre estes pontos existem dois cuja distância (no espaço) é menor ouigual a 0.5 cm.

Page 107: Eureka 2002

Sociedade Brasileira de Matemática

EUREKA! N°14, 2002

45

XXIII TORNEIO DAS CIDADESOutono 2002 – Nível O – Júnior

PROBLEMA 1É dado um número suficientemente grande de cartões retangulares a cm × b cm,onde a e b são inteiros positivos e a é menor do que b. Sabe-se que com esses cartõespodemos montar (sem sobrepor cartões e sem buracos) um retângulo 49 cm × 51 cme um 99 cm × 101 cm.Os valores de a e b estão determinados unicamente a partir desses dados?

PROBLEMA 2Dado um triângulo qualquer, é possível cortá-lo em quatro conjuntos convexos: umtriângulo, um quadrilátero, um pentágono e um hexágono?

PROBLEMA 3

O número 22 yxyx ++ , no qual x e y são inteiros positivos, escrito na notaçãodecimal termina em zero. Prove que ele termina em pelo menos dois zeros.

PROBLEMA 4Os lados AB, BC, CD e DA do quadrilátero ABCD são tangentes a umacircunferência nos pontos K, L, M e N, respectivamente; S é o ponto de intersecçãodos segmentos KM e LN. Sabe-se que o quadrilátero SKLB é inscritível.Prove que o quadrilátero SNDM também é inscritível.

PROBLEMA 5a) São dadas 128 moedas de duas massas distintas, 64 de cada tipo. Como obter duasmoedas de massas distintas fazendo não mais de 7 pesagens um uma balança comdois braços (e sem pesos auxiliares)?

b) São dadas oito moedas de duas massas distintas, 4 de cada tipo. Como obter duasmoedas de massas distintas fazendo não mais de duas pesagens em uma balança comdois braços (e sem pesos auxiliares)?

Page 108: Eureka 2002

Sociedade Brasileira de Matemática

EUREKA! N°14, 2002

46

Outono 2002 – Nível A – Júnior

PROBLEMA 1Sejam a, b, e c as medidas dos lados de um triângulo. Prove a desigualdade

333 3 cabcba >++

PROBLEMA 2Quatro peças (duas brancas e duas pretas) são colocadas em um tabuleiroquadriculado 23 × 23: as peças brancas são colocadas no canto superior esquerdo eno canto inferior direito; as peças pretas são colocadas no canto inferior esquerdo eno canto superior direito. As brancas e pretas se movem alternadamente, sendo queas brancas começam. Em cada movimento, uma peça é movida para qualquer casavizinha (isto é, que tem uma lado em comum) que não tenha peça. O objetivo daspeças brancas é ocupar duas casas vizinhas. As pretas podem evitar que issoaconteça?

PROBLEMA 3Sejam E e F os pontos médios dos lados BC e CD, respectivamente, do quadriláteroconvexo ABCD. Os segmentos AE, AF e EF dividem o quadrilátero em 4 triânguloscujas áreas (em alguma ordem) são inteiros positivos consecutivos.Qual é a maior área que o triângulo ABC pode ter?

PROBLEMA 4N lâmpadas estão enfileiradas. Inicialmente, algumas delas estão acesas. A cadaminuto, todas as lâmpadas acesas são apagadas e acendemos cada lâmpada apagadaque for vizinha a exatamente uma lâmpada que estava acesa. Para que valores de n épossível escolher uma configuração inicial de lâmpadas de modo que em qualquermomento pelo menos uma lâmpada esteja acessa?

PROBLEMA 5Um triângulo acutângulo é cortado por uma reta em dois pedaços (nãonecessariamente triangulares). Em seguida, um dos pedaços é cortado por uma retaem dois pedaços, e assim por diante: a cada passo um dos pedaços obtido emqualquer passo anterior é escolhido e cortado por uma reta em dois novos pedaços.Após um certo número de passos, todos os pedaços são triangulares. É possível quetodos tenham um ângulo obtuso?

Page 109: Eureka 2002

Sociedade Brasileira de Matemática

EUREKA! N°14, 2002

47

PROBLEMA 6Em uma seqüência crescente de inteiros positivos, cada termo, a partir do 2002-ésimo, divide a soma de todos os termos anteriores. Prove que cada termo daseqüência, a partir de um certo ponto, é igual à soma de todos os anteriores.

PROBLEMA 7Dada uma cadeia de dominós, montada de acordo com as regras usuais, é permitida aseguinte operação: escolha uma subcadeia contida nela cujas extremidades são iguais(têm o mesmo número de pontos marcados), retire-a, inverta sua ordem e recoloque.Mostre que, dadas duas cadeias montadas a partir de um mesmo conjunto dedominós e com extremidades respectivamente iguais (isto é, os inícios são iguaisentre si, bem como os finais), é possível transformar, através de uma seqüência deoperações como a descrita, uma das cadeias na outra.

Page 110: Eureka 2002

Sociedade Brasileira de Matemática

EUREKA! N°14, 2002

48

OLIMPÍADAS AO REDOR DO MUNDO

Como sempre acontece no período em que é realizada a maior parte dascompetições internacionais o comitê editorial da revista EUREKA! se preocupa emmostrar as resoluções das competições nacionais dos anos anteriores para otreinamento dos nossos atuais e futuros competidores. Por isto, estivemos ausentesda edição anterior.

Continuamos a disposição na OBM para aqueles que estiverem interessadosna solução de algum problema particular. Para tanto, basta contactar a OBM, atravésde carta ou e-mail.

Antonio Luiz Santos

151. (Irlanda-2001) Seja ABC um triângulo de lados ABCABC ,, cujas medidas sãorespectivamente iguais a a, b, c. Se D e E são os pontos médios de AC e ABrespectivamente, mostre que a mediana BD é perpendicular a CE se, e somente

se, 222 5acb =+ .

152. (Irlanda-2001) Mostre que se um número primo ímpar p pode ser colocado sob a

forma 55 yx − para alguns inteiros x e y então

2

1

5

14 2 +=+ vp

para algum inteiro ímpar v.

153. (Irlanda-2001) Determine os números reais x não negativos para os quais33 1313 xx −++

é um número inteiro.

154. (Irlanda-2001) Determine todas as funções →:f que satisfazem

( )( ) ( ) N.,, ∈∀+=+ yxyxfyfxf

155. (Rússia-2001) Os números de 1 a 999999 são divididos em dois grupos: noprimeiro, cada número nele colocado é tal que o quadrado perfeito mais próximodele é o quadrado de um número ímpar. No segundo, os números estão maispróximos de quadrados perfeitos de números pares. Determine em qual dosgrupos a soma dos números a ele pertencentes é maior.

Page 111: Eureka 2002

Sociedade Brasileira de Matemática

EUREKA! N°14, 2002

49

156. (Rússia-2001) Sobre o maior lado AC de um triângulo ABC , toma-se um pontoN de modo que as mediatrizes dos segmentos AN e NC intersectam os ladosAB e BC nos pontos K e M, respectivamente. Prove que o circuncentro O dotriângulo ABC pertence ao círculo circunscrito ao triângulo KBM.

157. (Rússia-2001) Dois círculos são tangentes internamente no ponto N . Umatangente traçada de um ponto K do círculo interno intersecta o círculo externonos pontos A e B . Se M é o ponto médio do arco AB que não contém o pontoN , mostre que o raio do círculo circunscrito ao triângulo BMK não depende daescolha do ponto K do círculo interior.

158. (Rússia-2001) A seqüência ( )nx é tal que 11 =x , 11 +⋅=+ nn xsennx . Mostre queesta seqüência não é periódica.

159. (Eslovênia-2001) Sejam 4321 ,,, aaaa e 5a números reais distintos. Denotando por

m o número de valores distintos das somas ji aa + , onde 51 ≤≤≤ ji ,

determine o menor valor possível de m .

160. (Eslovênia-2001) Sejam edcba ,,,, e f números reais positivos tais que a

seqüência ( )dcba ,,, seja aritmética e a seqüência ( )dfea ,,, seja geométrica.Mostre que efbc ≥ .

161. (Eslovênia-2001) Seja D o pé da altura relativa ao lado BC do triângulo ABC .Sabendo que a bissetriz interna do ângulo C∠ intersecta o lado oposto no ponto

E e que 4

π=∠ CEA , determine a medida do ângulo EDB∠ .

162. (Eslovênia-2001) (a) Mostre que a desigualdade

12

11 −−<<−+ nn

nnn

é verdadeira para todo inteiro positivo n .

(b) Mostre que a parte inteira da expressão

22

1

1

1

3

1

2

11

mm+

−+⋅⋅⋅+++

onde m é um inteiro positivo é igual a 22 −m ou 12 −m .

Page 112: Eureka 2002

Sociedade Brasileira de Matemática

EUREKA! N°14, 2002

50

163. (Croácia-2001) Resolva a inequação

1,0,2log1 ≠>>+ aaxax xa

164. (Croácia-2001) A partir dos pontos médios dos lados de um triângulo acutângulosão traçadas perpendiculares aos outros dois lados. Mostre que a área dohexágono definido por esses segmentos é igual à metade da área do triângulo.

165. (Croácia-2001) Determine todos os pares ordenados de números reais yx, quesatisfazem à equação

( ) ( )2

22

2 2

11

cos2

1cos2log

−−=−

+ y

xyxy

166. (Croácia-2001) Quatro esferas de raios iguais a R são mutuamente tangentes entresi. Determine o raio r da maior esfera que pode ser inserida entre elas.

167. (Bielorússia-2001) No gráfico da parábola 2xy = no plano cartesiano marcamos ospontos BA, e C (com A entre B e C ). No segmento BC marca-se o ponto

N de modo que AN seja paralelo ao eixo das ordenadas. Se 1S e 2S são as

áreas dos triângulos ABN e ,ACN respectivamente, determine a medida dosegmento AN.

168. (Bielorússia-2001) A comissão organizadora da OBM preparou algumas variantesde uma competição com problemas4 cada uma. Duas variantes distintas podemconter um mesmo problema mas não mais do que um. Qual o menor número deproblemas necessários para que a comissão organizadora prepare 10 variantesda competição?

169. (Bielorússia-2001) O quadrilátero ABCD está inscrito num círculo 1S ; um outro

círculo 2S passa pelo ponto D , pelo ponto O , de interseção das diagonais doquadrilátero, e intersecta AD e CD em M e N, respectivamente. Sabendo queOM e AB intersectam-se em R enquanto que ON e BC intersectam-se emT ( R , T e A pertencem ao mesmo semiplano em relação a BC ), mostre que

TRO ,, e B pertencem ao mesmo círculo.

Page 113: Eureka 2002

Sociedade Brasileira de Matemática

EUREKA! N°14, 2002

51

170. (Bielorússia-2001) Determine o resto da divisão de

( ) 40601!20013!11!25!1 2 ⋅+⋅⋅⋅+++⋅+⋅⋅⋅+⋅+⋅ kkk por 2004 .

171. (Inglaterra-2001) Determine todos os números naturais N de dois algarismos para

os quais a soma dos algarismos de NN −10 é divisível por 170 .

172. (Inglaterra-2001) Um círculo S é interior a um círculo T e o tangencia no pontoA . De um ponto AP ≠ sobre T , traçam-se as cordas PQ e PT de T quetangenciam S em X e Y, respectivamente. Mostre que XAYQAR ∠=∠ 2 .

173. (Inglaterra-2001) Uma seqüência ( )na é tal que nnan += , onde n é um

inteiro positivo e x é o inteiro mais próximo a x , sendo as metades

arredondadas para cima se for necessário. Determine o menor inteiro k para osquais os termos 20001,...,, ++ kkk aaa formam uma seqüência de 2001 inteirosconsecutivos.

174. (Inglaterra-2001) As medidas dos lados de um triângulo são cba ,, e a medida doraio do círculo circunscrito ao triângulo é R . Mostre que o triângulo é retângulo

se, e somente se, 2222 8Rcba =++ .

175. (Torneio das Cidades-2001) Diga se existem inteiros positivos 10021 ... aaa <<< tais

que para todo k , 1002 <≤ k , o mínimo múltiplo comum de 1−ka e ka é maior

que o mínimo múltiplo comum de ka e 1+ka .

176. (Torneio das Cidades-2001) Os vértices de um triângulo têm coordenadas ( )11, yx ,

( )22 , yx e ( )33 , yx . Para quaisquer inteiros h e k , o triângulo cujos vértices têm

coordenadas ( )kyhx ++ 11 , , ( )kyhx ++ 22 , e ( )kyhx ++ 33 , é disjunto dotriângulo original.

(i) É possível que a área deste triângulo seja maior que ?2

1

(ii) Qual a área máxima deste triângulo?

Page 114: Eureka 2002

Sociedade Brasileira de Matemática

EUREKA! N°14, 2002

52

177. (Torneio das Cidades-2001) Sejam a e d inteiros positivos tais que, para qualquerinteiro positivo n , a expansão decimal de nda + contém um bloco dealgarismos consecutivos igual à expansão decimal de n. Prove que d é umapotência de 10.

178. (Áustria-Polônia-2001) Determine o número de inteiros positivos a para os quaisexistem inteiros não negativos 200110 ,...,, xxx que satisfazem a

∑=

=2001

1

0

k

xx kaa

179. (Estônia-2001) Os ângulos de um ágonon − convexo são ααα n,...,2, .Determine todos os valores possíveis de n e os valores correspondentes de α .

180. (Estônia-2000) Mostre que para todo inteiro 1>a , existe um número primo p tal

que 121 −+⋅⋅⋅+++ paaa é composto.

Acusamos o recebimento de soluções de problemas anteriores dos seguintes leitoresde EUREKA!:. No próximo número publicaremos algumas delas.

Alessandra A. da Gama Gomes de A. Rio de Janeiro – RJ Prob. 119.Anderson Torres São Paulo – SP Prob. 4, 16, 28, 31, 36, 46, 47, 53, 57, 59, 65,66, 68, 78, 80, 85, 88,

89, 95, 104, 109, 121, 123, 130,135,137, 145.Bruno Borges de Souza Lima Goiânia – GO Prob. 102, 107, 147.Bruno de Souza Ramos Rio de Janeiro – RJ Prob. 127, 128, 131, 132, 134, 138, 139, 143, 144.Carlos José Amorim da Silva Rio de Janeiro – RJ Prob. 133, 138, 143.Diego Alonso Teixeira Santos – SP Prob. 121.Evandro Makiyama de Melo São Paulo – SP Prob. 51, 121, 123, 128, 131, 133, 135, 142, 147.Filipe Rodrigues de Souza Moreira Rio de Janeiro – RJ Prob. 64, 121, 128, 131, 133, 135, 138, 145, 147.Geraldo Perlino Jr. Cotia – SP Prob. 121, 122, 125 a 139, 141 a 144, 146 a 148.Gibran Medeiros de Souza Natal – RN Prob. 138.Helainy Ignácio de Almeida Torres Rio de Janeiro – RJ Prob. 135, 137, 147.Helder Oliveira de Castro Mogi das Cruzes – SP Prob. 32, 57, 68, 69, 76, 87, 98, 121,127, 132, 138, 139.Karla Detagne Santos Rio de Janeiro – RJ Prob. 128, 138.Leonardo Freitas de Lima Rio de Janeiro – RJ Prob. 138.Luiz Sérgio Carvalho de Mello Rio de Janeiro – RJ Prob. 8, 47, 69, 127, 128, 132, 143.Marcelo Ribeiro de Souza Rio de Janeiro – RJ Prob. 121, 122, 124, 126, 127, 128, 130, 132, 133, 134, 138, 143,

147.Marcelo Rufino de Oliveira Belém – PA Prob. 121 a 150.Marcílio Miranda de Carvalho Teresina – PI Prob. 66, 87, 106.Mauro Félix de Souza Rio de Janeiro – RJ Prob. 121, 128, 134.Okakamo Matsubachi São Paulo – SP Prob. 130.Raquel Teresa de Souza Gomes Rio de Janeiro – RJ Prob. 128, 133, 143.Renato Francisco Lopes Mello J. dos Guararapes – PE Prob. 127, 132, 143.Wallace Alves Martins Rio de Janeiro – RJ Prob. 121, 128, 130, 133, 138.

Page 115: Eureka 2002

Sociedade Brasileira de Matemática

EUREKA! N°14, 2002

53

SOLUÇÕES DE PROBLEMAS PROPOSTOS Publicamos aqui algumas das respostas enviadas por nossos leitores.

57) Dado n números reais x1, x2 …, xn satisfazendo as condições x1 +…+ xn = 0 e

1... 221 =++ nxx , prove que existem i e j tais que .

1

nxx ji −≤

SOLUÇÃO DE ZOROASTRO AZAMBUJA NETO (RIO DE JANEIRO – RJ):

Podemos supor que .0,..., ,0,..., 11 <≥ + nkk xxxx

Temos ∑ ∑= +=

+−==+k

i

n

kjji n

k

n

knxx

1 1

22 .1

Assim, ∑=

−≥k

ii n

knx

1

2 ou ∑+=

≥n

kjj n

kx

1

2 . Supomos sem perda de generalidade que

∑=

−≥k

ii n

knx

1

2 (o outro caso é análogo). Assim, se ε≤ix para ,1 ki ≤≤ temos

∑=

−≥k

ii n

knx

1 ε, pois ∑ ∑

= =

≤k

i

k

iii xx

1 1

2 .ε Como ∑ ∑+= =

=−n

kj

k

iij xx

1 1

,)( temos

∑+=

−≥−n

kjj n

knx

1

)(ε

, donde existe njk ≤≤+ 01 com .1

0 nx j ε

≥− Tomando

,max1

iki

x≤≤

=ε temos que existe ki ≤0 com ,0

ε=ix donde n

xxx jji

1)(

000≥−=− ε , e

logo .1

00 nxx ji −≤

63) Prove que existem infinitos números naturais múltiplos de 51000 sem nenhum 0na representação decimal.

SOLUÇÃO DE EDUARDO CASAGRANDE STABEL (PORTO ALEGRE – RS):

Provaremos por indução que existe um número de k algarismos, todos diferentes de0 (zero), divisível por 5k. Para k = 1, tome a1 = 5; para k = 2, tome a2a1 = 25. Agorasuponhamos que seja n = ak – 1ak – 2…a2a1 divisível por 5k – 1.

Page 116: Eureka 2002

Sociedade Brasileira de Matemática

EUREKA! N°14, 2002

54

Temos .5

25

...10

5

...1

11

1211

1121

−−

−−

−− +=

+=

kk

kkk

kk

kkk n

aaaaaaaaa

É preciso que

05

21

1 ≡+ −−

kk

k

na módulo 5 para que ak ak – 1…a2a1 seja múltiplo de 5k. A equação é

equivalente a 11

11 35

32 −−

−− −≡≡ kk

kkkk

naa módulo 5.

Podemos escolher um tal ak no conjunto 1, 2, 3, 4, 5, pois aí temos um conjunto derestos da divisão por 5. Para concluir, basta tomar k = 1000, 1001, 1002,…

64) Iniciando de um certo inteiro positivo, é permitido fazer apenas umaoperação: o dígito das unidades é separado e multiplicado por 4, e então estevalor é somado ao restante do número. Por exemplo, o número 1997 étransformado para 7.4 + 199 = 227. A operação é feita repetidamente. Proveque se a seqüência de números obtida contém 1001, então nenhum dosnúmeros na seqüência pode ser um número primo.

ADAPTAÇÃO DA SOLUÇÃO DE MARCELO RIBEIRO DE SOUZA (RIO DE JANEIRO – RJ):Vamos ver que todos os termos da seqüência são múltiplos de 13, caso a seqüênciacontenha 1001 (que é múltiplo de 13). De fato, se bn = 10kn + rn , com 0 ≤ rn ≤ 9, opróximo termo será bn + 1 = kn + 4rn, ou seja, .3910)4(10 11 nnnnnn rbrrbb −=+−= ++

Assim, bn é múltiplo de 13 ⇔ bn + 1 é múltiplo de 13, ou nenhum deles é. Como ostermos antes de 1001 são maiores que 1001 e, partindo de 1001, obtemos aseqüência 1001 → 104 → 26 → 26 → …, todos os termos da seqüência sãomúltiplos de 13, mas nenhum é igual a 13, donde nenhum é primo.

65) Determine todos os inteiros N tais que, em base 10, os dígitos de 9N são osmesmos dígitos de N na ordem inversa, e N possui no máximo um dígito iguala 0.

SOLUÇÃO DE DANIEL DE SOUZA RAMOS (PIRASSUNUNGA – SP):Seja N um número de n dígitos na base 10, com no máximo um dígito igual a 0.Sabemos pelo enunciado definição do problema que 9N tem n dígitos. Daí temos queao multiplicarmos N por 9, 9N só terá n dígitos se o primeiro dígito de N for 1.Isto implica que o último dígito de 9N é 1. Mas tal fato só será possível se o últimodígito de N for 9. Assim N = 1…9 e 9N = 9…1.Imaginemos a multiplicação por 9. Para que o primeiro algarismo de 9N seja 9 énecessário que o dígito anterior ao dígito 1 de N seja 0 ou 1, uma vez que qualquerdígito maior do que 1, em tal posição, fará com que 9N tenha um número de dígitosdiferente do de N.

Page 117: Eureka 2002

Sociedade Brasileira de Matemática

EUREKA! N°14, 2002

55

Analisemos então os 2 casos:1o. Caso) N = 11…9 (9N = 9…11); 0 ≤ k ≤ 9, k ∈

7 )10(mod7

)10(mod189

9k...11

9

9...118

=⇒≡≡+

⇒× kk

kk

Daí: N = 11…79(9N = 97…11), 0 ≤ k', k'' ≤ 9, k', k'' ∈

1''8' )10(mod82'

)10(mod7'19

1 7....1 9

9

9 7....1 18k'''

=⇒=⇒≡−≡≡+⋅

⇒× kkk

kk

Porém com k'' = 1 não será possível que 9N seja da forma 97…11. Portanto o caso N= 11…9 não satisfaz.

2o. Caso) N = 10…9 (9N = 9…01); 0 ≤ k ≤ 9, k ∈

8 )10(mod8

)10(mod089

9k...01

9

9...108

=⇒≡≡+

⇒× kk

kk

Daí: N = 10…89(9N = 98…01), 0 ≤ k', k'' ≤ 9, k', k'' ∈ (4)

81). 99 e

80, achegar 8 a somado pode, 89 apenas (pois 9 ou

8 aconteça, situação esta que Para

1 0 ....' 8 9

9

9 8 '....0 188*8

=⋅⋅=

=

⇒× k

k

kk

kk

Para o caso k = 8 [k = 8 ⇒ k' = 0 pois 9k' + 8 ≡ 8 (mod 10) ∴ k' ≡ 0(mod 10)]

proposta. situação a com condizendo não ,0

dígito um de mais possuiria número o caso Neste

1 0 8....0 8 9

9

9 8 0....8 0 18888

N

⇒×

Para o caso k = 9:

Page 118: Eureka 2002

Sociedade Brasileira de Matemática

EUREKA! N°14, 2002

56

9')10(mod9'

)10(mod98'9

1 0 9....' 8 9

9

9 8 '....90 1888

=⇒≡≡+

⇒× kk

k

k

k

Temos agora que N = 1 0 9…9 8 9. Desenvolvendo:

9ou 8

aconteça situação esta que Para

1 0 9....'0 8 9

9

9 8 9 ' ... 9 0 18888

==⇒×

kkkk

kk

Para o caso k = 8.

0')10(mod0'

)10(mod889

1 0 9 8....' 9 8 9

9

9 8 9 ' ...8 9 0 1888

=⇒≡≡+

⇒×

kk

k'

k

k

Não satisfaz o problema pois possui mais de um dígito 0.

Para o caso k = 9.

9')10(mod9'

)10(mod989

1 0 9 9....' 9 8 9

9

9 8 9 ' ...9 9 0 188888

=⇒≡≡+

⇒×

kk

k'

k

k

Temos agora que N = 1 0 9 9 … 9 9 8 9. Desenvolvendo:

1 0 9 9....' 9 8 9 9

9

9 8 9 9 ' ...9 9 0 18888888

kk

kk

×

Procedendo de forma análoga ao momento em que sabia-se que N = 1 0 9 … 9 8 9,teremos N = 1 0 9 9 9… 9 9 9 8 9.Conclui-se então que todos os inteiros N que satisfazem a condição do problema sãoda forma:

4,899...910

)4(

≥∧∈=−

nnNgitosídn

.

Page 119: Eureka 2002

Sociedade Brasileira de Matemática

EUREKA! N°14, 2002

57

67) Este problema saiu com o enunciado errado, devido a um erro tipográfico,como observaram Rodrigo Villard Milet, Anderson Torres e Carlos da SilvaVictor, que enviaram versões corrigidas, com soluções. Pedimos desculpasaos leitores pelo erro e publicamos as versões corrigidas a seguir:

VERSÃO DE RODRIGO VILLARD MILET (RIO DE JANEIRO – RJ) eANDERSON TORRES (SÃO PAULO – SP):Seja ABCD um quadrilátero tal que os círculos circunscritos aos triângulos ABC eBCD são ortogonais. Prove que os círculos circunscritos aos triângulos ACD e ABDtambém são ortogonais.

SOLUÇÃO:Vamos denotar por (XYZ) o círculo que passa por X, Y e Z e por ϑ(w1, w2) e ânguloentre as curvas w1 e w2.Considere uma inversão ψ com centro em B e de razão k. Como (ABC) e (BCD)passam por B, ψ((ABC)) = A' C' e ψ((BCD)) = C'D'. Como ϑ((ABC), (BCD)) = 90º,segue que ∠ A'C'D' = 90º.Temos ).'''())(( DCAACD =ψ Daí, A'D' é diâmetro de (A'C'D'), pois ∠ A'C'D' =

90º. E assim, ϑ((A'C'D'), A'D') = 90º, logo ϑ(ψ ((A'C'D')), ψ (A'D')) = ϑ((ACD),(ABD)) = 90º, pois ψ (A'D') = (ABD)(A'D' não passa por B). Então os círculos (ACD)e (ABD) são ortogonais.

VERSÃO DE CARLOS ALBERTO DA SILVA VICTOR (NILÓPOLIS – RJ):Seja ABCD um quadrilátero tal que os círculos circunscritos aos triângulos ABC eACD são ortogonais. Prove que os círculos circunscritos aos triângulos BCD e DABtambém são ortogonais.

SOLUÇÃO:Sejam O1, O2, O3 e O4 os centros dos círculos circunscritos aos triângulos ABC,ADC, ADB e BDC respectivamente.Sejam: aCBO =∠ 1 ; bCDO =∠ 2 ; θ=∠ CAO1 ; eDBC =∠ ; dDBA =∠ ;

cBDA =∠ e .fBDC =∠ Note que, pela hipótese do problema,

.9022 θ−°=∠=∠ CAOACO Podemos concluir que

°++=−°++=+∠+∠+∠= 9090ˆ12211 babaCDOCAOCAOCBOC θθ .

Além disso, é fácil ver que θ⋅−°=∠ aABO 901 e ,2 bADO −=∠ θ portanto

=−+−°++−−°=∠+∠+∠+∠ baADOACOACOABO θθθθ )90()90(2211

.180 ba −−°= Veja também que feODO +=∠ 34 e .43 dcODO +=∠ Observe

agora no triângulo 43ODO em que e + f + c + d = 180° – 43 DOO∠ e, pelo

Page 120: Eureka 2002

Sociedade Brasileira de Matemática

EUREKA! N°14, 2002

58

quadrilátero ABCD temos °=+++ 360ˆˆˆˆ DCBA , ou seja e + f + c + d = 360° –

=+ )ˆˆ( AC 360° – [90° + a + b + 180° – a – b] ∴ e + f + c + d = 360° – 270° = 90°.

Conclusão: °=+++−°=∠−∠−°=∠ 90)(180180 433443 dcfeODOODODOO

e os círculos circunscritos aos triângulos BCD e DAB também são ortogonais.

Agradecemos também o envio das soluções e a colaboração de:

Fábio Nunes Ribeiro Maia Rio de Janeiro – RJAnderson Torres São Paulo – SPFábio Dias Moreira Rio de Janeiro – RJMarcelo Rufino de Oliveira Belém – PAHelder Oliveira de Castro Mogi das Cruzes – SPOswaldo Mello Sponquiado Olimpia – SPRenato Francisco Lopes Mello Jaboatão dos Guararapes – PEMarcílio Miranda de Carvalho Teresina – PIEvandro Makiyama de Melo São Paulo – SPGibran Medeiros de Souza Natal – RNCarlos A. Gomes Natal – RNJorge Silva Júnior Cachoeiro de Itapemirim – ES

Seguimos aguardando o envio de soluções do problema proposto Nº. 66 publicado na revistaEureka! Nº. 12

!" # ! $ % & '()& * ( + ,-'.,./ 0 * % * '(

1 % 2345

1 % 6 * % 7" %8 '6 9 * * * 8 : &

Page 121: Eureka 2002

Sociedade Brasileira de Matemática

EUREKA! N°14, 2002

59

PROBLEMAS PROPOSTOS

Convidamos o leitor a enviar soluções dos problemas propostos e sugestões de novosproblemas para os próximos números.

68) Seja ABC um triângulo de lados inteiros e área racional. Prove que existempontos X, Y, Z com coordenadas inteiras no plano 2 tais que o triângulo XYZé congruente ao triângulo ABC.

69) Sejam a e b inteiros positivos tais que an – 1 divide bn – 1 para todo inteiropositivo n.Prove que existe k ∈ tal que b = ak.

70) P1

X1

P2 P5

X5

Q5

Q3

Q4

Q1

Q2

X3

P4 P3

X4 X2

Na figura acima, para jXj ,51 ≤≤ é o ponto de interseção dos círculos

circunscritos aos triângulos Qj –1 PjQj e Qj Pj + 1 Qj + 1 distintos de Qj (os índicessão tomados módulo 5). Prove que o pentágono X1X2X3 X4X5 é inscritível.Obs: O pentâgono P1P2P3P4P5 não é necessariamente regular.

Page 122: Eureka 2002

Sociedade Brasileira de Matemática

EUREKA! N°14, 2002

60

71) Considere três circunferências, tangentes duas a duas. Prove que há apenasduas circunferências tangentes às três simultaneamente, e mostre comoconstruí-las.

72) Ache todas as funções f : → tais que )),(()()( 34 yffxfxyxf +=+., ∈∀ yx

73) Prove que, dado um inteiro positivo n, existe uma progressão aritméticacrescente formada por n inteiros positivos cujas somas dos dígitos tambémformam uma progressão aritmética crescente, mas não existe uma progressãoaritmética infinita de inteiros positivos cujas somas dos dígitos formem umaprogressão aritmética crescente.

Problema 70 proposto por Jiang Zemin, presidente da China, a membros da direção da UniãoInternacional de Matemática, durante uma reunião preparatória do Congresso Internacional deMatemática (ICM), realizado em agosto de 2002, em Beijing, China; Problema 71 proposto porMarcelo Ribeiro de Souza (Rio de Janeiro – RJ); Problema 72 adaptado da 14a. Asian PacificMathematical Olympiad; Problema 73 adaptado do 17o.Torneio das Cidades, 1995.

Page 123: Eureka 2002

Sociedade Brasileira de Matemática

EUREKA! N°14, 2002

61

AGENDA OLÍMPICA

XXIV OLIMPÍADA BRASILEIRA DE MATEMÁTICA

NÍVEIS 1, 2 e 3Primeira Fase – Sábado, 8 de junho de 2002

Segunda Fase – Sábado, 14 de setembro de 2002Terceira Fase – Sábado, 19 de outubro de 2002 (níveis 1, 2 e 3)

Domingo, 20 de outubro de 2002 (níveis 2 e 3 - segundo dia de prova).

NÍVEL UNIVERSITÁRIOPrimeira Fase – Sábado, 14 de setembro de 2002

Segunda Fase – Sábado, 19 e Domingo, 20 de outubro de 2002♦

VIII OLIMPÍADA DE MAIO11 de maio de 2002

XIII OLIMPÍADA DE MATEMÁTICA DO CONE SUL22 a 28 de junho de 2002

Fortaleza – CE, Brasil♦

XLIII OLIMPÍADA INTERNACIONAL DE MATEMÁTICA18 a 31 de julho de 2002Glasgow, Reino Unido

♦XVII OLIMPÍADA IBEROAMERICANA DE MATEMÁTICA

30 de setembro a 5 de outubro de 2002El Salvador

♦V OLIMPÍADA IBEROAMERICANA DE MATEMÁTICA UNIVERSITÁRIA

9 de novembro de 2002

♦♦ ♦

Page 124: Eureka 2002

Sociedade Brasileira de Matemática

EUREKA! N°14, 2002

62

COORDENADORES REGIONAIS

Alberto Hassen Raad (UFJF) Juiz de Fora – MG

Amarísio da Silva Araújo (UFV) Viçosa – MG

Benedito Tadeu Vasconcelos Freire (UFRN) Natal – RN

Carlos Frederico Borges Palmeira (PUC-Rio) Rio de Janeiro – RJ

Claudio Arconcher (Colégio Leonardo da Vinci) Jundiaí – SP

Claus Haetinger (UNIVATES) Lajeado – RS

Cleonor Crescêncio das Neves (UTAM) Manaus – AM

Élio Mega (Colégio Etapa) São Paulo – SP

Florêncio Ferreira Guimarães Filho (UFES) Vitória – ES

Gisele de Araújo Prateado Gusmão (UFGO) Goiânia – GO

Irene Nakaoka (UEM) Maringá – PR

Ivanilde Fernandes Saad (UC. Dom Bosco) Campo Grande– MS

Jacqueline Fabiola Rojas Arancibia (UFPB) João Pessoa – PB

João Benício de Melo Neto (UFPI) Teresina – PI

João Francisco Melo Libonati (Grupo Educacional Ideal) Belém – PA

José Carlos Pinto Leivas (UFRG) Rio Grande – RS

José Cloves Saraiva (UFMA) São Luis – MA

José Gaspar Ruas Filho (ICMC-USP) São Carlos – SP

José Luiz Rosas Pinho (UFSC) Florianópolis – SC

José Vieira Alves (UFPB) Campina Grande – PB

Licio Hernandes Bezerra (UFSC) Florianópolis – SC

Luzinalva Miranda de Amorim (UFBA) Salvador – BA

Marcelo Rufino de Oliveira (Grupo Educacional Ideal) Belém – PA

Marcondes Cavalcante França (UFC) Fortaleza – CE

Pablo Rodrigo Ganassim (Liceu Terras do Engenho) Piracicaba – SP

Ramón Mendoza (UFPE) Recife – PE

Reinaldo Gen Ichiro Arakaki (INPE) SJ dos Campos – SP

Ricardo Amorim (Centro Educacional Logos) Nova Iguaçu – RJ

Roberto Vizeu Barros (Colégio Acae) Volta Redonda – RJ

Rosângela Souza (Colégio Singular) Santo André – SP

Sérgio Cláudio Ramos (IM-UFRGS) Porto Alegre – RS

Tadeu Ferreira Gomes (UEBA) Juazeiro – BA

Tomás Menéndez Rodrigues (U. Federal de Rondônia) Porto Velho – RO

Valdenberg Araújo da Silva (U. Federal de Sergipe) São Cristovão – SE

Wagner Pereira Lopes (CEFET – GO) Jataí – GO

Page 125: Eureka 2002

CONTEÚDO

AOS LEITORES 2

XLIII OLIMPÍADA INTERNACIONAL DE MATEMÁTICA 3Enunciados, Soluções e Resultado Brasileiro

XVII OLIMPÍADA IBEROAMERICANA DE MATEMÁTICA 15Enunciados, Soluções e Resultado Brasileiro

ARTIGOS

A FÓRMULA DE CARDANO ALÉM DAS CÚBICAS 24José Cloves Verde Saraiva, São Luis - MA

RECIPROCIDADE QUADRÁTICA 27Carlos Gustavo T. de A. Moreira & Nicolau Corção Saldanha, Rio de Janeiro - RJ

APLICAÇÕES DE PLANOS PROJETIVOS EM TEORIA DOS NÚMEROS E COMBINATÓRIA 31Carlos Yuzo Shine, São Paulo - SP

OLIMPÍADAS AO REDOR DO MUNDO 43

SOLUÇÕES DE PROBLEMAS PROPOSTOS 54

PROBLEMAS PROPOSTOS 59

COORDENADORES REGIONAIS 61

Page 126: Eureka 2002

Sociedade Brasileira de Matemática

EUREKA! N°15, 2002

2

AOS LEITORES

Chegamos a esta última edição do ano 2002 muito contentes com odesempenho olímpico do Brasil: Pelo segundo ano consecutivo todos os integrantesda equipe brasileira ganharam medalha na Olimpíada Internacional de Matemática, ealém disso tivemos excelentes resultados na Olimpíada de Matemática do Cone Sul ena Olimpíada Iberoamericana de Matemática, onde ganhamos a maioria dasmedalhas de ouro em disputa. Publicamos aqui as soluções dos problemas da IMO eda Ibero, sendo a maioria delas dos membros das equipes brasileiras.

Agradecemos mais uma vez a crescente colaboração dos leitores, enviandoproblemas propostos e soluções, e pedindo que publiquemos soluções de problemasde várias fontes, como das Olimpíadas ao Redor do Mundo.

Esse intercâmbio é fundamental para nós, e ajuda a manter a revista Eureka!viva e interessante.

Abraços e feliz 2003 para todos!

Os editores.

Page 127: Eureka 2002

Sociedade Brasileira de Matemática

EUREKA! N°15, 2002

3

XLIII OLIMPÍADA INTERNACIONAL DE MATEMÁTICAEnunciados, Soluções e Resultado Brasileiro

A XLIII Olimpíada Internacional de Matemática foi realizada na cidade deGlasgow, Reino Unido no período de 18 a 31 de julho de 2002.

A equipe brasileira foi liderada pelos professores Edmilson Motta (SãoPaulo – SP) e Ralph Costa Teixeira (Niterói – RJ).

O Resultado da Equipe Brasileira

BRA 1 Alex Corrêa Abreu BronzeBRA 2 Larissa Cavalcante Queiroz de Lima PrataBRA 3 Guilherme Issao Camarinha Fujiwara BronzeBRA 4 Yuri Gomes Lima BronzeBRA 5 Davi Máximo Alexandrino Nogueira BronzeBRA 6 Thiago da Silva Sobral Bronze

PRIMEIRO DIADURAÇÃO: 4 horas e meia.

PROBLEMA 1Seja n um inteiro positivo. Seja T o conjunto de pontos (x; y) no plano onde x e y sãointeiros não negativos e x + y < n. Cada ponto de T é pintado de vermelho ou azul.Se um ponto (x; y) é vermelho, então todos os pontos (x'; y') com x' ≤ x e y' ≤ ytambém são. Um conjunto X é um conjunto de n pontos azuis com abcissas todasdistintas, e um conjunto Y é um conjunto de n pontos azuis com ordenadas todasdistintas. Prove que o número de conjuntos X é igual ao número de conjuntos Y.

SOLUÇÃO DE GUILHERME FUJIWARA (SÃO PAULO – SP)Primeiramente, seja x(i) o número de pontos azuis cuja ordenada é i, e y(i) o númerode pontos azuis de abscissa i.

Veja que o número de X-conjuntos é ∏−

=

1

0

)(n

i

iy , e o número de Y-conjuntos é∏−

=

1

0

)(n

i

ix .

Para provar que o número de X-conjuntos é igual ao número de Y-conjuntos, ésuficiente provar que os números x(0), x(1), x(2),...,x(n – 1) são uma permutação dosnúmeros y(0), y(1), y(2),..., y(n – 1). Provaremos este lema por indução no n.Se n = 1, temos que x(0) = y(0) = 1 ou 0 dependendo se (0; 0) é azul ou não.Suponhamos que o lema é verdadeiro para n < k, provaremos para n = k.Vamos olhar para a última diagonal de T (reta x + y = k – 1):Se nela não houver pontos vermelhos, então tome T' como os conjuntos de pontos de

Page 128: Eureka 2002

Sociedade Brasileira de Matemática

EUREKA! N°15, 2002

4

T que não estão na última diagonal. Temos que o lema vale para os x'(i) e y'(i) de T',

e como x(i) = x'(i) + 1 e y(i) = y'(i) + 1, então x'(a) = y'(b) ⇔ x(a) = y(b), e alémdisso x(k – 1) = y(k – 1) = 1, portanto o lema vale para T (vide fig. 1).

Y (ordenada)

X (abscissa)

Só pontos azuis na reta y + x = k – 1

Fig. 1

Se nela houver algum ponto vermelho, digamos (a; k – 1 – a), então aplicamos ahipótese de indução nos dois conjuntos T' formados acima e à direita de T, que sãomenores que T, e assim demonstramos o lema para T (vide fig. 2).Provamos então o nosso lema e, como já foi visto anteriormente, segue o que épedido no enunciado.

Y (ordenada)

X (abscissa)

2 conjuntos T' menores que T, noqual aplicamos a hipótese deindução.

Fig. 2

PROBLEMA 2

Page 129: Eureka 2002

Sociedade Brasileira de Matemática

EUREKA! N°15, 2002

5

Seja BC um diâmetro do círculo Γ de centro O. Seja A um ponto de Γ tal que

.1200 00 <∠< AOB Seja D o ponto médio do arco AB que não contém C. A reta quepassa por O e é paralela a DA encontra a reta AC em J. A mediatriz de OA corta Γem E e F. Prove que J é o incentro do triângulo CEF.

SOLUÇÃO DE YURI GOMES LIMA (FORTALEZA – CE)Como EF é perpendicular ao raio OA, os arcos EA e AF são iguais, donde ∠ ECA =∠ ACF ⇒ ∠ ECJ = ∠ JCF. Portanto, J já pertence à bissetriz de ∠ ECF. Semostrarmos então que AJ = AE, acabou (pois sabemos que o médio do arco EFeqüidista de E, de F e do incentro de CEF).Mas, se l é a paralela a DA por O, temos que l // DA e DO // AC ( pois ∠ DOB =∠ AOB/2 = ∠ ACB ) ⇒ DAJO paralelogramo ⇒ AJ = DO. Também, sendo EFmediatriz de OA, segue que EA = EO. Mas EO = raio de Γ = DO , donde AJ = EA =AF.

OBS: é necessário ∠ AOB < 120°, pois caso contrário teríamos ∠ AOC ≤ 60° e∠ BOD ≥ 60°, o que implicaria J = l ∩ AC fora de Γ, não podendo então ser este oincentro de CEF.

PROBLEMA 3Encontre todos os pares de inteiros m, n ≥ 3 tais que há infinitos inteiros positivos a

para os quais 1

12 −+−+

aa

aan

m

é inteiro.

SOLUÇÃO DE ALEX CORRÊA ABREU (NITERÓI – RJ)

Seja 1)( −+== xxxPP m e ][,1)( 2 xRTxxxQ n∈∃⇒−+= tq

),()()()( xRxQxTxP +⋅=

com deg QR deg< ou ⇒≡ 0R )()(

)(

)(

)(

)(

)()(

)(

)(aT

aQ

aP

aQ

aR

aQ

aRaT

aQ

aP −=⇒+= é inteiro

para infinitos a's mas se nR ∃⇒≠ 0 tq 1)(

)(0 <<⇒>

xQ

xRnx pois

deg 1)(

)(deg <⇒<

aQ

aRQR se ⇒> na existe apenas um número finito de a's tq

)(

)(

aQ

aR é inteiro =−+⋅⇒⇒⋅=⇒≡⇒ )()1()()()()()()()(0 xQxxPxQxPxQxQxTxPR

)1(11 1221 −+=+−−−++= −+−+ nmnmnnmm xxxxxxxx ; note que temos, nm ≥

Page 130: Eureka 2002

Sociedade Brasileira de Matemática

EUREKA! N°15, 2002

6

(pois ))(|)( xPxQ mas ⇒−+⇒=−= −+− 1)(1),1()),(( 12 nmnmnn xxxQxxmdcxxQmdc

por raciocínio análogo ao anterior, ).(121 Imnnnm +≤⇒≥+−Agora, como 1)0( −=Q , 101)1( <<∃⇒= αQ tq 0)()()(0)( ==⇒= αααα QTPQ

⇒−+==−+⇒ )(1012 IImn ααααde (I), ⇒≥ + )(12 IIImn αα por (II), ⇒−=⋅≥− )1()1( 22 ααααα m

ααααααα ≥+−⇒−≥+− 222 )1)(1()1()1()1( pois ⇒<1α3101 2323 ≤⇒=−≤⇒≤−+ nnααααα mas 33 =⇒≥ nn e a desigualdade só

não é estrita em (III) se 521 =⇒=+ mnm logo se existir uma solução será (5, 3),que de fato é uma solução, pois obviamente

( )( ) ).()1()(111 22325 xQxxxPxxxxxx ⋅+−=⇒−++−=−+

SEGUNDO DIADURAÇÃO: 4 horas e meia.

PROBLEMA 4Seja n inteiro maior que 1. Os divisores positivos de n são d1, d2,…,dk , onde

nddd k =<<<= ...1 21

Seja D = d1d2 + d2d3 +…+dk – 1 dk.(a) Prove que D < n2.(b) Encontre todos os valores de n para os quais D é um divisor de n2.

SOLUÇÃO DE THIAGO DA SILVA SOBRAL (FORTALEZA – CE)

Lema: ik

id

nd

−+

=1

Prova:Observando que se d é divisor de n então n/d também o é, podemos agrupar osdivisores aos pares, donde concluímos que ndd iki =−+1 , ki ≤≤0 . Esse fato

também vale para n sendo quadrado perfeito, pois ndd kk

k == +−+

+2

11

2

1

Corolário: ik

ndi −+

≤1

Page 131: Eureka 2002

Sociedade Brasileira de Matemática

EUREKA! N°15, 2002

7

Prova:De fato, sendo ikd −+1 o (k + 1 – i)-ésimo divisor de n, temos ikd ik −+≥−+ 11 , e

segue o resultado pelo lema.Pelo corolário,

221

1

2

1

1

213221

11

1

11

1

11

12211

nk

njj

n

jjn

nn

k

n

k

n

k

n

k

nddddddD

k

j

k

jkk

<

−=

+

=

=+

=+−−

+−

≤+++=

∑−

=

=−

b) Veja que se n é primo, nD ×=1 , e assim temos que 2| nD . Suponha ncomposto, e seja p o seu menor fator primo. Pelo lema,

p

nn

p

npddddddD kk

2

13221 1 >×++×=+++= −

Veja então que 22

nDp

n << , e como p

n2

é o maior divisor de 2n menor que 2n ,

concluímos que 2| nD /Por fim, concluímos que nnD ⇔2| é primo.

PROBLEMA 5Encontre todas as funções f de em tais que

)()())()())(()(( yzxtfztxyftfyfzfxf ++−=++

para todo x, y, z, t ∈ .

SOLUÇÃO DE LARISSA CAVALCANTE QUEIROZ DE LIMA (FORTALEZA – CE))()())()())(()(( yzxtfztxyftfyfzfxf ++−=++

)0()0(2)0(2))0(2())0(2(:0,,, 2 ffffftyzx =⋅⇒⋅=⋅⋅⋅←

2

1)0(1)0(20)0( =⇒=⋅⇒≠ fff

)0(2))0(2))(0()((:0,, fffxftyz ⋅=⋅+←

se 0)0( ≠f (ou seja )2

1)0( =f , ∈∀=−=⇒=+ xxffxf

2

1

2

11)(1)0()(

Page 132: Eureka 2002

Sociedade Brasileira de Matemática

EUREKA! N°15, 2002

8

⇒ Se ,0)0( ≠f então 2

1)0( =f e ∈∀= xxf ,

2

1)( (o que é uma solução).

Suponha então 0)0( =f

[ ] )()()()(; 222 yxfxyxyfyfxfxtyz ++−=+⇒←←

[ ] 22222 )()()(2)()()()( yfyfxfxfyxfyfxf ++=+=+⇒)()())(2))((2(; xyxyfxyxyfyfxfytxz ++−=⋅⋅⇒←←

)2()()(4 xyfyfxf =⋅⇒)00()0())0()())(0()((0, ++−=++⇒← fxyffyffxfzt

)()()( xyfyfxf =⋅⇒ f é multiplicativa.)2()(4 xyfxyf =⋅⇒

)()())()())(()((; tzxyfzyxtfyftfzfxfytty ++−=++⇒←←)()()()( ztxyfzyxtftzxyfzyxtf −++=++−⇒

Suponhamos que f não seja identicamente nula (note que f(x) ≡ 0 é uma solução).

Suponha 0)( <af e que ayxyx =+∃ 22:,

[ ] 0)()()()( 222 ≥+=+=⇒ yfxfyxfaf , contradição!Assim, 0. ,0)( ≥∀≥ aaf

)()())()())(()(( zyxtfztxyftfyfzfxfzz −++=+−+⇒−← ))()())(()(( yftfzfxf ++=

)()()()( zfxfzfxf +=−+⇒)()( zfzf =−

1)1()()1()( =−⇒⋅−=− fzffzf = f (1).)1()1(2)1)((:1,, ++−=⋅+← xfxfxftyz

)1()1(2)(2 ++−=+⇒ xfxfxf ..4)2()()2()2()(4* =⇒⋅==⋅ fxyffxyfxyf

* conjectura: 2)( xxf =222222 )()())(( yzxtztxytyzx ++−=++

2222222222222222 22 zyxtyztxxyzttzyxtzyztxyx +++−+=+++⇔Ok!! A função 2)( xxf = funciona!!!

)1()1(2)(2 ++−=+ xfxfxf )4)2( ;1)1(( == ff2)( mmf = ., ∈≤∀ mnm

)1()1(2)(2 ++−=+⋅ nfnfnf2222 )1(12)1(22)1( +=++=−−+⋅=+⇒ nnnnnnf

Page 133: Eureka 2002

Sociedade Brasileira de Matemática

EUREKA! N°15, 2002

9

∈∀=⇒ nnnf ,)( 2 .

como );()( nfnf =− temos ∈∀= zzzf ,)( 2 .

Tome ∈r ; 0,,;1),(, ≠∈== qqpqpmdcq

pr

22

2

)()(

)(

)(

)1()(

1)()( r

q

p

qf

pf

qf

fpf

qfpf

q

pfrf ===⋅=

⋅=

=

∈∀=⇒ rrrf ,)( 2 .

)()()(2))()(( 22 rrxfrrxfrfrfxf −⋅++⋅=⋅+

)()()(2)()(2 222 rxrfrrxfrfrfxf −++⋅=+

)()(2)(2 224 rxrfrxrfrxrf −++=+ ∈∀ x ,

.,),()(2)(2 224 ∈∈∀−++=+⇒∈ ryryfryfryfr

Suponha que ry,∃ tais que 0≥y , r > 0 e

)()( 2 yfryf <+

( )[ ] ( )22)( yfrfyf <+⇒

( ) ( )

( ) ,)()(22

0

2

00

2yfrfrfyfyf <+⋅+⇒

>>≥

Absurdo!!!

)()( 2 yfryf >+⇒ fy ⇒∈∀ é crescente em +.

Vamos mostrar que f é contínua em +: note que se y, r ≥ 0, f(y +r2) – f(y) =

),)(2())()(2)(()()()()(2)( 2222 ryfrrfyfrfyfrfrfyfyf +=+=−++⇒se ∈r . Assim, dado 0>ε , para ∈0r suficientemente pequeno, temos

.)()( 20 ε<−+ yfryf

e se 0rr < , temos ε<−+<−+ )()()()( 20

2 yfryfyfryf2

0 se ryx <−⇒ temos )()()( yxyfxf ><− ε .

* εε <−<−−+=−− 40

40

20

20 22)()()()( rryfryfryfyf

Se 0rr < então )()( 20

220

2 ryfryfryry −>−⇒−>−

ε<−−<−−⇒ )()()()( 20

2 ryfyfryfyf

⇒ para 20ryx <− temos ε<− )()( yfxf

portanto f é contínua.

Suponha 0,)( 2 ≠+= θθwwf

Page 134: Eureka 2002

Sociedade Brasileira de Matemática

EUREKA! N°15, 2002

10

1) 0>θTemos que 00 >∃ r tal que

θ<−⇒<− )()(20 wfxfrwx

ou seja θθ <−− 22 wx caso ∈x

mas se x < w temos 22 wx < e portanto 022 >− xw

( ) θθθθ >+−=−+=−−⇒ 222222 xwxwwx , contradição!!

(existe ∈x tal que 200 rxw <−< ).

2) θθ −=⇒< 2)(0 wwf

temos que 00 >∃ r tal que

θ<−⇒<− )()(20 wfxfrwx

ou seja θθ <+− 22 wx caso ∈x

Note que ∈∃ x tal que 200 rwx <−<

θθθ >+−=+−⇒>⇒>⇒ 222222 wxwxwxwx contradição!!

∈∀=⇒ xxxf ,)( 2 .As únicas funções são:

0)(;2

1)( ≡≡ xfxf ou ,)( 2xxf = ∈∀ x .

PROBLEMA 6Sejam nΓΓΓ ,...,, 21 círculos de raio 1 no plano, onde n ≥ 3. Seus centros são O1,O2,…,On, respectivamente.Suponha que não exista reta que intercepte mais que dois dos círculos. Prove que

∑≤<≤

−≤nji ji

n

OO1

.4

)1(1 π

SOLUÇÃO DE LUCIANO GUIMARÃES CASTRO (RIO DE JANEIRO – RJ)Seja αij a medida, em radianos, do ângulo agudo formado pela reta OiOj com umareta tangente a Γ j passando por Oi. As circunferências têm raio 1, e ijij α≤αsen ,

, ,2 ,1, nji ∈∀ . Assim,

∑ ∑∑≤<≤ ≤<≤≤<≤

≤=nji nji

ijijnji jiOO 1 11

sen1 αα .

Page 135: Eureka 2002

Sociedade Brasileira de Matemática

EUREKA! N°15, 2002

11

É suficiente, portanto, provar que 4

)1(

1

πα −≤∑≤<≤

n

njiij , o que parece menos assustador

que o enunciado original.

Vejamos de que maneira a condição de nenhuma reta cortar mais de duascircunferências limita a soma dos αij. Fixemos i. Para cada j, a união de todas asretas que passam por Oi e cortam a circunferência Γ j forma dois ângulos opostos pelovértice Oi, cada um medindo 2αij. Como essas retas não cortam outra circunferênciaalém de Γ i e Γ j, variando j obtemos ângulos disjuntos com vértice Oi, de forma que asoma de suas medidas não pode ultrapassar 2π, ou seja,

πα 241

≤∑≠

≤≤ij

njij . (1)

O i

2α i2

i1

ij2α

Γj

Γ2

Γ1

Agora, somando estas desigualdades para , ,2 ,1 ni ∈ e observando que

jiij α=α , obtemos

∑∑∑≤≤

≠≤≤≤≤

≤ni

ijnj

ijni 111

24 πα ⇔ πα nnji

ij 2421

≤∑≤<≤

⇔ 41

πα n

njiij ≤∑

≤<≤

.

Page 136: Eureka 2002

Sociedade Brasileira de Matemática

EUREKA! N°15, 2002

12

Hummmm... quase! De fato, este resultado já é assintoticamente equivalente aodesejado. Apesar de que conseguir um n − 1 no lugar daquele n é a parte difícil desteproblema, você vai perceber que se trata apenas de ir adaptando esta primeira idéia.

Intuitivamente, o que acabamos de fazer foi “girar” uma reta 180o em torno de cadaponto Oi, sabendo que neste percurso ela cortará todas as outras circunferências, masnunca duas ao mesmo tempo. Para melhorar a estimativa, precisamos encontrar umaforma de girar menos que 180o e, ainda assim, encontrar todas as demaiscircunferências. Isto não é possível para todos os Oi, mas podemos fazê-lo com os“mais afastados”.

Mais precisamente, nossa idéia é trabalhar com o fecho convexo do conjunto O1,O2, ..., On, ou seja, o menor conjunto convexo que contém O1, O2, ..., On. Semperda de generalidade, podemos supor que esse fecho convexo é o polígonoO1O2O3... Om )( nm ≤ . Desta forma, os pontos Om+1, Om+2, ..., On são interiores aopolígono.

Vamos separar a soma ∑≤<≤ nji

ij1

α em quatro partes:

mmi

iiL 111

1, αα += ∑−≤≤

+ (soma dos αij tais que OiOj é lado do polígono);

LDmji

ij −= ∑≤<≤1

α (soma dos αij tais que OiOj é diagonal do polígono);

∑≤<

≤≤

=

njmmi

ijT1

α (soma dos αij tais que Oi é vértice do polígono e Oj é interior);

∑≤<<

=njim

ijI α (soma dos αij tais que Oi e Oj são interiores ao polígono).

Observe que ∑≤<≤

=+++nji

ijITDL1

α .

Vamos usar os ângulos externos do polígono para limitar L. Sejam ai e ei,respectivamente, as medidas dos ângulos interno e externo do polígono no vértice Oi.Para simplificar a notação, trabalharemos com o vértice O2. Seja t a tangente comumexterior a Γ1 e Γ3, mais próxima de Γ2. Então Γ2 está totalmente contida nosemiplano determinado por t que não contém Γ1 e Γ3 (caso contrário existiria umareta cortando Γ1, Γ2 e Γ3). Sejam r a paralela a t por O2 e P1 o ponto de r tal que∠ P1O2O1 é agudo e sua bissetriz é perpendicular à reta O1P1. Então a distância de O1

a essa bissetriz é igual a 2

11PO. Esta distância é maior que 1, pois P1 pertence a r e a

Page 137: Eureka 2002

Sociedade Brasileira de Matemática

EUREKA! N°15, 2002

13

distância de O1 a r é maior que 2. Isto significa que a bissetriz de ∠ P1O2O1 é exterior

a Γ1, do que concluímos que 2

)( 12112

OOPm ∠<α . Procedendo de forma análoga com

a circunferência Γ3 e somando as duas desigualdades, temos

2222

2312

ea =−

<+ παα .

O 1 O 3

O 2

t

r

α12

P 1

Fazendo o mesmo para os demais vértices do polígono e somando as desigualdadesobtidas, concluímos que

∑≤≤

<mi

ieL

1 22 ⇔ 2L < π. (2)

Agora utilizaremos os ângulos internos do polígono em um procedimento parecidoao que fizemos para descobrir a desigualdade (1). Salvo Γ1, Γ2 e Γ3, todas as demaiscircunferências estão completamente contidas no ângulo interno Ô2 = ∠ O1O2O3.Para cada j ≠ 2, considere o conjunto união das semi-retas com origem O2, interioresa Ô2, que cortam a circunferência Γ j. Este conjunto forma um ângulo de medida α2j,para j = 1 e j = 3, e um ângulo de medida 2α2j para os demais valores de j. Comocada semi-reta pode cortar apenas uma circunferência além de Γ2, os conjuntoscorrespondentes a distintos valores de j são disjuntos. Assim,

24

22312 2 ani

j ≤++ ∑≤≤

ααα .

Procedendo de forma análoga com os demais vértices do polígono e somando asdesigualdades obtidas concluímos que

π)2(2421

−=≤++ ∑≤≤

maTDLmi

i . (3)

Page 138: Eureka 2002

Sociedade Brasileira de Matemática

EUREKA! N°15, 2002

14

Para concluir, seja Oi um ponto interior ao polígono, ou seja, i > m. Já provamos a

desigualdade (1): πα 241

≤∑≠

≤≤ij

njij .

Somando essas desigualdades para todos os pontos Oi interiores temos

∑∑∑≤<

≠≤≤≤<

≤nim

ijnj

ijnim

πα 241

⇔ π2)(84 mnIT −≤+ ⇔ π)(42 mnIT −≤+ . (4)

Agora basta somar as desigualdades (2), (3) e (4), obtendoπππ )()2(422422 mnmITTDLL −+−+≤+++++

⇔ π)1(4444 −≤+++ nITDL ⇔ 4

)1( π−≤+++ nITDL .

Portanto

4

)1(sen

1

1 11

παα −≤+++=≤= ∑ ∑∑≤<≤ ≤<≤≤<≤

nITDL

OO nji njiijij

nji ji

,

como queríamos demonstrar.

Page 139: Eureka 2002

Sociedade Brasileira de Matemática

EUREKA! N°15, 2002

15

XVII OLIMPÍADA IBEROAMERICANA DE MATEMÁTICAEnunciados, Soluções e Resultado Brasileiro

A XVII Olimpíada Iberoamericana de Matemática foi realizada na cidade deSão Salvador, El Salvador no período de 30 de setembro a 5 de outubro de 2002.

A equipe brasileira foi liderada pelos professores Eduardo Wagner (Rio deJaneiro – RJ) e Onofre da Silva Farias (Fortaleza – CE).

O Resultado da Equipe Brasileira

BRA 1 Guilherme Camarinha Fujiwara OuroBRA 2 Humberto Silva Naves OuroBRA 3 Larissa Cavalcante Queiroz de Lima OuroBRA 4 Yuri Gomes Lima Prata

PRIMEIRO DIADURAÇÃO: 4 horas e meia.

PROBLEMA 1Os números inteiros desde 1 até 2002, ambos incluídos, escrevem-se num quadro porordem crescente 1, 2,..., 2001, 2002. Em seguida apagam-se os que ocupam oprimeiro lugar, quarto lugar, sétimo lugar, etc, ou seja, os que ocupam os lugares daforma 3k + 1.Na nova lista apagam-se os números que estão nos lugares da forma 3k + 1. Repete-se este processo até que se apagam todos os números da lista. Qual foi o últimonúmero que se apagou?

SOLUÇÃO DE GUILHERME CAMARINHA FUJIWARA (SÃO PAULO - SP)Considere uma sequência infinita ao invés de uma sequência até 2002. Seja entãoR(k) o primeiro número a ser apagado na k-ésima seção de apagamento.Temos então R(1) = 1, R(2) = 2, R(3) = 3, R(4) = 5, R(5) = 8, R(6) = 12.Queremos então achar o maior R(k) menor ou igual à 2002.

Vamos provar que R(k + 1) =

⋅ )(

2

3kR (onde k é o menor inteiro maior ou igual

a x).Para tal, basta ver que se considerarmos a seqüência que sobra após a primeira sériede apagamentos, teremos que o R(k)-ésimo termo será o primeiro a ser apagado nak-ésima próxima série de apagamento. Considerando também o primeiroapagamento, temos que o R(k)-ésimo número dessa lista sérá o primeiro a serapagado na (k + 1)-ésima série de apagamento, logo será o R(k + 1).

Page 140: Eureka 2002

Sociedade Brasileira de Matemática

EUREKA! N°15, 2002

16

É fácil ver que o n-ésimo termo da sequência que sobra após a primeira série de

apagamentos será o

⋅ n2

3(basta ver os dois casos de paridade de n), logo temos

que R(k +1) =

⋅ )(

2

3kR .

Fazendo as contas, temos então R(6) = 12, R(7) = 18, R(8) = 27, R(9) = 41, R(10) =62, R(11) = 93, R(12) = 140, R(13) = 210, R(14) = 315, R(15) = 473, R(16) = 710,R(17) = 1065, R(18) = 1598 e finalmente R(19) = 2397.Como R(18) = 1598 e 2002 < R(19) = 2397, então o último número apagado foi1598.

PROBLEMA 2Dado qualquer conjunto de 9 pontos no plano entre os quais não existem trêscolineares, demonstre que para cada ponto P do conjunto, o número de triângulosque têm como vértices três dos oito pontos restantes e P no seu interior é par.

SOLUÇÃO DE HUMBERTO SILVA NAVES (SÃO JOSÉ DOS CAMPOS - SP)Seja S o conjunto dos 9 pontos.

Se um ponto P está no interior do triângulo ABC com A, B, C e P ∈ S, temos:

A

P

A' B

β C

C' B'

α

γ

Sejam A’ = AP ∩ BC ; B’ = BP ∩ ΑC e C’ = CP ∩ ΑB. As semi-retas PA’, PB’ ePC’ dividem o plano em três regiões: α, β e γ (A ∈ α, Β ∈ β e C ∈ γ).Vamos construir um grafo G cujos vértices representam os triângulos (com vérticesem S) com o ponto P em seu interior. Ligaremos 2 vértices deste grafo se e somentese os triângulos correspondentes tiverem um lado em comum. Vamos agora provarque o grau de cada vértice é 5.

Seja Q ∈ S – A ; B ; C ; P. Temos 3 possibilidades:

Page 141: Eureka 2002

Sociedade Brasileira de Matemática

EUREKA! N°15, 2002

17

1) Se Q ∈ α: Temos que o ponto P está no triângulo QBC, logo ∆QBC ∈ V(G).Como os triângulos ∆ΑBC e ∆QBC tem um lado em comum, eles estão ligados poruma aresta em G. Obs.: Claro que P não está no ∆QAB e nem no ∆QAC.2) Se Q ∈ β: ∆QAC ∈ V(G) e está ligado à ∆ABC em G.3) Se Q ∈ γ : ∆QAB ∈ V(G) e está ligado à ∆ABC em G.

Logo deg (∆ABC) = #(S – A ; B ; C ; P) = 5. E como Σ deg(∆) = 2 × #E(G)⇒ #V(G) é par. Portanto o número de triângulos com P em seu interior é par.

PROBLEMA 3Um ponto P é interior ao triângulo equilátero ABC e é tal que °=∠ 120APC . SejamM a intersecção de CP com AB e N a intersecção de AP com BC. Encontrar o lugargeométrico do circuncentro do triângulo MBN quando P varia.

SOLUÇÃO DE YURI GOMES LIMA (FORTALEZA – CE)

B

X Y N O1

O P

A C

Ω

M

Γ

Vamos mostrar que tal L.G. está contido na reta mediatriz do raio BO , onde O é o

centro de ABC. Para isso, seja O1 o circuncentro de BMN. Daí, como °=120ˆCPA ,

Page 142: Eureka 2002

Sociedade Brasileira de Matemática

EUREKA! N°15, 2002

18

temos BNPMMPN ⇒°=120ˆ inscritível. Para O1 pertencer a , devemos ter

11 OOBO = , ou seja, O também deve pertencer à circunferência Γ circunscrita em

BMPN . Vamos mostrar que O é o médio do arco MN .

Agora, veja que, como °=120ˆCPA , temos então que MCACANNAB ˆˆ60ˆ =−°= .Assim, .CAMABN ∆≡∆ Logo, a rotação com centro em O e ângulo °120 que leva Bem A e A em C também leva N em M.

Assim, °=120ˆMON e MONO = . Mas sendo °=120ˆMON , segue que BMON éinscritível. Assim, provamos o que queríamos e .1 ∈O

Mas o LG não é a reta toda. De fato, devemos ter °< 90ˆ1 NBO , pois BNO1∆ é

isósceles, daí, se X e Y são os pontos de interseção de com a circunferência

Ω circunscrita a ABC, teremos que O1 pertence ao INTERIOR do segmento XY ,

pois °== 90ˆˆ ABYCBX (para ver isto, observe que XBYO é losango com

⇒°=⇒= 60ˆBOXBOXO X é o médio do arco AB).

Agora, dado O1 pertencente ao interior do segmento XY , trace a circunferência de

centro O1 e raio BO1 (que passa por O).

Ela determinará dois pontos M e N sobre AB , BC tais que

MNOMBONBONMO ˆˆˆˆ === e °=120ˆNOM . Daí, a mesma rotação considerada

antes levará N em M, levando então o ABN∆ no CMANCAM =⇒∆ e

°=+⇒−°=−°⇒= 60ˆˆˆ60ˆ60ˆˆ PCACAPMCANABMCANAB , onde CMANP ∩= ,

donde .120ˆ °=CPA

Logo, o L.G. procurado é o interior do segmento .XY

SEGUNDO DIADURAÇÃO: 4 horas e meia.

PROBLEMA 4Num triângulo escaleno ABC traça-se a bissectriz interna BD, com D sobre AC.Sejam E e F, respectivamente, os pés das perpendiculares traçadas desde A e C até àrecta BD, e seja M o ponto sobre o lado BC tal que DM é perpendicular a BC.Demonstre que .DMFEMD ∠=∠

Page 143: Eureka 2002

Sociedade Brasileira de Matemática

EUREKA! N°15, 2002

19

SOLUÇÃO DE YURI GOMES LIMA (FORTALEZA – CE) B

A

E

M A'

D C

F

Seja A' a interseção de AE com BC . Então, como BE é bissetriz, segue que

'EAAE = . Mas então os triângulos ADE e A'DE são congruentes, donde

EADEAD 'ˆˆ = (I). Como AE e CF são perpendiculares a BD , então

FCDDAECFAE ˆˆ// =⇒ (II)Também:

⇒°== 90ˆˆ CMDCFD o quadrilátero MCFD é inscritível .ˆˆ FMDFCD =⇒⇒°== 90ˆ'ˆ' DEADMA o quadrilátero A'MED é inscritível .'ˆˆ DAEDME =⇒

Mas, por (I) e (II), temos que ⇒= FCDDAE ˆ'ˆ

FMDDME ˆˆ =⇒ .

Obs. Por ABC ser escaleno, temos que BD não é perpendicular a AC , i.e,.EFDE ≠≠≠

PROBLEMA 5A sucessão de números reais ,..., 21 aa define-se como:

561 =a e n

nna

aa1

1 −=+ para cada inteiro .1≥n

Demonstre que existe um inteiro ,20021, ≤≤ k k tal que .0<ka

Page 144: Eureka 2002

Sociedade Brasileira de Matemática

EUREKA! N°15, 2002

20

SOLUÇÃO DE LARISSA CAVALCANTE QUEIROZ DE LIMA (FORTALEZA – CE)Lema: 0,; >< kk amma

mmak

11 −<⇒ +

Prova: maam

makk

k

1111 −<−⇒<⇒<

mma

mm

aa k

kk

1111 −<∴−<−⇒ +

*2

2211

112

1

kkkkk

kkk

aaaaa

aaa +⋅−=⇒−= ++

21

2

221 −+=⇒ +

k

kka

aa

Soma telescópica

* 21

21

21

22 −+=

aaa

21

22

22

23 −+=

aaa

(+) 21

2

221 −+=+

k

kka

aa

kaaa

aak

k ⋅−++++=⇒ + 21

...11

222

21

21

21

1) Suponha que ∃ 1999≤i tal que )0(2 >< ii aa

Isso implica: 2

3

2

121 =−<+ia . Caso 01 >+ia , .1

6

5

6

49

3

2

2

32 <=−=−<+ia

Se 2002319993;01

110 32 =+≤+=−<⇒> ++ iaa ii

20021, ≤≤∃⇒ kk tal que .0<ka

2) Suponha que ∃/ 1999≤i tal que 2<ia

2

1

4

11

2

119991 2

iii

aaia ≥⇒≥⇒≤≤∀≥⇒

Page 145: Eureka 2002

Sociedade Brasileira de Matemática

EUREKA! N°15, 2002

21

199924

1199956)1999(2

1...

156 2

21999

21

222000 ⋅−⋅+≤−+++=⇒

aaa

04

735003136

4

7)12000(3136

4

1219995622

2000 <+−=−−=

−−≤⇒ a

Absurdo!Portanto 1999 ≤∃ i tal que 2<ia

⇒ temos que 21 ,, ++ iii aaa ou 3+ia é menor que zero (por 1)

20021, ≤≤∃∴ kk tal que .0<ka

PROBLEMA 6Um polícia tenta capturar um ladrão num tabuleiro de .20012001× Eles jogamalternadamente. Cada jogador, na sua vez, deve mover-se uma casa num dos trêsseguintes sentidos:

(abaixo); (direita); (diagonal superior esquerda).

Se o polícia se encontra na casa da esquina inferior direita, pode usar a sua jogadapara passar directamente para a casa da esquina superior esquerda (o ladrão não podefazer esta jogada). Inicialmente o polícia está na casa central e o ladrão está na casavizinha diagonal superior direita do polícia. O polícia começa o jogo.Demonstre que:a) O ladrão consegue mover-se pelo menos 10000 vezes sem ser capturado.b) O polícia possui uma estratégia para capturar o ladrão.Nota: O polícia captura o ladrão quando entra na casa em que está o ladrão. Se oladrão entra na casa do polícia, não há captura.

SOLUÇÃO OFICIAL: Pinte o tabuleiro com 3 cores da seguinte forma:Figura1:

a

a a

a b b

b

c c

c c b a

c

c c a b

1 2 3 4

1 2 3 2001

1001

2001 c

c b a

a b

b a b

… …

c c

Movimentos

Page 146: Eureka 2002

Sociedade Brasileira de Matemática

EUREKA! N°15, 2002

22

Observe que os movimentos nos dão o seguinte ciclo:

a b

c

Quer dizer: de uma casa a só se vai para b, de uma bsó se vai para c e de uma c só se vai para a.

Inicialmente o polícia começa numa casa c e o ladrão também.(casas hachuradas na figura)Assim teremos as seqüênciasPolícia: c → a → b → c O polícia nunca poderia entrar numa casa de mesma cor do ladrão.Ladrão: c → a → b → c Para sua sorte existe o túnel.Se pensarmos um pouco, veremos que o polícia deve atravessar o túnel 2 vezes parapoder tornar compatível seu ciclo com o do ladrão, ou seja, jogar e cair numa casa demesma cor do ladrão (podendo pegá-lo).Seja X = casa superior esquerda

Y = casa inferior direita.Logo o polícia precisa de 2000 movimentos para chegar até Y, cruza o túnel (1movimento), mais 4000 para chegar de novo até Y, cruza o túnel (1 movimento).Neste momento o ladrão deve estar próximo de Y e o polícia precisará de mais 3999movimentos (pelo menos) para capturar o ladrão (que ficará rodando no quadrado 2× 2 inferior esquerdo, totalizando2000 + 4000 + 3999 + 2 = 10001 movimentos do polícia, ou seja, 10000movimentos do ladrão.b) Vejamos agora uma estratégia para que o polícia prenda o ladrão. Suponha que ele já tenha passado 2 vezes pelo túnel.Numere as linhas do tabuleiro de 1 a 2001, de cima para baixo e as colunas de 1 a2001, da esquerda para a direita.Após sair do túnel, o polícia se encontra na casa da linha 1 e coluna 1.A estratégia é a seguinte:

i) O polícia deve se mover para a direita até que o ladrão fique na mesmadiagonal (inferior direita) do polícia ou uma casa à direita.

ii) Em seguida, o polícia deve fazer o seguinte movimento:

Page 147: Eureka 2002

Sociedade Brasileira de Matemática

EUREKA! N°15, 2002

23

Ladrão Joga Polícia Joga

iii) O polícia deve repetir os passos i) e ii) Note que a diferença entre osnúmeros das linhas do polícia e do ladrão sempre diminui e o ladrão semprefica na região à direita da diagonal inferior direita do polícia.

Assim, após repetirmos i) e ii) um número finito de vezes, chegamos na situação (navez do ladrão)

PolíciaLadrão

(note que não pode ocorrer outra situação na qual a diferença entre linhas é 1, pois,na vez do ladrão, os dois devem estar em casas do mesmo tipo).Seguindo o que é feito em ii), o ladrão, em algum momento, se move para a casa àdireita do polícia. Assim, na próxima jogada, o polícia prende o ladrão.

Page 148: Eureka 2002

Sociedade Brasileira de Matemática

EUREKA! N°15, 2002

24

A FÓRMULA DE CARDANO ALÉM DAS CÚBICASJosé Cloves Verde Saraiva, São Luis – MA

♦ Nível Avançado.

INTRODUÇÃO:Motivado pela leitura do trabalho Equação do Terceiro Grau do Professor Alberto deAzevedo [1], ocorreu-me a curiosidade de saber as fórmulas das raízes calculadaspor radicais de uma equação polinomial do 5º grau, solúvel, que não fosse a trivial

05 =+ ax , que todos conhecem. Daí então, seguindo os mesmos passos da deduçãoda fórmula de Cardano para as equações polinomiais cúbicas foi possível provar que

a 05

1 235 =−+− rxppxx , já estudada por DE MOIVRE, tem raízes dadas por uma

fórmula análoga a de Cardano. Além desta, outras fórmulas semelhantes sãopossíveis deduzir para graus maiores que o quinto.Deixamos para o leitor essa generalização!.

A FÓRMULA DE CARDANO:É fascinante toda a história da resolução das equações polinomiais do 3º grau. Emresumo a referência [2] apresenta o seguinte:"O descobridor do método foi Scipione del Ferro (1465 - ≈ 1562), matemáticoitaliano, que antes de morrer o revelou aos discípulos Antônio Maria Fior e AnnibaleDella Nave"."Houve uma disputa matemática entre Fior contra Niccolo Fontana (1500 – 1557),conhecido pelo apelido de Tartaglia (gago, em italiano). A vitória deste último,muito divulgada, foi do conhecimento do médico e professor Girolano Cardano(1501 – 1576) que conseguiu lhe atrair para ensinar a regra de resolução sob ojuramento de jamais publicá-la. Cardano procurou a demonstração da regra - e achou- e ainda motivou seu discípulo Ludovico Ferrari (1522 – 1565) a descobrir soluçãopara as equações do quarto grau.""Cardano, numa visita a Della Nave, soube do manuscrito de Del Ferro contendo aregra de Tartaglia que já existia há 30 anos. Motivo que o levou quebrar o juramento.Publicou os métodos no seu famoso livro Ars Magna, em 1545, onde não deixou defazer referência aos descobridores, embora a contragosto de Tartaglia que seconsiderou traído."

Podemos representar a equação geral do 3º grau na forma 0322

13 =+++ axaxax e

por uma mudança de variável 31a

yx −= a equação fica mais simples na forma

Page 149: Eureka 2002

Sociedade Brasileira de Matemática

EUREKA! N°15, 2002

25

.03 =−− qpyy Calculando o cubo de um binômio 32233 33)( vuvvuuvu +++=+ , epondo em evidência 3uv, temos:

(*) )()(3)( 333 vuvuuvvu +++=+ou melhor,

0)()(3)( 333 =+−+−+ vuvuuvvuisto é, y = u + v é uma raiz para valores de p = 3uv e q = u3 + v3, onde podemos

elevar ao cubo a primeira e ter 333

27vu

p = e 33 vuq += e cair num problema onde

u3 e v3 são as raízes de uma equação do 2º grau conhecendo a soma e o produto dasraízes, cuja solução é conhecida:

2742

323 pqq

u −+= e 2742

323 pqq

v −−=

donde obtemos a famosa fórmula de Cardano:

332

332

27422742

pqqpqqvuy −−+−+=+=

A FÓRMULA DE CARDANO ALÉM DAS CÚBICAS:O resultado principal destas notas foi motivado por uma analogia da dedução na

fórmula de Cardano. Vamos provar que: a equação 05

1 235 =−+− rxppxx tem

uma de suas raízes dada pela fórmula:

552

552

312542312542

prrprrvux +−+++=+=

DEMONSTRAÇÃO:

Considere 035 =−−− rqxpxx . Calculemos polinômios do binômio:543223455 510105)( vuvvuvuvuuvu +++++=+

pondo em evidência obtemos:

)()(10)(5)( 5522335 vuvuvuvuuvvu +++++=+da igualdade (*) obtemos que:

)(3)()( 333 vuuvvuvu +−+=+para substituir no desenvolvimento donde obtemos que:

)()(10)(15)(5)( 55222235 vuvuvuvuvuvuuvvu +++++−+=+ ,isto é,

Page 150: Eureka 2002

Sociedade Brasileira de Matemática

EUREKA! N°15, 2002

26

)()(5)(5)( 552235 vuvuvuvuuvvu +++−+=+o que permite obter as igualdades: p = 5uv; q = –5u2v2 e r = u5 + v5.

Estabelecemos ,25 222 vup = logo temos que 5

2pq −= faz com que a equação seja

da forma ,05

1 235 =−+− rxppxx se x = u + v for uma raiz.

Verificar as relações

=+=

uvp

vur

5

55

nos leva ao já estudado na dedução da fórmula de

Cardano 55 vur += e 555

5vu

p =

, da mesma forma as raízes são:

5

525

542

prru −+= e

5

525

542

prrv −−=

de onde concluímos que a raiz x = u + v é dada pela fórmula:

552

552

312542312542

prrprrx −−+−+= .

OBSERVAÇÕES FINAIS:Esta fórmula torna mais fácil a determinação das raízes do que a indicada por DeMoivre estudada na referência [3], onde uma análise completa das raízes e o estudodos Grupos de Galois nos diversos casos é feito.

Como exercício estude a sétima 0357 =++−+ srxqxpxx e generalize.Finalizando, seria interessante o leitor paciente calcular todas as raízes da equaçãoabaixo estudada por Adriaan van Roomen (1561 – 1615) por polinômiostrigonométricos.

−+−+−+− 33353739414345 37645657402591111501230094545 xxxxxxx

+−+−+− 2325272931 3786588002360306521176791004695570014945040 xxxxx

−+−+−+ 1315171921 105306075232676280384942375488494125483841800 xxxxx

22453795956341138500781137534512075 357911 +=+−+−+− xxxxxx(Ver referência [4], pp 154).

REFERÊNCIAS:[1] Alberto de Azevedo, Equação do 3º grau, Depto. Matemática, UNB, 2002.[2] César Polcino Milies, A Resolução das equações de terceiro e quarto graus, Notas de aula, IME-USP, 2000.[3] R.L. Borger, On De Moivre' s Quintic, American Math. Monthly, pp. 171 - 174, vol. 15, 1908.[4] Paulo A. Martin, Introdução à Teoria dos Grupos e a Teoria de Galois, IME-USP, 1996.

Page 151: Eureka 2002

Sociedade Brasileira de Matemática

EUREKA! N°15, 2002

27

RECIPROCIDADE QUADRÁTICACarlos Gustavo T. de A. Moreira & Nicolau C. Saldanha, Rio de Janeiro - RJ

♦ Nível Avançado.

A lei de Gauss de reciprocidade quadrática afirma que se p e q são primos háuma relação direta entre p ser quadrado módulo q e q ser quadrado módulo p. Esteteorema fornece um rápido algoritmo para determinar se a é quadrado módulo ponde a é um inteiro e p um número primo. Lembramos que a é quadrado módulo n

se existe x ∈ com ).(mod2 nax ≡ Este artigo foi adaptado de [3].

Definição: Seja p um primo e a um inteiro. Definimos o símbolo de Lagrange

p

a

por

−=

. módulo quadrado é e divide não se 1

módulo quadrado é não se 1

divide se 0

paap

pa

ap

p

a

Proposição: Seja p um primo ímpar e a ∈ tal que p não divide a.

Então 2

1−

p

ap

a(mod p).

Demonstração: Sabemos que se p não divide a então )(mod11 pa p ≡− , ou seja,

11 −−px tem como raízes 1, 2,…, p – 1 em /p. Por outro lado,

.111 2

1

2

11

+

−=−

−−−

ppp xxx Se existe b ∈ tal que )(mod2 pba ≡ então

);(mod112

1

pba pp

≡≡ −−

ou seja, ).(mod1 2

1

pap

ap−

≡=

Como )(mod)(mod22 pyxpyx ±≡⇔≡ , há pelo menos 2

1−p quadrados em

(/p)*, logo os quadrados são exatamente as raízes de 12

1

−−p

x em /p, donde os

não quadrados são exatamente as raízes de 12

1

+−p

x , ou seja, se 1−=

p

bentão

).(mod12

1

pbp

−≡−

Page 152: Eureka 2002

Sociedade Brasileira de Matemática

EUREKA! N°15, 2002

28

Corolário: Se p é primo ímpar então ( ) .11 2

1−

−=

−p

p

Vamos agora reinterpretar a proposição. Seja ( ) .*/ pa ∈ Para cada

2

1,...,2,1

−= pj escrevemos a ⋅ j como jj mε com 1,1−∈jε e

−∈

2

1,...,2,1

pm j .

Se ji mm = temos jaia ⋅=⋅ ou ;jaia ⋅−=⋅ a primeira possibilidade implica

ji = e a segunda é impossível. Assim, se ji ≠ temos ji mm ≠ donde

.2

1,...,2,1;...;;

2

121

−=

−p

mmm p Assim,

2

1−

p

ap

a=

( )( )≡

−⋅⋅⋅

−⋅⋅⋅

21

...21

2

1...21

p

paaa

2

1...21

......2

121

2

121

−⋅⋅⋅

⋅⋅⋅≡

−−

p

mmm ppεεε)(mod...

2

121 pp−= εεε

donde ,...2

121 −=

pp

a εεε pois ambos pertencem a – 1, 1. Assim, ( )m

p

a1−=

,

onde m é o número de elementos j de

2

1,...,2,1

p tais que .1−=jε Como

primeira conseqüência deste fato temos o seguinte resultado.

Proposição: Se p é um primo ímpar então

( )

±≡−±≡

=−=

8). (mod 3 se ,1

8), (mod 1 se ,11

28

12

p

p

p

p

Demonstração: Se )4(mod1≡p , digamos ,14 += kp temos .22

1k

p =− Como

2

121

−≤≤ pj para kj ≤ e 12

2

1 −≤<−pj

p para kjk 21 ≤≤+ , temos

( )

≡−≡

=−=

).8(mod 5 se ,1

),8(mod 1 se ,11

p

p

p

a k

Page 153: Eureka 2002

Sociedade Brasileira de Matemática

EUREKA! N°15, 2002

29

Se )4(mod3≡p , digamos ,34 += kp temos .122

1 +=−k

p Para kj ≤≤1 temos

2

121

−≤≤ pj e para 121 +≤≤+ kjk temos 12

2

1 −≤<−pj

p, donde

( )

≡≡−

=−=

+

).8(mod 7 se ,1

),8(mod 3 se ,11 1

p

p

p

a k

Teorema: (Lei de reciprocidade quadrática) Sejam p e q primos ímpares.

Então ( ) .1 4/)1)(1(

−=

−−

p

q

q

p qp

Demonstração: Na notação acima, com a = q, para cada j ∈ P, onde,2/)1(,...,2,1 −= pP

temos que 1−=jε se e só se existe y ∈ tal que .02/)1( <−≤−− pyqjp Tal y

deve pertencer a Q, onde .2/)1(,...,2,1 −= qQ

Assim, temos que m

p

q)1(−=

onde Xm = e

02/)1(),( <−≤−−×∈= pyqxpQPyxX ;

note que qx – py nunca assume o valor 0. Analogamente, ( )n

q

p1−=

, onde Yn =

e .2/)1(0),( −≤−<×∈= qpyqxQPyxY

Daí segue que k

p

q

q

p)1(−=

onde Znmk =+= onde

2/)1(2/)1(),( −≤−≤−−×∈= qpyqxpQPyxZ

pois qx – py nunca assume o valor 0. Temos BACk −−= onde ,QPC ×=

,2/)1(),( −−<−∈= ppyqxCyxA

.2/)1(),( −>−∈= qpyqxCyxB

Como 4/)1)(1( −−= qpC , basta mostrar que .BA = Mas CCf →: definida

por ))2/)1((,)2/)1(((),( yqxpyxf −+−+= define uma bijeção entre A e B

Page 154: Eureka 2002

Sociedade Brasileira de Matemática

EUREKA! N°15, 2002

30

Exemplo: Se 1≥n e 122 +=n

p é primo, então 3 não é quadrado módulo p (e logo 3é raiz primitiva módulo p; ver [1]).

De fato, como

=

3

3),4(mod1

p

pp , mas )3(mod122 ≡

n

, como pode ser

facilmente mostrado por indução, donde )3(mod2122 ≡+=n

p , e .13

2

3−=

=

p

REFERÊNCIAS:

[1] Carlos Gustavo T. de A. Moreira, Divisibilidade, Congruências e Aritmética módulo n, Eureka! No.2, pp. 41-52, 1998.[2] Guilherme Camarinha Fujiwara, Inteiros de Gauss e Inteiros de Eisenstein, Eureka! No. 14, pp. 23-31, 2002.[3] Carlos Gustavo T. de A. Moreira e Nicolau C. Saldanha, Primos de Mersenne (e outros primosmuito grandes), 22o. Colóquio Brasileiro de Matemática, IMPA, 1999.

Page 155: Eureka 2002

Sociedade Brasileira de Matemática

EUREKA! N°15, 2002

31

APLICAÇÕES DE PLANOS PROJETIVOSEM TEORIA DOS NÚMEROS E COMBINATÓRIA

Carlos Yuzo Shine, São Paulo - SP

♦ Nível Avançado.

1. Definição de plano projetivoA definição dada aqui é a mais geral. Dizemos que um conjunto S é um planoprojetivo se existem subconjuntos ,..., 21 de S que satisfazem as seguintespropriedades:(i) Se P e Q pertencem a S, um e somente um dos subconjuntos i contém P e

Q.(ii) A interseção de i e j consiste sempre de um único elemento, para todo

i ≠ j.(iii) Existem pelo menos quatro elementos de S tais que, entre eles não haja três

contidos em um dos conjuntos i .

Os elementos de S são normalmente chamados pontos e os subconjuntos i , retas.Note que a propriedade (i) pode ser entendida como "por dois pontos passa umaúnica reta" e a propriedade (iii) nos diz que "existem quatro pontos, três a três nãocolineares".

Exercícios01. Um torneio de tênis é disputado entre duas equipes. Cada membro de uma

equipe joga com um ou mais membros da outra equipe, de modo que(i) Dois membros de uma mesma equipe têm exatamente um oponente em

comum;(ii) Não existem dois membros de uma equipe que enfrentam, juntos, todos os

membros da outra equipe.Prove que cada jogador deve jogar um mesmo número de partidas.

02. Mostre que as seguintes proposições são equivalentes em planos projetivos:(1) Existe uma reta que passa por exatamente n + 1 pontos;(2) Existe um ponto que está contido em exatamente n + 1 retas;(3) Todas as retas passam por exatamente n + 1 pontos;(4) Todos os pontos estão contidos em exatamente n + 1 retas;(5) Há exatamente n2 + n + 1 retas;(6) O plano projetivo tem exatamente n2 + n + 1 pontos (diz-se nesse caso que o

plano projetivo tem ordem n).

Page 156: Eureka 2002

Sociedade Brasileira de Matemática

EUREKA! N°15, 2002

32

03. (Cone Sul 1998) O prefeito de uma cidade deseja estabelecer um sistema detransportes com pelo menos uma linha de ônibus, no qual:

(i) cada linha passe exatamente por três paradas;(ii) cada duas linhas distintas tenham exatamente uma parada em comum;(iii) para cada duas paradas de ônibus distintas exista exatamente uma linha que

passe por ambas.Determine o número de paradas de ônibus da cidade.Dica: com o que parecem as condições (ii) e (iii)? Para encontrar os possíveisvalores, faça uma contagem dupla.

2. Construção de planos projetivos baseados em corposSeja K um corpo (ou seja, um conjunto onde estão definidas duas operações, + e ⋅, talque todo elemento admite oposto e todo elemento não nulo admite inverso). Então oconjunto P2(K) de ternas (x, y, z) ≠ (0, 0, 0), x, y, z ∈ K, onde ternas da forma (x, y, z)e (kx, ky, kz) devem ser consideradas iguais, é um plano projetivo na qual a retacorrespondente ao ponto (a, b, c), reta dual de (abc), é o conjunto de pontos (x, y, z)que satisfazem

ax + by + cz = 0

DemonstraçãoBasta mostrar que o conjunto dado tem as propriedades (i), (ii) e (iii).(i) Sejam (x1, y1, z1) e (x2, y2, z2) pontos distintos, ou seja, tais que não existe

K∈λ tal que ),,(),,( 222111 zyxzyx λ= . Temos que mostrar que existesomente uma reta que contém ambos os pontos, ou seja, que existe umaúnica terna (a, b, c) (que são os coeficientes da reta) tal que

0

0

222

111

=++=++

czbyax

czbyax (*)

Vejamos (*) como um sistema em a, b e c.Como ),,(),,( 222111 zyxzyx λ≠ para todo K∈λ , a matriz completa

=

222

111

zyx

zyxC do sistema (*) tem posto 2. Suponhamos, sem perda de

generalidade, que .022

11 ≠yx

yx Logo, resolvendo (*) em a e b, obtemos (verifique!)

a = mc e b = nc, onde m e n são constantes de K. Logo, as soluções de (*) são daforma (mc, nc, c) = c(m, n, 1). Note que 0≠c pois c = 0 implicaria a = b = c = 0.Assim, no plano projetivo P2(K), todas as soluções são equivalentes, ou seja, é única.Logo existe somente uma reta que passa por esses dois pontos.

Page 157: Eureka 2002

Sociedade Brasileira de Matemática

EUREKA! N°15, 2002

33

(ii) Análogo ao item (i). Tente você fazer!(iii) Observe que (1, 1, 1), (1, 0, 0), (0, 1, 0) e (0, 0, 1) satisfazem essas

condições. Aqui, 0 e 1 são os elementos neutros da soma e produto em K,respectivamente.

3. DualidadeObserve que se r e s são retas duais dos pontos R = (a, b, c) e S = (d, e, f) de P2(K),respectivamente, então

rScfbeadfcebdasR ∈⇔=++⇔=++⇔∈ 00A propriedade rSsR ∈⇔∈ é a chave do princípio da dualidade. Por isso, pontose retas no plano projetivo se comportam de maneira semelhante quando se fala deincidência.

4. O caso K = Quando K = , temos o plano projetivo P2() visto na referência [2]. Vamosentender a semelhança entre o plano definido aqui e o plano estudado em [2] (sevocê não leu essa referência, você pode pular essa seção; mas leia o artigo, elerealmente vale muito a pena!!).Primeiro, vamos explicar a primeira "estranheza" do plano projetivo P2(). Por queele tem três coordenadas, e não duas? A coordenada adicional é que faz aparecerempontos e retas no infinito. Podemos fazer a seguinte correspondência: o ponto (x, y)do plano 2 corresponde ao ponto (x, y, 1) de P2(). Sobraram os pontos do tipo(x, y, 0) de P2(), que são os pontos do infinito.A reta ax + by + c = 0 de 2 pode ser transformada na reta ax + by + cz = 0 de P2()substituindo-se x e y na reta de 2 por x/z e y/z. Aqui, nós homogeneizamos aequação ax + by + c = 0.Você pode visualizar a correspondência que fizemos da seguinte forma: seimaginarmos as ternas (x, y, z) como pontos no espaço, percebemos que os pontos deP2() correspondem a retas que passam pela origem. Se tomarmos o plano 1: =zαde 3, por exemplo, fazemos corresponder um ponto de P2() com o traço da retacorrespondente nesse plano.

Page 158: Eureka 2002

Sociedade Brasileira de Matemática

EUREKA! N°15, 2002

34

z

x

y

α A(a, b, 1)

)1,,( baλ

A reta dual de (a, b, c) corresponderia ao traço do plano ax + by + cz = 0 em π.Com alguns cálculos verifica-se que se R = (a, b, c) e r: ax + by + cz = 0, então,sendo O = (0, 0, 1), R' e r'os respectivos correspondentes ao ponto R e à reta r em α,então OR' ⊥ r'.

4.1. O plano projetivo e curvas elípticasConsidere a curva cúbica de R2

,0223223 =+++++++++ jiyhxgyfxyexdycxyybxax (**)onde todas as letras de a a j são números racionais.Gigantesco, não? Estamos interessados em saber sobre os seus pontos racionais (istoé, pontos cujas coordenadas são ambas racionais). Para isso, é feita a seguinteoperação: tomamos um ponto racional da curva, denominado O. Dados dois pontosP e Q racionais da curva, encontre o terceiro ponto P * Q de interseção de PQ com acurva. Defina P + Q (isso mesmo, estamos somando pontos!) como o terceiro pontode interseção da reta que passa por O e P * Q com a curva (isto é, P + Q = O * (P *Q). Pode-se provar que P + Q também é um ponto racional.

P * Q

Q

P + Q

P O

Só que a gigantesca equação (**) pode ser simplificada para a formay2 = x3 + ax2 + bx + c, ( )

Page 159: Eureka 2002

Sociedade Brasileira de Matemática

EUREKA! N°15, 2002

35

que é bem mais simples. Esta é a forma de Weierstrass. Para isso, usamos ageometria projetiva.Vamos fazer um caso particular. Considere a curva u3 + 2v3 = 2uv + 1. Um de seuspontos racionais é (1; 1).Primeiro devemos homogeneizar a curva. Fazendo u = U/W, obtemos U3 + 2V3 =2UVW + W3. Note que agora vamos trabalhar no plano projetivo P2().Depois, escolhemos um ponto racional O na curva. Uma escolha é O = (1; 1; 1).Agora, nós vamos mudar as coordenadas. A tangente à curva pelo ponto O é o nossoeixo de equação Z = 0. Se a tangente encontra a curva novamente em P (isso sempreocorre se O não for ponto de inflexão), o eixo de equação X = 0 é a tangente à curvaque passa por P. O eixo de equação Y = 0 pode ser qualquer reta que passe por O. SeO for ponto de inflexão, podemos escolher qualquer reta que não passa por O comoo eixo X = 0. Observe que como P2() tem três coordenadas, precisamos de trêseixos.

O

Z = 0

Y= 0

X = 0

Fazendo algumas contas (use um pouquinho de cálculo; para trabalhar só com duasvariáveis, "desomogenize" - essa palavra existe? - a equação), obtemos que atangente a O = (1; 1; 1) é a reta U + 4V – 5W = 0. Esta reta corta novamente a curvaem (–3; 2; 1). A tangente por esse ponto (use cálculo novamente) é 23U + 30V + 9W= 0. Por fim, tomamos a reta U – V = 0 como o eixo Y = 0. Assim, queremos que

310

535310

925310

935

93023

6262

54

ZYXW

ZYXV

ZYXU

XWVU

YVU

ZWVU

−−=

+−=

++=

⇔=++=−=−+

(usamos Y = 62U – 62V para simplificar um pouco as contas).Substituindo U, V e W (veja que o 310 dos denominadores vai cancelar), temos

Page 160: Eureka 2002

Sociedade Brasileira de Matemática

EUREKA! N°15, 2002

36

3

33

)535()535(

)925()935(2)925(2)935(

ZYXZYX

ZYXZYXZYXZYX

−−+−−

⋅+−⋅++=+−+++

A equação parece mais gigantesca que antes, mas após "desomogeneizar" (bastasubstituir Z = 1) e abrir tudo (eu colocaria as contas, mas a margem neste artigo émuito pequena para isso) obtemos

5159625)304( 22 −+−=−− xxyxxyEm geral, quando fazemos essa mudança de eixos (a transformação que fizemos aoresolver o sistema é uma transformação projetiva) a equação se reduz à forma

edxcxybaxxy ++=++ 22 )(

Multiplicando por x nos dois membros, temos ( ) exdxcxxybaxxy ++=++ 232 )(

Agora substitua z = xy: exdxcxzbaxz ++=++ 232 )(

Completando quadrado no primeiro membro e substituindo )(2

1baxz ++ por t

chegamos finalmente em 2t = cúbica em xPara a cúbica ser mônica (ou seja, ter coeficiente dominante unitário e não serdentuça), como o coeficiente dominante é c, basta multiplicar ambos os membrospor c2 e fazer as substituições (são as últimas!) t = m/c e x = n/c. Vamos fazer isso nonosso exemplo.Multiplicando por x, fazendo z = xy e completando quadrado:

22322 )152(5159625)152()304( −+−+−=−+−− xxxxxzxz

22557510025)152( 232 +−+−=+−⇔ xxxxz

Agora, t = z – 2x + 15: 22557510025 232 +−+−= xxxt

Por fim, 25

mt −= e

25

nx −= e chegamos à forma de Weierstrass:

14062514375100 232 +++= nnnm

Se você escrever m e n em função de u e v (e vice-versa), vai obter funções racionais(isto é, o quociente de dois polinômios) com todos os coeficientes racionais. Logopontos racionais são transformados em pontos racionais, de modo que achar ospontos racionais da curva original é a mesma coisa que achar os pontos racionais dacurva na forma de Weierstrass (fora, é claro, os pontos que anulam osdenominadores das funções racionais, que não são muitos e são fáceis de achar).Qual a vantagem da forma de Weierstrass? A vantagem é que podemos normalizar acurva para a forma ( ), além da soma de pontos. O ponto O pode ser um ponto

infinito, por exemplo. Homogeneizando ( ), obtemos ,32232 cZbXZZaXXZY +++=

Page 161: Eureka 2002

Sociedade Brasileira de Matemática

EUREKA! N°15, 2002

37

e não é difícil ver que o ponto do infinito (0; 1; 0) pertence à curva (e além disso, é oúnico!). Então podemos fazer O = (0; 1; 0). Isso facilita um pouco as contas paraadição de pontos. Se P * Q = (a; b)((a; b; 1) no plano projetivo), a reta que passa porO e P * Q é Y = bZ, ou y = b. Conseqüentemente, como a curva é simétrica emrelação ao eixo Ox, temos P + Q = (a; –b).

P + Q

Q

y

P * Q

P

x

Exercícios04. Prove que se P e Q são pontos racionais numa cúbica então P + Q também é

racional.

05. Transforme a cúbica u3 + v3 = α na forma de Weierstrass (um ponto racional dacurva homogeneizada é (1; –1; 0) e a conta não é tão terrível assim).

06. (OBM 2002, nível U) Considere a curva .16643|);( 322 +−=∈= xxyRyxC(a) Seja Q = (a; b) um ponto de C. Suponha que a reta tangente a C no ponto Q

intersecte C num único outro ponto, Q'. Determine as coordenadas de Q'.(b) Seja P0 = (3; 8). Para cada inteiro não negativo n, definimos Pn + 1 = P'n, o ponto

de interseção de C com a reta tangente a C em Pn. Determine P2002.

07. A adição de pontos é definida somente quando a cúbica no segundo membro de( ) tem três raízes distintas (não necessariamente todas reais). Os outros casossão mais fáceis!

(a) Encontre todos os pontos racionais da curva y2 = x2 (x – 1).(b) Encontre todos os pontos racionais da curva y2 = x3.

Page 162: Eureka 2002

Sociedade Brasileira de Matemática

EUREKA! N°15, 2002

38

5. O caso K = /p

Sendo K qualquer corpo, podemos tomar K = /p (os inteiros vistos mod p), emque p é um primo. Nesse caso, sendo /p um corpo finito (com p elementos) oplano projetivo P2(/p) é finito.

Por exemplo, fazendo p = 2, obtemos o plano de Fano (rimou!), como pode ser vistona figura.

(0, 0, 1)

(0, 1, 1) (1, 1, 1)

(1, 0, 1)

(0, 1, 0) (1, 0, 0) (1, 1, 0)

Vamos fazer algumas continhas.

6. Algumas contagens

6.1. Número de elementos de P2(/p)

Temos p3 – 1 ternas (a, b, c) ≠ (0, 0, 0). Como (a, b, c) é equivalente a ),,( cbaλ e

λ pode assumir p – 1 valores (1 a p – 1), cada ponto está sendo contado p – 1 vezes.

Logo P2(/p) tem 111 2

3

++=−−

ppp

p pontos.

Observe que, pelo princípio da dualidade, há também p2 + p + 1 retas.

6.2. Número de pontos em cada retaFixados a, b, e c, não todos nulos, queremos contar o número de soluções nãoequivalentes (x, y, z) ≠ (0, 0, 0) da congruência

0≡++ czbyax (mod p) (***)

Suponhamos, sem perda de generalidade, que 0≠c (mod p). Temos então

(***) ybcxacz 11 −− −−≡⇔ (mod p)

Page 163: Eureka 2002

Sociedade Brasileira de Matemática

EUREKA! N°15, 2002

39

Podemos escolher x e y de p2 maneiras. Porém, não podemos escolher x = y = 0, poisisso implicaria z = 0. Logo, considerando que cada elemento de P2(/p) tem p – 1

equivalentes, temos que cada reta tem 1112

+=−−

pp

p pontos.

6.3. Número de retas que passam por um pontoPelo princípio da dualidade, há p + 1 retas que passam por um ponto dado.

7. Um problemaA seguir, o problema 3 da Olimpíada Iberoamericana de 1996, realizada na CostaRica.

Temos um tabuleiro quadriculado de 12 +− kk linhas e 12 +− kk colunas, ondek = p + 1 e p é um número primo. Para cada primo p, dê um método para distribuirnúmeros 0 e 1, um número em cada casa do tabuleiro, de modo que em cada linhahaja exatamente k números 0, em cada coluna haja exatamente k números 0 e, alémdisso, não haja nenhum retângulo, de lados paralelos aos lados do tabuleiro, comnúmeros 0 em seus quatro vértices.

Resolução

Para 11)1()1(1,1 222 ++=++−+=+−+= ppppkkpk (coincidência? destino?

ou puramente sorte?). Assim, devemos preencher um tabuleiro (p2 + p + 1) ×(p2 + p + 1) com 0 e 1 de modo que haja p + 1 (que coisa…) 0 em cada linha ecoluna, e sem retângulos com 0 vértices.Considere o plano projetivo P2(/p) (por que será?) e a cada linha associe umponto e a cada coluna associe uma reta. Coloque 0 na casa (i, j) se, e somente se, oponto i pertence à reta j. Nas demais casas, coloque 1.Há claramente p + 1 zeros em cada coluna. Pelo princípio da dualidade, também háp + 1 zeros em cada linha.Agora, suponha que exista um retângulo de vértices (a, c), (a, d), (b, c) e (b, d), a ≠ be c ≠ d, todos com 0. Logo, pela nossa construção, os pontos a e b pertencem aambas as retas c e d. Absurdo, pois a interseção de duas retas é exatamente umponto.É claro que na prova você teria que demonstrar todas as propriedades quedemonstramos antes.

8. Outros fatos sobre planos projetivos finitosA construção baseada em corpos vale para qualquer corpo. Mas infelizmente, onúmero de elementos de um corpo finito deve ser potência de primo (um corpo com

Page 164: Eureka 2002

Sociedade Brasileira de Matemática

EUREKA! N°15, 2002

40

pn elementos, p primo, n inteiro positivo, é o conjunto dos polinômios comcoeficientes em /p, vistos módulo um polinômio P(x) de grau n e irredutível em/p[x]). Assim, tal construção só nos permite construir planos projetivos de ordempotência de primo. Será que existem planos projetivos com outra ordem?Conjectura-se que não, porém esse problema continua em aberto. O teorema deBruck-Ryser-Chowla ajuda um pouquinho, dizendo que se n ≡ 1 (mod 4) ou n ≡ 2(mod 4) e n é ordem de um plano projetivo então n deve ser soma de dois quadrados.Pensando nos casos pequenos, esse teorema elimina o caso n = 6. A demonstração deque não existe plano projetivo de ordem n = 10 foi obtida em 1989 por C. W. H.Lam (com o auxílio de um computador!) e a história da prova está disponível em [7].O caso n = 12 está em aberto.Planos projetivos também servem para construir alguns block designs. Um blockdesign consiste num sistema de incidência (v, k, λ , r, b) na qual um conjunto X de vpontos é particionado numa família A de b subconjuntos (chamados blocks) de modoque dois pontos quaisquer determinam λ blocks com k pontos em cada block, e cadaponto está contido em r blocks. Na verdade, os cinco parâmetros não sãoindependentes. Fica como exercício para você mostrar que vr = bk e λ (v – 1) =r (k – 1) (são duas contagens duplas). Assim, pode-se representar o block designsimplesmente como (v, k, λ ). Se b = v (e, conseqüentemente, r = k) o block design édito simétrico. Note que um plano projetivo de ordem n é block design(n2 + n + 1, n, 1). Outros exemplos e mais informações podem ser encontrados nosite: http://mathworld.wolfram.com/BlockDesign.html.

Exercícios08. (OPM 2001) No condado Heptaprojetivo havia 7 castelos, batizados segundograndes personagens: Arnold (A), Borcherds(B), Conway (C), Dilbert (D), Erdös (E),Faltings (F), Gowers (G). Havia também 7 ruas, cada uma com 3 castelos, comomostra o mapa a seguir (uma rua é circular):

A

B G

E

D F C

Page 165: Eureka 2002

Sociedade Brasileira de Matemática

EUREKA! N°15, 2002

41

Um belo dia, o conde Steiner decidiu retirar as placas que identificavam os castelospara fazer uma limpeza. Na hora de recolocá-las, ninguém se lembrava do lugarcorreto de cada uma, nem mesmo os moradores dos castelos! Os arquivos docondado só indicavam os castelos que ficavam numa mesma rua, mas não a ordemem que eles estavam. Assim, o conde sabia que havia uma rua com os castelos A, B,D, outra com B, C, E, outra com B, F, G, etc.Frente aos fatos, Steiner resolveu determinar todas as maneiras de recolocar asplacas respeitando os arquivos do condado, isto é, todas as maneiras nas quais placasque estavam juntas em uma mesma rua continuassem juntas em uma rua,possivelmente em outra ordem. Duas destas maneiras estão representadas a seguir:

A

F C

B

E D G

B

G A

E

F C D

Chamaremos essas maneiras de válidas.(a) Prove que o total de maneiras válidas é 7 vezes o número de maneiras válidas

nas quais a placa A é colocada no castelo A.(b) Prove que o total de maneiras válidas nas quais a placa A é colocada no castelo

A é 6 vezes o número de maneiras válidas nas quais a placa A é colocada nocastelo A e a placa B é colocada no castelo B.

(c) Determine o número de maneiras válidas.

09. No reino da Alândia há n cidades, assim como no reino da Belândia. Foramconstruídas m estradas, sendo que cada estrada passa por exatamente duascidades, uma de cada reino. Mostre que se não existem quatro cidades ligadas

por um ciclo de estradas, então )341(2

−+≤ nn

m

Mostre que a igualdade pode ocorrer para infinitos valores de n.Dicas: a igualdade pode ser demonstrada com uma injeção e contagem dupla - vejao artigo Grafos e Contagem Dupla, na Eureka! 12; para mostrar que a igualdadepode ocorrer para infinitos valores de n, encontre primeiro os valores de n para osquais 4n – 3 é um quadrado perfeito - você vai se surpreender!

Page 166: Eureka 2002

Sociedade Brasileira de Matemática

EUREKA! N°15, 2002

42

10. (Extensão do exemplo do artigo Grafos e Contagem Dupla) Na Terra de Oz há ncastelos e várias estradas, sendo que cada uma liga dois castelos. Diz a lenda quese houver quatro castelos ligados em ciclo (ou seja, se existirem quatro castelosA, B, C, e D tais que A e B, B e C, C e D e D e A estão ligados), um dragãoaparecerá do centro dos castelos e destruirá a Terra de Oz. Mostre que para estadesgraça não acontecer o número de estradas deve ser menor ou igual a

( )4

341n

n −+ é mostre que para infinitos n é possível construir

( )4

1341

−−+ nn estradas. (para mostrar isto você vai precisar saber um pouco

de Álgebra Linear).

REFERÊNCIAS:

[1] D. Pedoe, Geometry: A Comprehensive Course. Este livro tem um bom texto não somenteintrodutório sobre geometria projetiva e contém mais aplicações geométricas. O problema 1 e osaxiomas de planos projetivos foram retirados deste livro, assim como a construção de planos projetivoscom coordenadas.[2] L. Castro, Introdução à Geometria Projetiva, Revista Eureka! 8. A melhor referência que conheçopara começar a estudar geometria projetiva aplicada a problemas de geometria.[3] O problema 9 foi adaptado de um exemplo retirado do livro Graph Theory: An Introductory Course,de B. Bollobás. Ele é um caso particular do problema de Zaranckiewsky: qual é o número máximo dearestas de um grafo bipartido com m vértices em uma classe V1 e n vértices na outra classe V2, de modoque não haja nenhum subgrafo bipartido completo com r vértices de V1 e s vértices de V2? Tal número énormalmente representado por z(m; n; r; s).[4] O problema 10 é uma extensão de um exemplo de meu artigo Grafos e Contagem Dupla, que estána Revista Eureka! 12.[5] A construção do exemplo do problema 10 pode ser encontrada no livro Proofs From The Book, deMartin Aigner e Günter M. Ziegler. Mas tente você mesmo fazê-la antes![6] Curvas elípticas e a redução para a forma de Weierstrass podem ser encontradas em Rational Pointson Elliptic Curves, de Joseph H. Silverman e Jonh Tate.[7] A história da demonstração de que não existe plano projetivo de ordem 10 está em

http://www.cecm.sfu.ca/organics/papers/lam/paper/html/paper.html[8] Um glosário de termos da Matemática e principais fatos relacionados está em

http://mathworld.wolfram.com/[9] Uma discussão muito interessante sobre o assunto está em

http://www.mathematik.uni-bielefeld.de/~sillke/PUZZLES/projective_plane

(o grande matemático John Conway inclusive participa desta discussão).

Page 167: Eureka 2002

Sociedade Brasileira de Matemática

EUREKA! N°15, 2002

43

OLIMPÍADAS AO REDOR DO MUNDO

Chegamos ao último número de 2002. Apresentamos, como sempre,questões que não são encontradas facilmente na Internet. Feliz 2003, divirtam-se eenviem as suas soluções.

Continuamos à disposição na OBM para aqueles que estiverem interessadosna solução de algum problema particular. Para tanto, basta contactar a OBM, atravésde carta ou e-mail.

Antonio Luiz Santos

Primeiramente vamos aos problemas propostos deste número

181.(Rússia-2002) Um número de quatro algarismos é escrito em um quadro negro. Épermitido adicionar 1 a quaisquer dois algarismos adjacentes se nenhum deles forigual a 9 ou subtrair 1 de quaisquer dois algarismos adjacentes se nenhum deles forigual a 0. Partindo de 1234 é possível obtermos 2002 após efetuarmos estasoperações diversas vezes ?

182.(Rússia-2002) Seja ABC um triângulo cujas medidas dos lados são distintas duas aduas. Exteriormente ao triângulo, são construídos sobre os seus lados os triânguloseqüiláteros ABC1, BCA1 e CAB1. Mostre que o triângulo A1B1C1 não é equilátero.

183.(Rússia-2002) Um polinômio quadrático de coeficientes inteiros e coeficiente dosegundo grau igual a 1 assume valores primos em três valores inteiros econsecutivos. Mostre que ele assume um valor primo em pelo menos mais um valorinteiro.

184.(Rússia-2002) Seja O o circuncentro de um triângulo isósceles ABC(AB = BC). Umponto M pertence ao segmento BO e o ponto M' é o simétrico de M em relação aoponto médio de AB. Sejam ainda K o ponto de interseção de M'O com AB e L umponto sobre o lado BC tal que BLMCLO ∠=∠ . Mostre que os pontos O, K, B, Lpertencem a um mesmo círculo.

185.(Rússia-2002) Qual o maior número de termos possível de uma progressãoaritmética de números inteiros positivos (a1, a2, a3,…,an) de razão igual a 2 tal que o

número 12 +ka seja primo para todo k = 1, 2, 3,…,n?

186.(Rússia-2002) Sobre o eixo x são escolhidos pontos x1, x2,.., xn com n ≥ 3 distintosdois a dois. Traçam-se então todas as parábolas de coeficientes do segundo grauigual a 1 e que cortam o eixo dos x somente nos pontos escolhidos. Sejam

Page 168: Eureka 2002

Sociedade Brasileira de Matemática

EUREKA! N°15, 2002

44

mfyfy == ,...,1 as equações destas parábolas. Mostre que a parábola

mffy +⋅⋅⋅+= 1 corta o eixo x em exatamente dois pontos.

187.(Rússia -2002) Uma seqüência de números (an) é tal que a0 = 0 e 11 +≥+ nn aa .

Mostre a desigualdade :2

11

3

≥ ∑∑

==

n

kk

n

kk aa

188.(Rússia-2002) No intervalo (22n, 32n) são escolhidos 12 12 +−n números ímpares.Mostre que podemos encontrar entre estes números dois números tais que oquadrado de cada um deles não é divisível pelo outro.

189.(Albânia-2002) Sendo →∗:f uma função definida por

( )

−=

+xx

x

xf 223

11

Determine o valor da soma :

+⋅⋅⋅+

+

+

+⋅⋅⋅+

+

1

20022

2000

20022

2001

20022

2002

2002

2002

2

2002

1ffffff

190.(Albânia-2002) Mostre como dividir um triângulo qualquer em :(i) 2002 triângulos semelhantes a ele.(ii) 2003 triângulos semelhantes a ele.

191.(Irlanda-2002) Em um triângulo ABC tem-se que AB = 20, AC = 21 e BC = 29. SeD e E são pontos do segmento BC tais que BD = 8 e EC = 9, determine a medida doângulo DAE∠ .

192.(Irlanda-2002) Suponha que n seja o produto de quatro números primos distintos a,b, c, d tais que :(i) a + c = d(ii) a(a + b + c + d) = c (d – b)(iii) 1 + bc + d = bdDetermine n.

Page 169: Eureka 2002

Sociedade Brasileira de Matemática

EUREKA! N°15, 2002

45

193.(Irlanda-2002) Seja (an) uma seqüência definida por 11,1, 321 === aaa e

2121 =− −−+ nnnn aaaa para todo 3≥n . Prove que na é um inteiro positivo para todo1≥n .

194.(Eslovênia-2002) 38 é o menor inteiro positivo tal que o seu quadrado termina comtrês quatros (382 = 1444). Qual o próximo inteiro positivo com esta propriedade ?

195.(Eslovênia-2002) Sejam M o ponto médio da base AB do trapézio ABCD; E umponto interior ao segmento AC tal que BC e ME intersectam-se em F; G o ponto deinterseção de FD e AB; H o ponto de interseção de DE e AB. Mostre que M é oponto médio do segmento GH.

196.(Eslovênia-2002) Sejam k um círculo do plano euclidiano, k1 e k2 dois círculosdisjuntos interiores a k e que o tangenciam nos pontos A e B respectivamente. Sejaainda (t) a tangente comum aos círculos k1 e k2 nos pontos C e D respectivamente talque k1 e k2 estejam situados num mesmo semiplano determinado pela reta (t)enquanto que o centro do círculo k pertence ao outro semiplano. Se E é o ponto deinterseção de AC e BD, mostre que o ponto E pertence ao círculo k.

197.(Eslovênia-2002) Determine o menor inteiro positivo que pode ser escrito comouma soma de 9, 10 e 11 inteiros positivos consecutivos.

198.(Estônia-2002) Em um triângulo ABC tem-se que C2B ∠⋅=∠ e a bissetriz doângulo A∠ intersecta o lado BC no ponto D tal que AB = CD. Determine a medidado ângulo A∠ .

199.(Estônia-2002) Um número natural de 10 algarismos distintos é dito mágico se elefor múltiplo de 99999. Quantos números mágicos não começados por zero existem ?

200. (Grécia-2002) Determine todos os inteiros positivos N tais que :(i) N possui exatamente16 divisores Nddd =<<<= 1621 ...1

(ii) ( ) 6425 dddd += .

201. (Inglaterra-2002) Um quadrilátero ABCD está inscrito em um círculo. As diagonaisAC e BD intersectam-se no ponto Q. Os prolongamentos dos lados DA e CB, a partirde A e B respectivamente, intersectam-se em P. Sabendo que CD = CP = DQ, mostreque 60ºCAD =∠ .

Page 170: Eureka 2002

Sociedade Brasileira de Matemática

EUREKA! N°15, 2002

46

202.(Inglaterra-2002) Doze pessoas estão sentadas em torno de uma mesa circular. Dequantos modos seis pares de pessoas podem trocar apertos de mão de modo que nãohaja cruzamentos de braços?(Não é permitido que uma pessoa troque apertos de mão com mais de uma pessoa de cada vez)

203.(Inglaterra-2002) Seja f uma função de em , onde é o conjunto dos númerosinteiros não negativos, que possui as seguintes propriedades :(i) ( ) ( )nfnf >+1 , para cada ∈n .

(ii) ( )( ) ( ) 1++=+ mnfmfnf , para todos ∈nm, .Determine todos os valores possíveis de f(2001).

204.(Inglaterra-2002) A altura traçada de um dos vértices de um triângulo ABCintersecta o lado oposto no ponto D. A partir do ponto D são traçadasperpendiculares aos outros dois lados intersectando-os nos pontos E e F. Mostre quea medida do segmento EF independe do vértice do qual a altura é traçada.

205.(Áustria-2002) Determine o menor inteiro positivo x para o qual todas as frações :

48

493,...,

10

113,

9

103,

8

93 ++++ xxxx

são irredutíveis.

206.(Áustria-2002) Determine o maior número real C tal que

( )( )( )( )( )

Cyx

yxyx ≥−

+−−+2

22 86

é verdadeira para todos os números reais ( )yxyex ≠ com xy = 2. Para quaispares ordenados (x, y) a igualdade é verdadeira ?

207.(Áustria-2002) Seja ( )39

9

+=

x

x

xf

Determine a soma de todas as expressões da forma

2002

kf onde k é um número

inteiro entre 0 e 2002 tal que a fração 2002

k seja irredutível.

208. (Bielorússia-2002) Determine o maior número possível de grupos que podem serextraídos do conjunto 1, 2, 3,…,19, 20, de modo que o produto dos números decada um dos grupos seja um quadrado perfeito .(O grupo pode conter apenas um número e, neste caso, o produto é igual a este número; além disso, cada númerodeve fazer parte de apenas um grupo)

Page 171: Eureka 2002

Sociedade Brasileira de Matemática

EUREKA! N°15, 2002

47

209. (Bielorússia-2002) A altura CH de um triângulo retângulo ABC, com 90º=∠ Cintersecta as bissetrizes AM e BN nos pontos P e Q respectivamente. Prove que areta que passa pelos pontos médios de QN e PM é paralela à hipotenusa de ABC.

210. (Bielorússia-2002) Um conjunto de números naturais de três algarismos formadoscom 1, 2, 3, 4, 5, 6 é chamado agradável se ele satisfaz à seguinte condição: paraquaisquer dois algarismos distintos de 1, 2, 3, 4, 5, 6, existe um número desteconjunto que contém ambos. Se, para qualquer conjunto agradável calcularmos asoma de seus elementos, determine o menor valor possível destas somas.

Agora vamos aos comentários e soluções dos leitores para alguns dosproblemas apresentados em números anteriores de EUREKA!. Como sempre, ocritério por nós adotado para este número foi apresentar as soluções dos problemasque foram, até o presente momento, resolvidos pelo maior número de leitores.

32. (Moldávia-1998) A seqüência ( )na , ∈n * verifica as relações 2

11 =a e

12 1

1

+=

n

nn

na

aa para todo número natural 1>n . Calcule 199821 aaa +⋅⋅⋅++ .

Resumo das soluções de diversos leitores:Escrevamos a lei de recorrência de uma forma ligeiramente diferente:

naaa

na nnnn

2111

21

11

=−⇔+=−−

Logo,

( )∑∑== −

=

n

k

n

k kk

kaa 22 1

211

O primeiro membro desta última igualdade é uma soma telescópica e igual a

1

11aan

− enquanto que o segundo membro é igual a

( )( )2

1242864

−+=+⋅⋅⋅+++ nnn . Daí,

Page 172: Eureka 2002

Sociedade Brasileira de Matemática

EUREKA! N°15, 2002

48

( )( ) ( ) ( ) 1

11

1

11

1122

1

+−=

+=⇔+=⇔−+=−

nnnnann

ann

an

nn

Finalmente,

9199

8199

9199

11

1

111998

1

1998

1

=−=

+−= ∑∑

== nii

nna

36.(China-1999) Seja PQRS um quadrilátero inscrito num círculo e cuja medida do

ângulo PSR∠ seja igual a o90 . Se H e K são os pés das perpendiculares baixadasde Q sobre PR e PS respectivamente (convenientemente prolongados se necessário).Mostre que HK divide QS ao meio.

Resumo das soluções de diversos leitores:Seja G o ponto de interseção de KH e SR. Como P, Q, R e S são cíclicos (pertencema um mesmo círculo), assim como K, Q, H e P tem-se que

.QSGQPRQKHQKG ∠=∠=∠=∠ Daí K, Q, G e S são cíclicos com

SKQ2KSG ∠==∠ . Desta forma, KQGS é um retângulo e KH divide QS ao

meio.

39. (Irlanda-1999) Determine todos os inteiros positivos m tais que a quarta potência donúmero de seus divisores positivos é igual a m.

Resumo das soluções de diversos leitores:Observemos em primeiro lugar que m = 1 possui a referida propriedade.Suponhamos agora que m ≥ 2. Como m é uma quarta potência ele pode ser escritounivocamente como

nnpppm 44

24

121 ×⋅⋅⋅××=

onde nppp <⋅⋅⋅<< 21 são números primos e 0,i > ni ,...2,1,= . O número dedivisores de m é dado por

( )( ) ( )141414 21 +⋅⋅⋅++ n

A requerida propriedade é

( )( ) ( ) n21

2

121 141414 nn ppp ×⋅⋅⋅××=+⋅⋅⋅++

Como o lado esquerdo é ímpar então 31 ≥p . Observe que 143 +> se 3≥β ;

também, 145 +> se 2≥β ; finalmente 14p +> se 1≥β e 7≥p . Além disso,

145 +≥ se 1≥β , assim, se 14 +< ββp para algum primo ímpar p e algum

1≥β então 3=p . Deste modo, devemos ter 31 =p ou 51 =p .

Page 173: Eureka 2002

Sociedade Brasileira de Matemática

EUREKA! N°15, 2002

49

1º Caso : 51 =p

Neste caso, 11 =β e assim

( ) ( ) n2

22 514145 nn pp ×⋅⋅⋅××=+⋅⋅⋅+Pela terceira desigualdade acima não podem existir outros fatores primos em m

distintos de 5 e portanto a única solução é 45=m .

2º Caso : 31 =p . Claramente 11 =β ou 21 =β uma vez que ( ) 13433 +> .

Se 11 =β , então ( ) ( ) n2

22 314145 nn pp ×⋅⋅⋅××=+⋅⋅⋅+ . Assim 52 =p . Se

22 >β então ( )143

55 22 +> o que nos leva a uma contradição. Deste modo,

12 =β ou 22 =β .

Se 12 =β então ( ) ( ) n3

33 15141425 nn pp ×⋅⋅⋅××=+⋅⋅⋅+ o que é uma contradiçãojá que 25 não divide o lado direito.

Se 22 =β , então ( ) ( ) n3

33 75141445 nn pp ×⋅⋅⋅××=+⋅⋅⋅+ o que é umacontradição já que 9 não divide o lado direito.

Se 21 =β então ( ) ( ) n2

22 914149 nn pp ×⋅⋅⋅××=+⋅⋅⋅+ .

Não podemos ter 72 ≥p logo, 1=n ou ( 2=n e 52 =p e 12 =β ). O primeiro caso

nos dá 83=m ; o último caso nos dá 48 53 ×=m . Concluindo existem quatrosoluções a saber,

4884 53e3,5,1 ×

40. (Irlanda-1999) Mostre que existe um número inteiro positivo na seqüência deFibonacci que é divisível por 1000 .

Resumo das soluções de diversos leitores:

Considere o conjunto de 1106 + pares

( ) 61 ,10...1,2,|, =+ iFF ii

tomados ( )310mod . Como o conjunto ( ) 999,0|, ≤≤ baba com a e b inteiros

possui apenas 610 elementos, o Princípio da Casa dos Pombos nos diz que existeminteiros ji > tais que 1j1i FF ++ − e ji FF − são ambos divisíveis por 1000. Mas

( ) ( )ijjiji FFFFFF −+−=− ++−− 1111

Page 174: Eureka 2002

Sociedade Brasileira de Matemática

EUREKA! N°15, 2002

50

Logo, 11 −− − ji FF é também divisível por 1000. Argumentando desta maneira de

frente para trás vemos que 11ji FF −+− e 0ji FF −− são divisíveis por 1000. Mas,

00 =F daí, jiF − é divisível por 1000.

60. (St.Petersburg-1999) Três mágicos apresentam um truque entregando a uma pessoada platéia um maço de cartas numeradas com 1, 2,...,2 n + 1 (n > 6). O espectadorfica com uma das cartas e aleatoriamente distribui as restantes entre o primeiro e osegundo mágicos (cada um deles fica com n cartas) . Estes olham suas cartas (sem secomunicar um ao outro) e cada um escolhe duas cartas formando um maço(ordenado) com estas cartas e as entrega ao terceiro mágico. O terceiro mágico olhaestas quatro cartas e anuncia a carta que ficou com o espectador. Explique como estetruque pode funcionar.

SOLUÇÃO DE JEAN-CHRISTOPHE YOCCOZ (PARIS - FRANÇA):A ideia é que cada mágico transmita de algum jeito a soma de suas n cartas módulo2n + 1 (e então o outro mágico descobre a carta que foi retirada, que é o simétrico dasoma dos dois resultados módulo 2n + 1).

Assim, basta definir uma função

)12(: +nf ×

)12( +n →

)12( +n tal

que, dado c em

)12( +n e um subconjunto arbitrário A de

)12( +n com n

elementos eles satisfazem a propriedade P(A, c): existem elementos distintos x e y deA com f(x, y) = c.A idéia é mostrar a existência de uma função f(x, y) como acima que sejaequivariante, isto é, que satisfaça f(x + c, y + c) = f(x, y) + c, para quaisquer x, y e c.É fácil ver que para uma tal função f vale P(A, c) se e somente se vale P(A – c, 0),onde A – c = x – c | x ∈ A. Assim, basta ver que todo A com n elementos satisfazP(A, 0). Agora, em vez de construir f, vamos construir a pré-imagem de 0 pela f (ouum subconjunto conveniente dela).Pela equivariância, se (x, y) e (u, v) são pares distintos com f(x, y) = f(u, v) = 0 entãoy – x tem que ser diferente de v – u, senão, para c = u – x, temos (u, v) = (x+ c, y + c),donde f(u, v) = c. Por outro lado, dado um conjunto de pares(x1, y1),...,(xk, yk) comyj – xj distinto de yi – xi para todo i distinto de j, existe uma função equivariante f comf(xi, yi) = 0 para todo i. Nosso problema agora se reduz a encontrar um conjunto depares (x1, y1),...,(xk, yk) com os yi – xi todos distintos (e distintos de 0) tal que paratodo A com n elementos exista i tal que xi e yi estejam em A. Fazemos com que esseconjunto de pares contenha os pares (0, 1), (2, 1), (0, 2), (3, 6), (9, 6), (3, 9), (4, 8),(12, 8) e (4, 12). Note que as diferenças yi – xi são 1, –1, 2, 3, – 3, 6, 4, – 4 e 8, e queum A ruim só pode conter um elemento de cada uma das triplas 0, 1, 2, 3, 6, 9 e

Page 175: Eureka 2002

Sociedade Brasileira de Matemática

EUREKA! N°15, 2002

51

4, 8, 12. Vamos completar nosso conjunto de pares com n – 4 pares que formarão

uma partição de

)12( +n \0, 1, 2, 3, 6, 9, 4, 8, 12. Um conjunto ruim só pode

ter um elemento de cada par, tendo pois no máximo n – 4 + 3 = n – 1 elementos,como queríamos. Para isso considere alguns casos:

I) n par, n ≥ 12: Escrevemos n = 2k. Os pares serão (– k + 6, k + 6),(– k + 7, k + 5),...,(–1, 13) (diferenças n, n – 2 ,...,14); (k + 7, 3k + 6),(k + 8, 3k + 5),...,(2k + 4, 2k + 9) (diferenças n –1, n – 3,...,5); e(2k + 5, 5), (2k + 6, 7), (2k + 7, 11) e (2k + 8, 10)(diferenças – n, 1 – n, 4 – n e 2 – n).

II) n ímpar, m ≥ 13: Escrevemos n = 2k + 1. Os pares serão (–1, 13),...,(– k + 6, k + 6) (diferenças 14 ,..., n –1); (k + 7, 3k + 8),...,(2k + 5, 2k + 10)(diferenças n ,..., 5); e (2k + 6, 5), (2k + 7, 7), (2k + 8, 11) e (2k + 9, 10)(diferenças – n, 1– n, 4 – n e 2 – n).

Casos particulares anteriores:n = 7: Pares (5, 10), (13, 11) e (7, 14) (diferenças 5, – 2 e 7).n = 8: Pares (5, –1), (7, 14), (15, 10) e (13, 11) (diferenças – 6, 7, – 5 e – 2).n = 9: Pares (5, 14), (16, 7), (10, 17), (18, 11) e (15, 13) (diferenças 9, –9, 7, –7 e–2).n = 10: Pares (20, 10),(19, 11),(18, 13),(5, 14),(16, 7) e (17, 15) (diferenças –10, – 8,– 5, 9, – 9 e – 2).n = 11: Pares (16, 5), (17, 7), (20, 11), (18, 10), (19, 13), (21, 14) e (15, 22)(diferenças – 11, – 10 , – 9, – 8, – 6, – 7 e 7).

Publicamos a seguir, a pedidos, uma solução do problema 12, proposto naEureka! No. 8.

12. (Irlanda-1999) Três números a < b < c estão em progressão aritmética se c – b = b –a. Definamos a seqüência (un), n = 0, 1, 2, 3,… da seguinte maneira: u0 = 0, u1 = 1 epara cada n ≥ 1, un + 1 é o menor inteiro positivo tal que nn uu >+1 e

,,...,, 110 +nn uuuu não possui três elementos em progressão aritmética. Determineu100.

Page 176: Eureka 2002

Sociedade Brasileira de Matemática

EUREKA! N°15, 2002

52

Vamos mostrar por indução que se 201 )...( aaan kk −= , i.e, ∑=

⋅=k

j

jjan

0

2 , com

ja j ∀∈ ,1,0 , então ∑=

− ⋅==k

j

jjkkn aaaau

0301 .3)...( Suponha que isso seja válido

para 120 −≤≤ kn (isso é verdade para k = 1).

Como ,...,,1210 −kuuu não possui três elementos em progressão aritmética e

∑−

=−

−=⋅=1

012 2

1331

k

j

kj

ku , então 12101210 3,...,3,3,,...,, −− +++ kk uuuuuu kkk também

não possui três elementos em progressão aritmética, pois se dois termos de uma

progressão aritmética estão entre 0 e ,2

13 −k

então o terceiro termo é menor ou igual

a ,313 kk <− e se dois termos de uma progressão aritmética estão entre 3k e 3k +

2

13 −k

então o termo anterior é maior ou igual a .2

13

2

13

2

133

−>+=−−kkk

k

Vamos ver que, de fato, kku 3

2= , donde j

kj

uu k +≥+3

2 para 120 −≤≤ kj , e logo,

pelo fato acima, ,32 j

kj

uu k +=+ para .120 −≤≤ kj Para isso, basta ver que se

kk

u 32

13 <<− então ,,...,,

1210 uuuu k − contém uma progressão aritmética de três

termos. Seja ∑−

=

⋅=1

0

3k

j

jjbu , com .,2,1,0 jb j ∀∈ Então 122 vvu −= , onde

∑∑−

=

=

⋅=⋅=1

01

1

02 ,3,3

k

j

jj

k

j

jj cvdv e, se 0=jb , então );0,0(),( =jj dc se 1=jb então

)1,1(),( =jj dc e, se bj = 2 então ).1,0(),( =jj dc

Em particular, ,,1,0, jdc jj ∀⊂ donde v1 e v2 pertencem a ,...,,1210 −kuuu , e

,122 vvvu −=− logo uvv << 21 estão em progressão aritmética.

Finalmente, 100 = 26 + 25 + 22, donde u100 = 36 + 35 + 32 = 981.

Page 177: Eureka 2002

Sociedade Brasileira de Matemática

EUREKA! N°15, 2002

53

Acusamos o recebimento de soluções de problemas anteriores dos seguintes leitoresde EUREKA!:

Anderson Torres São Paulo - SP Prob. 24, 32, 35, 37, 40, 43, 45, 51, 54, 56, 63, 88, 103, 114, 124,143, 151, 154, 158, 160, 164, 168, 169, 170, 171, 172.

Carlos A. Gomes Natal - RN Prob. 151, 162, 174.Erasmo de Souza Dias Goiânia - GO Prob. 151.Fernando Carvalho Ramos Santa Maria - RS Prob. 151, 160, 163, 174.Francisco B. de Lima Holanda Fortaleza - CE Prob. 36, 45, 47, 69, 87, 88, 102, 104, 109, 169, 170.Geraldo Perlino Itapecerica da Serra - SP Prob. 151, 160, 161, 164, 167.Geraldo Perlino Júnior Cotia - SP Prob. 151, 152, 153, 154, 158, 159, 160, 161, 162, 163, 165, 166,

167, 168, 170, 171, 172, 173, 174, 178, 179,Gibran Medeiros de Souza Natal - RN Prob. 163, 166.Helder Oliveira de Castro Mogi das Cruzes - SP Prob. 43, 45, 46, 54, 66, 63, 67, 70, 74, 86, 90, 99, 101, 102, 105,

115, 128, 133, 143.Luiz Sérgio Carvalho de Mello Rio de Janeiro - RJ Prob. 138, 151, 166.Marcelo Ribeiro de Souza Rio de Janeiro - RJ Prob. 151, 153, 154, 160, 162.Marcelo Rufino de Oliveira Belém - PA Prob. 151, 152, 153, 154, 158, 160, 162, 163, 164, 165, 166, 170,

171, 172, 173, 174, 179, 180.Okakama Matsubaxi São Paulo - Sp Prob. 130.Renato F. L. Mello J. dos Guararapes - PE Prob. 154, 160, 162, 166, 173, 179.Tertuliano C. de Souza Neto Salvador - BA Prob. 61, 62, 68, 69, 70, 72, 74, 76, 82, 83, 87, 88, 90.

Page 178: Eureka 2002

Sociedade Brasileira de Matemática

EUREKA! N°15, 2002

54

SOLUÇÕES DE PROBLEMAS PROPOSTOS Publicamos aqui algumas das respostas enviadas por nossos leitores.

66) Prove que, dados um inteiro n ≥ 1 e um conjunto 2/ nA ⊂ com n

elementos existe 2/ nB ⊂ com n elementos tal que

2/,| nByAxyxBA ⊂∈∈+=+ tem mais de n2/2 elementos.

SOLUÇÃO DE ZOROASTRO AZAMBUJA NETO (RIO DE JANEIRO - RJ):

Vamos mostrar que, dado um conjunto 2/ nX ⊂ , existe

2/ nt ∈ tal queXtA ∩+ )( tem pelo menos nX / elementos, onde X é o número de elementos

de X. De fato, ( ) XnXAxtantXtAnt Aa Xx

⋅=⋅==+∈=∩+∑ ∑∑∈ ∈ ∈

/2/

2

,

pois, para cada Aa ∈ e Xx ∈ , existe um único 2/ nt ∈ tal que a + t = x. Assim,

∑∈

=∩+

2/2

/)(1

nt

nXXtAn

, donde o número médio de elementos de

( ) XtA ∩+ é nX / , o que claramente implica a nossa afirmação.

Agora, dado 0≥k existem 2

21 /,...,, nttt k ∈ tais que

.)1

1(,...,, 221

2 kk n

ntttAn −⋅≤+− De fato, por indução, dados tais ,,...,, 21 kttt

pela afirmação acima existe 2

1 / ntk ∈+ com

( ) ( ) ( ) ),...,,(\/1

),...,,(\)/( 212

212

1 kkk tttAnn

tttAntA +⋅≥+∩+ + ,

donde ( ) ≤+∪+=+ ++C

kkC

k tAtttAtttA )),...,,((),...,,( 121121

122

21

11

11

11),...,,( .

11

+

−=

−⋅⋅

−≤+

kkC

kn

nn

nn

tttAn

(aqui CX

denota ( ) ;\/ 2 Xn temos CX = n2 – ).X

Assim, existe ,...,, 21 ntttB = tal que 2/1

1 222 nn

nBAnn

<

−⋅≤+− , donde

.2/2nBA >+

Page 179: Eureka 2002

Sociedade Brasileira de Matemática

EUREKA! N°15, 2002

55

Obs. Para ,21

11

11

1

11,1

1

=⋅

+≥

+>

−=

−≥

n

n

nn

n

nn

nnn

donde

.2

1

1<

n

n

n

70)

P1

X1

P2 P5

X5

Q5

Q3

Q4

Q1

Q2

X3

P4 P3

X4 X2

Na figura acima, para jXj ,51 ≤≤ é o ponto de interseção dos círculos

circunscritos aos triângulos Qj –1 PjQj e Qj Pj + 1 Qj + 1 distintos de Qj (os índices sãotomados módulo 5). Prove que o pentágono X1X2X3 X4X5 é inscritível.

Obs: O pentágono P1P2P3P4P5 não é necessariamente regular.

Page 180: Eureka 2002

Sociedade Brasileira de Matemática

EUREKA! N°15, 2002

56

SOLUÇÃO DE ALEX CORRÊA ABREU (NITERÓI – RJ):

A

D

E C

B

Y

Y1

Y4

F

Y2

Y3

.

.

.

.

Lema: As circunferencias circunscritas a BDF, CEF, CDA, BEA, tem um ponto emcomum.

Prova: Seja X a intersecção das circunferencias circunscritas a BDF, CEF.

Y1, Y2, Y3, Y4, o pé das perpendiculares por Y a CECDBEBD ,,, respectivamente ⇒321 ,, YYY são colineares pela reta de Simson já que Y pertence a circunferência

circunscrita a BDF e analogamente 432 ,, YYY também são logo 4321 ,,, YYYY sãocolineares. Portanto pela recíproca da reta de Simson, Y pertence à circunferênciacircunscrita a ABE, ( 421 ,, YYY colineares) e à circunferência circunscrita a ADC

431 ,,( YYY colineares). P1

X1

P2 P5

X5

Q5

Q3

Q4

Q1

Q2

X3

P4 P3

X4 X2

Page 181: Eureka 2002

Sociedade Brasileira de Matemática

EUREKA! N°15, 2002

57

Agora 222212ˆˆ XPQQXX = pois 2221 QPXX é inscritível e 342312

ˆˆ XPQXXQ = pois

1X ∈ circunferência circunscrita a 421 PQP pelo lema e 3X também.

Analogamente 343343ˆˆ XPQXXQ = e 243212222243

ˆˆˆˆ XXQQXXQPXXXQ =⇒= e

4321342312343312ˆˆˆˆ XXXXXXXXXXXXQXXQ ⇒=⇒= é inscritível; analogamente

5432 XXXX também é, logo 54321 XXXXX é inscritível.

72) Ache todas as funções f : → tais que )),(()()( 34 yffxfxyxf +=+., ∈∀ yx

SOLUÇÃO DE RODRIGO VILLARD MILET (RIO DE JANEIRO – RJ):

))(()()( 34 yffxfxyxf +=+ (I)

Em (I) , )),(()(0 yffyfx =⇒= logo )()()( 34 yfxfxyxf +=+ (I'). Agorafazendo x = 1 e y = 0, segue que f(0) = 0. Portanto, fazendo y = 0, temos

)()( 34 xfxxf = (II) (Em particular, f é ímpar). Em (I'), )()()( 44 yfxfyxf +=+ ,donde )()()( yfxfyxf +=+ , para todo x > 0. Mas como f é ímpar, é fácil ver quevale )()()( yfxfyxf +=+ , para todos x, y reais.

Lema: Todo x real pode ser escrito (de forma única) como x = u + v, onde f(u) = u ef(v) = 0.

Prova: ))(()( xfxxfx −+= . Tome )(xfu = e )(xfxv −= . Para a unicidade,

suponha que '' vuvux +=+= . Daí vvuu −=− '' e aplicando f, segue que u = u' eassim, v = v'.Agora, em (II), escreva x = u + v, como no lema. Então:

)(3)(4)(6)(4)()())(( 2322334 vuvuuvfvufvufvufvuvuf +=++⇒++=+ (III)Podemos trocar v por –v, pois f(–v) = 0, logo:

)(3)(4)(6)(4 23223 vuvuuvfvufvuf −−=−+− (IV)

Fazendo (III) + (IV): 2

)(22

22 vuvuf = . Pelo lema, ''22 vuvu += , com f(u') = u' e f(v')

= 0. Daí segue que ''2

'')''(' vu

vuvufu =⇒+=+= . Aplicando f, segue que u' = v' = 0,

ou seja 0≡u ou 0≡v , o que nos dá respectivamente as soluções 0)( ≡xf e

xxf ≡)( , que claramente satisfazem a equação (I).

Page 182: Eureka 2002

Sociedade Brasileira de Matemática

EUREKA! N°15, 2002

58

Agradecemos também o envio das soluções e a colaboração de:

Rildo Alves do Nascimento Santa Maria da Boa Vista – PELeno Silva Rocha Goiânia – GOMinh Perez Rehoboth, EUACarlos Alberto da Silva Victor Nilópolis – RJ

Seguimos aguardando o envio de soluções dos problemas propostos Nº. 68, 69, 71 e 73publicado na revista Eureka! Nº. 14

n n ⋅ ln n ! "

# $%& ' () '*+, - . n

n/ 0 *1 http://www.cse.iitk.ac.in/news/primality.html

http://www.utm.edu/research/primes/prove/prove4_3.html

Page 183: Eureka 2002

Sociedade Brasileira de Matemática

EUREKA! N°15, 2002

59

PROBLEMAS PROPOSTOS

Convidamos o leitor a enviar soluções dos problemas propostos e sugestões de novosproblemas para os próximos números.

74) Ache todas as funções →:f tais que: ,cos)(2)()( yxfyxfyxf ⋅=−++ ., ∈∀ yx

75) Seja Tn um triângulo retângulo cujos lados medem )14,14,4( 442 +⋅−⋅⋅ nnn ,

onde n é um número inteiro positivo. Seja nα a medida do ângulo oposto ao

lado de medida 24 n⋅ . Mostre que, se n varia dentro dos inteiros positivos,

.90... 0321 =+++ ααα

76) Mostre que um polígono qualquer pode ser recortado e os recortesreorganizados, sem superposição, de tal jeito que formem um quadrado.

77) Prove que as distâncias entre um ponto sobre uma circunferência e os quatrovértices de um quadrado nesta inscrita não podem ser todos números racionais.

78) Seja ABCD um quadrilátero convexo não trapézio, de diagonais AC e BD iguais.Tomamos sobre os lados AB e CD, respectivamente, pontos P e Q tais que:

BC

AD

QC

DQ

PB

AP ==

Mostre que os pontos P e Q são colinares com o ponto de interseção das mediatrizes dos lados AD e BC.

79) Temos uma fileira infinita de copos, cada um deles associado a um inteiro k, eum número finito de pedras distribuídas de alguma maneira por esses copos. Sehá pelo menos duas pedras no copo k podemos pular uma pedra para o copok – 1 e outra para o copo k + 1.Prove que fazendo movimentos desse tipo um número suficientemente grande devezes, chega-se necessariamente a uma situação onde não é possível fazernenhum movimento desse tipo (i.e., onde há no máximo uma pedra em cadacopo), e que a configuração final não depende da escolha dos movimentosdurante o processo.

Page 184: Eureka 2002

Sociedade Brasileira de Matemática

EUREKA! N°15, 2002

60

80) Sejam 2

51+=α , *, ∈= nnA α e *,2∈= nnB α . Prove que

=∩ BA ∅ e *=∪ BA .

Obs. x é o inteiro tal que .1+<≤ xxx

Problema 74 proposto por Gibran M. de Souza (Natal - RN); Problema 75 proposto por Carlos A.Gomes (Natal - RN); Problema 76 proposto por Eduardo Casagrande Stabel (Porto Alegre - RS);Problemas 77 e 78 propostos por Evandro Makiyama de Melo (São Paulo - SP).

Page 185: Eureka 2002

Sociedade Brasileira de Matemática

EUREKA! N°15, 2002

61

COORDENADORES REGIONAIS

Alberto Hassen Raad (UFJF) Juiz de Fora – MG

Amarísio da Silva Araújo (UFV) Viçosa – MG

Benedito Tadeu Vasconcelos Freire (UFRN) Natal – RN

Carlos Frederico Borges Palmeira (PUC-Rio) Rio de Janeiro – RJ

Claudio Arconcher (Colégio Leonardo da Vinci) Jundiaí – SP

Claus Haetinger (UNIVATES) Lajeado – RS

Cleonor Crescêncio das Neves (UTAM) Manaus – AM

Élio Mega (Colégio Etapa) São Paulo – SP

Florêncio Ferreira Guimarães Filho (UFES) Vitória – ES

Ronaldo Alves Garcia (UFGO) Goiânia – GO

Alexandra de Oliveira Abdala Cousin (UEM) Maringá – PR

Ivanilde Fernandes Saad (UC. Dom Bosco) Campo Grande– MS

Jacqueline Fabiola Rojas Arancibia (UFPB) João Pessoa – PB

João Benício de Melo Neto (UFPI) Teresina – PI

João Francisco Melo Libonati (Grupo Educacional Ideal) Belém – PA

José Carlos Pinto Leivas (UFRG) Rio Grande – RS

José Cloves Saraiva (UFMA) São Luis – MA

José Gaspar Ruas Filho (ICMC-USP) São Carlos – SP

José Luiz Rosas Pinho (UFSC) Florianópolis – SC

José Vieira Alves (UFPB) Campina Grande – PB

Licio Hernandes Bezerra (UFSC) Florianópolis – SC

Luzinalva Miranda de Amorim (UFBA) Salvador – BA

Marcelo Rufino de Oliveira (Grupo Educacional Ideal) Belém – PA

Marcondes Cavalcante França (UFC) Fortaleza – CE

Pablo Rodrigo Ganassim (Liceu Terras do Engenho) Piracicaba – SP

Ramón Mendoza (UFPE) Recife – PE

Reinaldo Gen Ichiro Arakaki (INPE) SJ dos Campos – SP

Ricardo Amorim (Centro Educacional Logos) Nova Iguaçu – RJ

Roberto Vizeu Barros (Colégio Acae) Volta Redonda – RJ

Rosângela Souza (Colégio Singular) Santo André – SP

Sérgio Cláudio Ramos (IM-UFRGS) Porto Alegre – RS

Tadeu Ferreira Gomes (UEBA) Juazeiro – BA

Tomás Menéndez Rodrigues (U. Federal de Rondônia) Porto Velho – RO

Valdenberg Araújo da Silva (U. Federal de Sergipe) São Cristovão – SE

Wagner Pereira Lopes (CEFET – GO) Jataí – GO